БХ англ тесты

You might also like

Download as pdf or txt
Download as pdf or txt
You are on page 1of 123

hange E hange E

XC di XC di
F- t F- t
PD

PD
or

or
!

!
W

W
O

O
N

N
Y

Y
U

U
B

B
to

to
ww

ww
om

om
k

k
lic

lic
C

C
.c

.c
w

w
tr re tr re
.

.
ac ac
k e r- s o ft w a k e r- s o ft w a

Ministry of Public Health of Ukraine

O. O. BOGOMOLETS NATIONAL MEDICAL UNIVERSITY

Department of Bioorganic and Biological Chemistry

LIST OF TEST QUESTIONS

for preparation of Content module № 2

"General regularities of metabolism of


carbohydrates, lipids, amino acids"
FOR STUDENTS OF THE 2ST YEAR OF STUDY

OF MEDICAL and STOMATOLOGICAL FACULTIES

Kyiv-2018
hange E hange E
XC di XC di
F- t F- t
PD

PD
or

or
!

!
W

W
O

O
N

N
Y

Y
U

U
B

B
to

to
ww

ww
om

om
k

k
Enzymes and coenzymes. Regulation of metabolism
lic

lic
C

C
.c

.c
w

w
tr re tr re
.

.
ac ac
k e r- s o ft w a k e r- s o ft w a
1. A denaturation of proteins can befound in B. Aspartate, glutamate, glycine
some substances. Specify thesubstance that is C. Aspartate, arginine, glutamate
used for the incompletedenaturation of D. Glutamate, valine, leucine
hemoglobin: E. Cysteine, glycine, proline
A. Urea
B. Toluene 7. Parodontitis is accompanied byactivation of
C. Sulfuric acid proteolysis in parodentiumtissues. Increase of
D. Nitric acid what oral fluid’scomponent is the evidence of
E. Sodium hydroxide proteolysisactivation?
A. Aminoacids
2. Some proteins of saliva have aprotective B. Organic acids
function. Which of themprotects the oral C. Glucose
mucosa from themechanical damage? D. Biogenic amines
A. Mucin E. Cholesterol
B. Lysozyme
C. Catalase 8. The student used conserved donor blood to
D. Peroxidase determine the time of its clotting. However, any
E. Renin positive result had not been obtained. The
reason for this is the lack in this sample of
3. For the study of serum proteinsvarious blood:
physical and physicochemicalmethods can be A. Ionized calcium
used. In particular, serumalbumins and B. Factor Hageman
globulins can be separatedby this method: C. Thromboplastin
A. Electrophoresis D. Fibrinogen
B. Polarography E. Vitamin K
C. Dialysis
D. Spectrography 9. Long-term effects on the body of toxic
E. Refractometry substances led to a significant reduction in
protein synthesis in hepatocytes. What
4. Electrophoretic study of blood serumof a organelles suffered from intoxication most?
patient with pneumonia revealed anincrease in A. Granular endoplasmic reticulum
one of the protein fractions.What fraction is it? B. Mitochondria
A. γ-globulins C. Microtubules
B. Albumins D. Lysosomes
C. α1-globulins E. The Golgi’s complex
D. α2-globulins
E. β-globulins 10. Children with kwashiorkor, along with other
signs, violations of the process of dentition were
5. A woman, 36 years after surgery, received revealed. At the heart of this phenomenon is the
intravenous injection of a concentrated albumin insufficient receipt into the body:
solution. This resulted in the increased A. Lipids
movement of water in the following direction: B. Proteins
A. From intercellular fluid to capillaries C. Carbohydrates
B. From the intercellular fluid to the cells D. Vitamin C
C. From cells to the intercellular fluid E. Vitamin B1
D. From capillaries to intercellular fluid
E. There will be no change in the flow of water 11. In the daily diet of an adult healthy person,
there must be fats, proteins, carbohydrates,
6. Cationic glycoproteins are the vitamins, mineral salts and water. Specify the
majorcomponents of parotid saliva. What daily amount of protein (g) that provides normal
aminoacids are responsible for their life activity of the body:
positivecharge? A. 100-120
A. Lysine, arginine, histidine B. 50-60
hange E hange E
XC di XC di
F- t F- t
PD

PD
or

or
!

!
W

W
O

O
N

N
Y

Y
U

U
B

B
to

to
ww

ww
om

om
C. 10-20 E. Glucose
k

k
lic

lic
C

C
.c

.c
w

w
tr re tr re
.

.
ac ac
k e r- s o ft w a k e r- s o ft w a
D. 70-80
E. 40-50 17. 60-year-old man went to the doctor after the
appearance of pain in the chest. In blood serum
12. A patient with a hepatic failure has been showed a significant increase in the activity of
examined for the electrophoretic spectrum of enzymes: CPK and its MB isoform, aspartate
blood serum proteins. Which of the physico- aminotransferase. On the development of the
chemical properties of the protein molecules lie pathological process in which tissue do these
in the basis of this method? changes indicate?
A. Presence of a charge A. Heart muscle
B. Hydrophilicity B. Lungs
C. Ability to swell C. Skeletal muscle
D. Optical activity D. Liver
E. Non-dialysable E. Smooth muscles

13. The surgeon used a 70% solution of ethyl 18. There are several groups of molecular
alcohol to disinfect the hands before surgery. mechanisms that are important in the
What is the main mechanism of antiseptic action pathogenesis of cell damage, which contributes
of the drug on microorganisms? to the development of pathology. What
A. Denaturation of protoplasmic proteins. processes provide protein damage mechanisms?
B. Interaction with amino groups of A. Inhibition of enzymes
protoplasmic proteins. B. Peroxide oxidation of lipids
C. Interaction with hydroxyl groups of enzymes. C. Acidosis
D. Blockade of sulfhydryl groups of enzyme D. Osmotic stretching of membranes
systems. E. Activation of phospholipases
E. Oxidation of organic components of
protoplasm. 19. It is known that the activity of parotid
salivary glands susceptible to age-related
14. What substance makes saliva viscousand changes. The activity of which enzyme will
mucous, has protective function, protects decrease sharply in saliva?
mucous membrane of oralcavity from A. Amylase.
mechanical damage? B. Hexokinase.
A. Mucin C. Renin.
B. Glucose D. Maltase.
C. Kallikrein E. Phosphatase.
D. Amylase
E. Lysozyme 20. For biochemical diagnostics of cardiac
infarction it is necessary to determine activity of
15. Only one factor can affect the charge of a number of enzymes and their izoenzymes in
amino acid radicals in the active center of the the blood. What enzyme assay is considered to
enzyme. Name of this factor is: be optimal for confirming or ruling out cardiac
A. pH infarction at the early stage, after the patient
B. Pressure develops thoracic pain?
C. Temperature A. Creatine kinase MB isoenzyme
D. The presence of a competitive inhibitor B. Creatine kinase MM isoenzyme
E. Excessproduct C. LDH1 isoenzyme
D. LDH5 isoenzyme
16. The conjugated protein necessarily contains E. Cytoplasmic isoenzyme of
special component as a nonprotein part. Choose aspartateaminotransferase
the substance that can’t carry out this function:
A. HNO3 21. For the treatment of some infectious
B. АТP diseases caused by bacteria, sulfonamides drugs
C. Thiamine pyrophosphate are used. What is the mechanism of action of
D. AMP these drugs?
hange E hange E
XC di XC di
F- t F- t
PD

PD
or

or
!

!
W

W
O

O
N

N
Y

Y
U

U
B

B
to

to
ww

ww
om

om
A. They are antivitamins of p-aminobenzoic the most likely development of the pathological
k

k
lic

lic
C

C
.c

.c
w

w
tr re tr re
.

.
ac ac
k e r- s o ft w a k e r- s o ft w a
acid. process?
B. They are alosteric enzyme inhibitors. A. Heart
C. They are alosteric enzyme activators. B. Pancreas
D. They are participate in redox processes. C. Liver
E. They are inhibit folic acid absorption. D. Kidneys
E. Skeletal muscle
22. In the intensive care unit entered a man 47-
years old with a diagnosis of myocardial 27. An increase in the activity of LDH 4,5,
infarction. Which of the lactate dehydrogenase AlAT, carbamoyl ornithine transferase was
(LDH) fractions will prevail in the blood serum detected in the patient's blood. In what organ
during the first two days? can assume development of the pathological
A. LDH 1 process?
B. LDH 2 A. Cardiac muscle (possible myocardial
C. LDH 3 infarction)
D. LDH 4 B. Liver (possible hepatitis)
E. LDH 5 C. Skeletal muscle
D. Kidneys
23. Researchers isolated 5 isoenzymicforms of E. Connective tissue
lactate dehydrogenase from thehuman blood
serum and studied theirproperties. What 28. Albumin concentration in the patient’s blood
property indicates thattheisoenzymic forms is 2.8 g/l, increased concentration of lactate
were isolated fromthe same enzyme? dehydrogenase 5 (LDH5). On the disease of
A. Catalyzation of the same reaction what organ does that indicate?
B. The same molecular weight A. Liver
C. The same physicochemical properties B. Kidney
D. Tissue localization C. Heart
E. The same electrophoretic mobility D. Lung
E. Spleen
24. One of the means of regulating enzyme
activity in a human body is the covalent 29. There is increased activity of AST, LDH1,
modification. Glycogen phosphorylase and LDH2, and CPK in the patient’s blood.
glycogen synthetase activity is regulated by the Pathological process most likely occurs in the:
following type of covalent modification: A. Heart
A. Phosphorylation-dephosphorylation B. Skeletal muscles
B. ADP-ribosylation C. Kidneys
C. Methylation D. Liver
D. Hydrolysis E. Adrenal glands
E. Sulfonation
30. In the patient's blood plasma the activity of
25. When the patient's blood analysis revealed a LDH1 and LDH2 isoenzymes increased. About
significant increase in the activity of MB-forms which pathology of the body does this mean?
CPK (CPK) and lactate dehydrogenase-1. What A. Myocardium
pathology can be assumed? B. Liver
A. Myocardial infarction C. Kidney
B. Hepatitis D. Brain
C. Rheumatism E. Skeletal muscle
D. Pancreatitis
E. Cholecystitis 31. In a patient revealed increased the activity of
AsAT, LDH1,2, and creatine phosphokinase. In
26. An increase in the activity of LDH1, LDH2, which organ (s) is the most likely development
AsAT, and creatine kinase was detected in the of the pathological process?
patient's blood. In which organ of the patient is A. Heart muscle
B. Skeletal muscle
hange E hange E
XC di XC di
F- t F- t
PD

PD
or

or
!

!
W

W
O

O
N

N
Y

Y
U

U
B

B
to

to
ww

ww
om

om
C. Kidney and adrenal gland D. LDH 4
k

k
lic

lic
C

C
.c

.c
w

w
tr re tr re
.

.
ac ac
k e r- s o ft w a k e r- s o ft w a
D. Connective tissue E. LDH 5
E. Liver and kidneys
34. The high level of lactate dehydrogenase
32. 49-year-old patient, a driver by profession, (LDH) isozymes concentration showed the
complains of unbearable compressive pain increase of LDH-1 and LDH-2 in a patient’s
behind the sternum, "giving away" to the neck. blood plasma. Point out the most probable
Pain occurred 2 hours ago. Objectively: the diagnosis:
condition is severe, pallor, heart sounds are A. Myocardial infarction
weakened. Laboratory studies have shown high B. Skeletal muscle dystrophy
activity of creatine kinase and LDH1. What C. Diabetes mellitus
diseases are characterized by such symptoms? D. Viral hepatitis
A. Acute myocardial infarction E. Acute pancreatitis
B. Acute pancreatitis
C. Stenocardia 35. Succinate dehydrogenase catalyses the
D. Cholelithiasis dehydrogenation of succinate. Malonic acid
E. Diabetes HOOC − CH2− COOH is used to interrupt the
action of this enzyme. Choose the inhibition
33. Six hours after myocardial infarction, the type:
lactate dehydrogenase activity in the patient A. Competitive
increased in the blood. What isoenzymes should B. Allosteric
be expected in this case? C. Non-competitive
A. LDH 1 D. Limited proteolysis
B. LDH 2 E. Dephosphorylation
C. LDH 3
hange E hange E
XC di XC di
F- t F- t
PD

PD
or

or
!

!
W

W
O

O
N

N
Y

Y
U

U
B

B
to

to
ww

ww
om

om
k

k
Water-soluble vitamins
lic

lic
C

C
.c

.c
w

w
tr re tr re
.

.
ac ac
k e r- s o ft w a k e r- s o ft w a
1. Examination of a patient revealed dermatitis, C. Retinol
diarrhea, dementia. What vitamin deficiency is D. Calciferol
the cause of this condition? E. Riboflavin
A. Nicotinamide
B. Ascorbic acid 6. A patient has painfulness along bignerve
C. Folic acid trunks and excessive content of pyruvate in
D. Biotin blood. What vitamin deficit may cause such
E. Rutin changes?
A. В1
2. A patient consulted a doctor about symmetric B. Biotin
dermatitis of open skin areas. It was found out C. РР
that the patient lived mostly on cereals and ate D. Pantothenic acid
too little meat, milk and eggs. What vitamin E. В2
deficiency is the most evident?
A. Nicotinamide 7. A woman who has been keeping to a clean-
B. Calciferol rice diet for a long time was diagnosed with
C. Folic acid polyneuritis (beri-beri). What vitamin deficit
D. Biotin results in development of this disease?
E. Tocopherol A. Thiamine
B. Ascorbic acid
3. A 3 year old child with symptoms of C. Pyridoxine
stomatitis, gingivitis and dermatitis of open skin D. Folic acid
areas was delivered to a hospital. Examination E. Riboflavin
revealed inherited disturbance of neutral amino
acid transporting in the bowels. These 8. A patient has an increased pyruvate
symptoms were caused by the deficiency of the concentration in blood. A large amount of it is
following vitamin: excreted with the urine. What vitamin is lacking
A. Niacin in this patient?
B. Pantothenic acid A. B1
C. Vitamin A B. E
D. Cobalamin C. B3
E. Biotin D. B6
E. B2
4. A patient is ill with dermatitis, diarrhea,
dementia. During history taking it was revealed 9. A patient, who has been subsisting
that the main food stuff of the patient was exclusively on polished rice, has developed
maize. These disturbances are caused by polyneuritis due to thiamine deficiency. What
deficiency of the following vitamin: substance is an indicator of such avitaminosis,
A. PP when it is excreted with urine?
B. B1 A. Pyruvic acid
C. B2 B. Malate
D. B9 C. Methylmalonic acid
E. B8 D. Uric acid
E. Phenyl pyruvate
5. A 50-year-old man addressed a hospital
withcomplaints of memory disorders, painful 10. Vitamin B1 deficiency causes disturbance of
sensations along the nerve trunks, decreased oxidative decarboxylation of α-ketoglutaric
mental ability, circulatory disorders and acid. This leads to the impaired synthesis of the
dyspepsia. Anamnesis states excessive alcohol following coenzyme:
consumption. What vitamin deficiency can A. Thiamine pyrophosphate
result in such signs? B. Nicotinamide adenine dinucleotide
A. Thiamine C. Flavine adenine dinucleotide
B. Niacin D. Lipoic acid
hange E hange E
XC di XC di
F- t F- t
PD

PD
or

or
!

!
W

W
O

O
N

N
Y

Y
U

U
B

B
to

to
16. In case of enterobiasis acrihine – the
ww

ww
om

om
E. Coenzyme A
k

k
lic

lic
C

C
.c

.c
w

w
tr re tr re
.

.
structural analogue of vitamin B2 – cisk e r- s o ft w a
ac a
k e r- s o ft w a

11. To improve the trophism of the heart administered. The synthesis disorder of which
muscle, the patient is prescribed a drug that enzymes does this medicine cause in
includes cocarboxylase (thiamine diphosphate) - microorganisms?
a coenzyme form of the vitamin: A. FAD-dependent dehydrogenases
A. B1 B. Cytochromeoxidases
B. B2 C. Peptidases
C. B5 D. NAD-depended dehydrogenases
D. B6 E. Aminotransferases
E. B12
17. The preparation comlex for periodontitis
12. Pyruvic acid as an intermediate metabolite treatment includes the medicine from the group
of carbohydrate, lipid and amino acid of water soluble vitamins, bioflavonide
metabolism can undergo oxidative derivative, which is prescribed together with
decarboxylation. The cause of this process is the ascorbic acid. This preparation has anti-
lack of the following nutrient in the diet: oxidative properties, decreases gingival
A. Thiamin hemorrhage. What preparation is meant?
B. Pyridoxine A. Rutin
C. Ascorbic acid B. Calcium pantothenate
D. Citrine C. Calcium panganate
E. Pangamic acid D. Cyanocobalamin
E. Folic acid
13. A patient with chronic alcoholism has
symptoms of polyneuritis and cardiac 18. Examination of a child who hasn’t got fresh
insufficiensy. What vitamin preparation should fruit and vegetables during winter revealed
be prescribed to this patient? numerous subcutaneous hemorrhages,
A. Thiamine gingivitis, carious cavities in teeth. What
B. Ergocalciferol vitamin combination should be prescribed in
C. Retinol this case?
D. Rutin A. Ascorbic acid and rutin
E. Phylloquinone B. Thiamine and pyridoxine
C. Folic acid and cobalamin
14. A 36-year-old female patient has a history of D. Riboflavin and nicotinamide
B2-hypovitaminosis. The most likely cause of E. Calciferol and ascorbic acid
specific symptoms (epithelial, mucosal,
cutaneous, corneal lesions) is the deficiency of: 19. Examination of a patient with frequent
A. Flavin coenzymes hemorrhages from internals and mucous
B. Cytochrome A1 membranes revealed proline and lysine being a
C. Cytochrome oxidase part of collagene fibers. What vitamin absence
D. Cytochrome B caused disturbance of their hydroxylation?
E. Cytochrome C A. Vitamin С
B. Vitamin K
15. Malaria is treated with structural analogs of C. Vitamin A
vitamin B2 (riboflavin). These drugs disrupt the D. Thiamine
synthesis of the following enzymes in E. Vitamin Е
plasmodium:
A. FAD-dependent dehydrogenase 20. In spring a patient experiences petechial
B. NAD-dependent dehydrogenase haemorrhages, loosening of teeth, high liability
C. Peptidase to colds. A doctor suspects hypovitaminosis C.
D. Cytochrome oxidase In this respect loosening of teeth can be
E. Aminotransferase explained by:
A. Structural failure of collagen in the
periodontal ligaments
hange E hange E
XC di XC di
F- t F- t
PD

PD
or

or
!

!
W

W
O

O
N

N
Y

Y
U

U
B

B
to

to
ww

ww
om

om
B. Structural change of glycosaminoglycans 25. A patient diagnosed with focal tuberculosis
k

k
lic

lic
C

C
.c

.c
w

w
tr re tr re
.

.
ac ac
k e r- s o ft w a k e r- s o ft w a
C. Increased permeability of periodont of the upper lobe of the right lung had been
membranes taking isoniazid as a part of combination
D. Mechanical damage of teeth therapy. After some time, the patient reported of
E. Disturbed oxidation-reduction process in the muscle weakness, decreased skin sensitivity,
periodont blurred vision, impaired motor coordination.
Which vitamin preparation should be used to
21. Most of the members of the expedition of address these phenomena?
Magellan to America died from vitamin A. Vitamin B6
deficiency. This disease was manifested by B. Vitamin A
general weakness, subcutaneous hemorrhage, C. Vitamin D
tooth loss, bleeding from the gums. As the name D. Vitamin B12
of this vitamin deficiency? E. Vitamin C
A. scurvy (scurvy)
B. Pellagra 26. A 40-year-old male patient with pulmonary
C. Rickets tuberculosis was administered isoniazid. What
D. Polyneuritis (beriberi) vitamin deficiency can develop as a result of
E. Biermer’s anemia taking this drug for a long time?
A. Pyridoxine.
22. A 10-year-old girl has a history of repeated B. Cobalamin.
acute respiratory viral infection. After C. Biotin.
recovering she presents with multiple petechial D. Thiamine.
hemorrhages on the sites of friction from E. Folic acid.
clothing rubbing the skin. What kind of
hypovitaminosis has this girl? 27. What vitamin is a component of glutamic
A. C acid decarboxylase, participates in the
B. B6 production of GABA, and its deficiency is
C. B1 manifested by seizures?
D. A A. Pyridoxine
E. B2 B. Cobalamin
C. Tocopherol
23. The patient has an increase in the D. Folic acid
permeability of the walls of blood vessels with E. Ascorbic acid
the development of increased bleeding and the
appearance of small point hemorrhages on the 28. A 9-month-old child feeds on artificial
skin, tooth loss. How does the vitamin exchange mixtures that are not balanced in terms of
disorder explain these symptoms? vitamin B6 content. A child has pellagra similar
A. Hypovitaminosis C dermatitis, convulsions, anemia. The
B. Hypervitaminosis D development of seizures may be associated with
C. Hypervitaminosis C impaired formation:
D. Hypovitaminosis D A. GABA
E. Hypovitaminosis A B. Histamine
C. Serotonin
24. A 20-year-old male patient complains of D. DOPA
general weakness, rapid fatigability, irritability, E. Dopamine
decreased performance, bleeding gums,
petechiae on the skin. What vitamin deficiency 29. The infant has epileptiform convulsions
may be a cause of these changes? caused by vitamin B6 deficiency. This is due to
A. Ascorbic acid a decrease in the nerve tissue of the inhibitory
B. Riboflavin mediator - γ-aminobutyric acid. What enzyme
C. Thiamine activity is reduced due to this?
D. Retinol A. Glutamate decarboxylase
E. Folic acid B. Alanine aminotransferase
C. Glutamate dehydrogenase
hange E hange E
XC di XC di
F- t F- t
PD

PD
or

or
!

!
W

W
O

O
N

N
Y

Y
U

U
B

B
to

to
ww

ww
om

om
D. Pyridoxal kinase B. Carboxybiotin
k

k
lic

lic
C

C
.c

.c
w

w
tr re tr re
.

.
ac ac
k e r- s o ft w a k e r- s o ft w a
E. Glutamate synthetase C. Thiamine diphosphate
D. Pyridoxal phosphate
30. In compliance with the clinical presentations E. Nicotinamide adenine dinucleotide
a man was prescribed pyridoxalphosphate. What
processes are corrected by this preparation? 35. Pterin derivatives (aminopterin and
A. Transamination and decarboxylation of methotrexate) are the inhibitors of dihydrofolate
amino acids reductase, so that they inhibit the regeneration
B. Oxidative decarboxilation of keto acids of tetrahydrofolic acid from dihydrofolate.
C. Desamination of purine nucleotides These drugs inhibit the intermolecular tranfer of
D. Synthesis of purine and pyrimidine bases monocarbon groups, thus suppressing the
E. Protein synthesis synthesis of the following polymer:
A. DNA
31. In the clinical practice for the treatment of B. Protein
tuberculosis drug isoniazid used - antivitamin C. Homopolysaccharides
which is able to penetrate into the tubercle D. Gangliosides
bacillus. The tuberculostatic effect is due to the E. Glycosaminoglycans
violation of replication processes, redox
reactions, due to the formation of not real 36. After an extended treatment with
coenzyme from: sulfanamides a patient has developed
A. NAD+ macrocyticanemia. Production of active forms
B. FAD of the following vitamin is disrupted in such a
C. FMN condition:
D. TDP A. Folic acid
E. KoQ B. Thiamine
C. Riboflavin
32. Reactions of intermolecular transport of D. Pyridoxine
one-carbon radicals are necessary for the E. Cyanocobalamin
synthesis of proteins and nucleic acids. From
which of the following vitamins does the 37. It is known that part of carbon dioxide is
coenzyme form necessary for the above used in the body in the biosynthesis of fatty
reactions? acids, urea, gluconeogenesis, etc. Which
A. Folic acid vitamin forms the CO2-transporting form for
B. Thiamine these reactions?
C. Pantothenic acid A. Biotin
D. Ascorbic acid B. thymine
E. Riboflavin C. Riboflavin
D. Nicotinamide
33. In the synthesis of purine nucleotides E. Retinol
involved some amino acids, derivatives of
vitamins, phosphorus esters of ribose. What 38. A patient was diagnosed with seborrheic
coenzyme form of vitamin provides one-carbon dermatitis associated with vitamin H (biotin)
fragments for this synthesis? deficiency. The patient has disturbed activity of
A. Folic acid the following enzyme:
B. Pantothenic acid A. Acetyl-CoA-carboxylase
C. Nicotinic acid B. Pyruvate decarboxylase
D. Riboflavin C. Alcohol dehydrogenase
E. Pyridoxine D. Amino transferase
E. Carbomoyl phosphate synthetase
34. In the normal course of a replication process
need thymidylic nucleotides, the synthesis of 39. The patient was diagnosed megaloblastic
which occurs with the participation of anemia. Specify a vitamin deficiency which can
thymidylate synthetase, are used as a coenzyme: lead to the development of this disease.
A. Methylenetetrahydrofolate A. Cyanocobalamin.
hange E hange E
XC di XC di
F- t F- t
PD

PD
or

or
!

!
W

W
O

O
N

N
Y

Y
U

U
B

B
to

to
ww

ww
om

om
B. Rutin. function, which is accompanied by anemia.
k

k
lic

lic
C

C
.c

.c
w

w
tr re tr re
.

.
ac ac
k e r- s o ft w a k e r- s o ft w a
C. Nicotinamide. What substance deficiency causes the
D. Thiamine. development of B12 hypovitaminosis in a patient
E. Cholecalciferol. and the appearance of anemia?
A. Castle factor
40. A patient 43 years old with chronic atrophic B. Thiamine
gastritis and hyperchromic megaloblastic C. Biotin
anemia increased methylmalonic acid excretion D. Pyridoxine
in the urine. The lack of which vitamin caused E. Calciferol
the occurrence of this symptom complex?
A. В12 45. After the surgical removal of part of the
B. В2 stomach of the patient disrupted the absorption
C. В3 of vitamin B12, it is excreted in the feces.
D. В5 Anemia has developed. What factor is necessary
E. В6 for the absorption of this vitamin?
A. Gastromukoprotein
41. After removing 2/3 of the stomach in the B. Gastrin
patient’s blood, the hemoglobin content C. Hydrochloric acid
decreased, the number of red blood cells D. Pepsin
increased, the size of these blood cells E. Folic acid
increased. What vitamin deficiency leads to
such changes in the blood? 46. In a 65-year-old patient with long-lasting
A. B12 complaints characteristic of chronic gastritis,
B. C megalocytes were found in peripheral blood and
C. P megaloblastic erythropoiesis in the bone
D. B6 marrow. What is the most likely diagnosis?
E. PP A. B12-folic deficiency anemia
B. Aplastic anemia
42. In examining the oral cavity of the patient, C. Hypoplastic anemia
the dentist paid attention to the presence of an D. Hemolytic anemia
inflammatory and dystrophic process in the E. Iron deficiency anemia
mucous membrane (Hunter’s glossitis, atrophic
stomatitis). A blood test revealed hyperchromic 47. A 13-year-old girl has been prescribed a
anemia. What factor is the cause of this disease? certain drug for treatment of megaloblastic
A. Hypovitaminosis B12 anemia. This drugstimulates a transfer from
B. Hypovitaminosis B1 megaloblastic haemopoiesis to normoblastic,
C. Hypovitaminosis B6 participates in synthesis of purine and
D. Increasing pH of the gastric juice pyrimidine bases, activates proteine and
E. Hypovitaminosis A methionine synthesis.What drug does the patient
take?
43. A year after subtotal stomach resection on A. Cyanocobalamin
account of ulcer of lesser curvature the B. Erythropoietin
following blood changes were revealed: anemia, C. Haemostimulinum
leukocytopenia and thrombocytopenia, color D. Ferricsulfate
index - 1,3, megaloblasts and megalocytes. E. Rosehiptea
What factor deficiency caused the development
of thos pathology? 48. A 50-year-old patient has been examined by
A. Castle’s factor a dentist and found to have crimson smooth
B. Hydrochloride acid tongue. Blood analysis revealed a decrease in
C. Mucin RBC level and hemoglobin concentration,
D. Pepsin colour index of 1,3, symptoms of megaloblastic
E. Gastrin hematopoiesis, degenerative changes in
44. In the patient with complaints about pain in WBCs.What blood disorder was found in this
the stomach found a decrease in its secretory patient?
hange E hange E
XC di XC di
F- t F- t
PD

PD
or

or
!

!
W

W
O

O
N

N
Y

Y
U

U
B

B
to

to
ww

ww
om

om
A. B12-folic-acid-deficiency anemia
k

k
lic

lic
C

C
.c

.c
w

w
tr re tr e
ar
.

.
ac ac
k e r- s o ft w a
B. Iron deficiency anemia 50. A doctor recommends a patient withk e r- s o ft w
C. Myeloid leukemia duodenal ulcer to drink cabbage and potato juice
D. Aplastic anemia after the therapy course. Which substances
E. Hemolytic anemia contained in these vegetables help to heal and
prevent the ulcers?
49. Coenzym A participates in numerous A. Vitamin U
important metabolic reactions. It is aderivative B. Pantothenic acid
of the following vitamin: C. Vitamin C
A. Pantothenic acid D. Vitamin B1
B. Thiamine E. Vitamin K
C. Niacin
D. Calciferol
E. Ubiquinone
hange E hange E
XC di XC di
F- t F- t
PD

PD
or

or
!

!
W

W
O

O
N

N
Y

Y
U

U
B

B
to

to
ww

ww
om

om
k

k
Fat -soluble vitamins
lic

lic
C

C
.c

.c
w

w
tr re tr re
.

.
ac ac
k e r- s o ft w a k e r- s o ft w a
1. A 2 year old child suffers fromintestinal blood - decrease in activity II, VII, X blood
dysbacteriosis that lead to thedevelopment clotting factors; lengthening of blood clotting
hemorrhagic syndrome. Themost probable cause time. What vitamin deficiency are caused by
of hemorrhage is: these changes?
A. Vitamin K deficiency A. Vitamin K
B. Activation of tissue thromboplastin B. Vitamin A
C. Hypovitaminosis PP C. Vitamin C
D. Fibrinogen deficiency D. Vitamin D
E. Hypocalcemia E. Vitamin E

2. Surgery in patients with obstructive jaundice 7. Plasma coagulation factors are subject to
and malabsorption in the intestine complicated posttranslational modification with the
by bleeding. What vitamin insufficiency has led participation of vitamin K. As a cofactor, it is
to this? needed in the γ-carboxylation enzyme system of
A. Vitamin K protein coagulation factors due to an increase in
B. Vitamin B12 the affinity of their molecules by calcium ions.
C. Vitamin C What amino acid is carboxylated in these
D. Vitamin B6 proteins?
E. Folic acid A. Glutamine
B. Valin
3. Hepatic disfunctions accompaniedby C. Serine
insufficient in flow of bile to the bowels result D. Phenylalanine
in coagulation failure. This phenomenon can be E. Arginine
explained by:
A. Vitamin K deficiency 8. A 49-year-old man is observed in the clinic
B. Leucopenia with a significant increase in blood clotting
C. Thrombocytopenia time, gastrointestinal bleeding, subcutaneous
D. Erythropenia hemorrhage. What vitamin deficiency can
E. Iron deficiency explain these symptoms?
A. K
4. The function of protein synthesis in the liver B. B1
due to a lack of vitamin K is reduced in a young C. PP
man of 16 years after suffering the disease. This D. H
can lead to a violation of: E. E
A. Blood coagulation
B. Erythrocyte sedimentation rate 9. As a result of post-translational modifications
C. Education anticoagulants of certain proteins involved in blood
D. Formation of erythropoietin coagulation, in particular prothrombin, they
E. Blood pH acquire the ability to bind calcium. In this
process participate vitamin
5. In the patient observed hemorrhage, reduced A. K
the concentration of prothrombin in the blood. B. C
What vitamin deficiency led to a violation of the C. A
synthesis of this blood clotting factor? D. B1
A. K E. B2
B. A
C. D 10. To prevent postoperative bleeding, a 6-year-
D. C old child is recommended to take vikasol, which
E. E is a synthetic analogue of vitamin K. Specify
which post-translational changes of blood
6. In the patient 37 years against the background clotting factors are activated under the influence
of long-term use of antibiotics observed of vikasol?
increasing bleeding after small injuries. In the A. Carboxylation of glutamic acid
hange E hange E
XC di XC di
F- t F- t
PD

PD
or

or
!

!
W

W
O

O
N

N
Y

Y
U

U
B

B
to

to
ww

ww
om

om
B. Phosphorylation of serine radicals 15. In the patient with renal insufficiency has
k

k
lic

lic
C

C
.c

.c
w

w
tr re tr re
.

.
ac ac
k e r- s o ft w a k e r- s o ft w a
C. Partial proteolysis developed osteodystrophy, accompanied by
D. Polymerization intense bone demineralization. Violation of the
E. Glycosylation formation of which vitamin active form was the
cause of this complication?
A. Calciferol
11. In patients with the biliary tract obstruction B. Retinol
the blood coagulation isinhibited; the patients C. Thiamine
have frequent haemorrhages caused by the D. Naphthoquinone
subnormal assimilation of the following E. Riboflavin
vitamin:
A. К 16. The child has a delayed teething, their
B. А wrong arrangement, upon examination
C. D noticeable dryness of the oral cavity, in the
D. E corners of the mouth there are cracks with
E. C suppuration. With the lack of the vitamin this
condition may be related?
12. A few days before an operation apatient A. Vitamin D.
should be administered vitamin K or its B. Vitamin C.
synthetic analogue Vicasol. Vitamin K takes C. Vitamin E.
part in the following posttranslational D. Vitamin K.
modification of the II, VII, IX, X blood clotting E. Vitamin A.
factors:
A. Carboxylation 17. In a child of the first year of life is observed
B. Decarboxylation an increase in the size of the head and abdomen,
C. Deamination late teething, violation of the enamel structure.
D. Transamination The consequence of vitamin deficiencies are
E. Glycosylation these changes?
A. Hypovitaminosis D
13. A patient, who has been suffering for a long B. Hypovitaminosis C
time from intestine disbacteriosis, has increased C. Hypovitaminosis A
hemorrhaging caused by disruptionof D. Hypovitaminosis B1
posttranslational modification of blood- E. Hypovitaminosis B2
coagulation factors II, VII, IХ, and Х in the
liver. What vitamin deficiency is thecause of 18. A 6-year-old child suffers from delayed
this condition? growth, disrupted ossification processes,
A. К decalcification of the teeth.What can be the
B. B12 cause?
C. B9 A. Vitamin D deficiency
D. С B. Decreased glucagon production
E. Р C. Insulin deficiency
D. Hyperthyroidism
14. 10 month old child has high excitability, E. Vitamin C deficiency
sleep disturbance, amyotonia, retarded dentition,
teeth erupt withinadequate enamel calcification. 19. A 35-year-old female patient with a chronic
Thesechanges are caused by deficiency of renal disease has developed osteoporosis. The
thefollowing vitamin: cause of this complication is the deficiency of
A. Cholecalciferol the following substance:
B. Riboflavin A. 1,25-dihydroxy-D3
C. Thiamine B. 25-hydroxy-D3
D. Retinol C. D3
E. Nicotinamide D. D2
E. Cholesterol
hange E hange E
XC di XC di
F- t F- t
PD

PD
or

or
!

!
W

W
O

O
N

N
Y

Y
U

U
B

B
to

to
ww

ww
om

om
20. A child with renal insufficiency exhibits C. K
k

k
lic

lic
C

C
.c

.c
w

w
tr re tr re
.

.
ac ac
k e r- s o ft w a k e r- s o ft w a
delayed teeth eruption. This is mostlikely D. B1
caused by the abnormal formationof the E. PP
following substance:
A. 1,25 (OH)2D3 26. A 5 years old child has insufficient
B. Glycocyamine calcification of enamel, tooth decay. Which
C. Glutamate vitamin hypovitaminosis leads to the
D. α-ketoglutarate development of this process?
E. Hydroxylysine A. Calciferol
B. Tocopherol
21. When examining a child, the doctor revealed C. Biotin
signs of rickets. Which compounds D. Nicotinic acid
insufficiency in the child’s body facilitates the E. Folic acid
development of this disease?
A. 1,25 [OH] -dihydroxycholecalciferol 27. On examination of 11 months’ child, the
B. Biotin pediatrician found a curvature of the bones of
C. Tocopherol the lower extremities and a delay in the
D. Naphthoquinone mineralization of the bones of the skull. What
E. Retinol vitamin deficiency leads to this pathology?
A. Cholecalciferol
22. During regular check-up a child is detected B. Thiamine
with interrupted mineralization of the bones. C. Pantothenic acid
What vitamin deficiency can bethe cause? D. Bioflavonoids
A. Calciferol E. Riboflavin
B. Riboflavin
C. Tocopherol 28. A child with signs of rickets has been
D. Folic acid prescribed a certain liposoluble vitamin drug by
E. Cobalamin pediatrician and dentist. This drug affects the
metabolism of phosphorus and calcium in the
23. In patients after gallbladder removal body and facilitates calcium accumulation in
processes are hampered Ca2+ absorption through bone tissue and dentine. If its content in the
the intestinal wall. What vitamin prescription body is insufficient, there developdisruptions of
will stimulate this process? ossification process, dental
A. D3 structureandocclusion. Name this drug:
B. RR A. Ergocalciferol
C. C B. Retinolacetate
D. B12 C. Tocopherolacetate
E. K D. Menadione (Vicasolum)
E. Thyroidin
24. A patient who suffers from chronic renal
insufficiency fell ill with osteoporosis. 29. Hormonal form of a certain vitamin induces
Disturbed synthesis of what mineral genome level synthesis of Ca binding proteins
metabolism’s regulator is the cause of and enterocytes thus regulating the intestinal
osteoporosis? absorption of Ca2+ ions required for dental
A. Formation of 1, 25(OH)2D3 tissuedevelopment. What vitamin is it?
B. Proline hydroxylation A. D3
C. Lysine hydroxylation B. A
D. Glutamate carboxylation C. B1
E. Cortisol hydroxylation D. E
E. K
25. A patient has enamel erosion. What vitamin
should be administered for itstreatment?
A. D3
B. C
hange E hange E
XC di XC di
F- t F- t
PD

PD
or

or
!

!
W

W
O

O
N

N
Y

Y
U

U
B

B
to

to
ww

ww
om

om
30. A 4-year-old child with hereditary renal 35. In order to accelerate healing of a radiation
k

k
lic

lic
C

C
.c

.c
w

w
tr re tr re
.

.
ac ac
k e r- s o ft w a k e r- s o ft w a
lesion has signs of rickets; vitamin D ulcer a vitamin drug was administered. What
concentration in blood is normal. What is the drug is it?
most probable cause of rickets development? A. Retinol acetate
A. Impaired synthesis of calcitriol B. Retabolil
B. Increased excretion of calcium C. Prednisolone
C. Hyperfunction of parathyroid glands D. Levamisole
D. Hypofunction of parathyroid glands E. Methyluracil
E. Lack of calcium in food
36. While the examination of patient’s oral
31. Vitamin A together with specific cavity the dentist found xerostomia, numerous
cytoreceptors penetrates through thenuclear erosions. What vitamin deficit caused this
membranes, induces transcriptionprocesses that effect?
stimulate growth anddifferentiation of cells. A. Vitamin A
This biologicalfunction is realized by the B. Vitamin K
following formof vitamin A: C. Vitamin P
A. Trans-retinoic acid D. Vitamin H
B. Carotin E. Vitamin PP
C. Cis-retinal
D. Retinol 37. In order to prevent gum inflammation and to
E. Trans-retinal improve regeneration of epithelial periodontium
cells manufacturers add to the tooth pastes one
32. A patient suffers from vision impairment- of the following vitamins:
hemeralopy (night blindness). What vitamin A. Retinol
preparation should beadministered the patient in B. Calciferol
order torestore his vision? C. Thiamine
A. Retinol acetate D. Biotin
B. Vicasol E. Phyloquinone
C. Pyridoxine
D. Thiamine chloride 38. A 64 year old woman has impairment of
E. Tocopherol acetate twilight vision (hemeralopy). What vitamin
should be recommended in the first place?
33. An oculist detected increased time of A. A
darkness adaptation of a patient’s eye. What B. B2
vitamin deficiency can cause such symptom? C. E
A. А D. C
B. Е E. B6
C. С
D. К 39. The patient has worsened twilight vision.
E. D Which of the vitamin preparations should be
prescribed to the patient?
34. A patient complains of photoreception A. Retinol acetate
disorder and frequent acute viral diseases. He B. Cyanocobalamin
has been prescribed a vitamin that affects C. Pyridoxine hydrochloride
photoreception processes by producing D. Ascorbic acid
rhodopsin, the photosensitive pigment. What E. Nicotinic acid
vitamin is it?
A. Retinol acetate 40. The patient with periodontal disease was
B. Tocopherol acetate prescribed a fat-soluble vitamin preparation,
C. Pyridoxine hydrochloride which is actively involved in the redox
D. Cyanocobalamin processes in the body. Antioxidant is a growth
E. Thiamine factor, antixerphthalmic, provides normal
vision. In dental practice it is used to accelerate
hange E hange E
XC di XC di
F- t F- t
PD

PD
or

or
!

!
W

W
O

O
N

N
Y

Y
U

U
B

B
to

to
ww

ww
om

om
epithelization in case of mucosal diseases with A. Tocopherol
k

k
lic

lic
C

C
.c

.c
w

w
tr re tr re
.

.
ac ac
k e r- s o ft w a k e r- s o ft w a
periodontitis. Identify this drug: B. Thiamine
A. Retinol acetate C. Gluconate
B. Ergocalciferol D. Pyridoxine
C. Tocopherol acetate E. Choline
D. Vikasol
E. Cyanocobalamin 46. When treating sialadenitis (inflammation of
the salivary glands), preparations of vitamins
41. A patient has the folowing changes: disorder are used. Which of the following vitamins plays
of twilight vision, drying out of conjunctiva and an important role in antioxidant defense?
cornea. Such disordersmay be caused by A. Tocopherol
deficiency of vitamin: B. Pantothenic acid
A. Vitamin A C. Riboflavin
B. Vitamin B D. Thiamine
C. Vitamin C E. Pyridoxine
D. Vitamin D
E. Vitamin B12 47. A woman, who had undergone mastectomy
due to breast cancer, wasprescribed a course of
42. A patient complains of photoreception radiation therapy.What vitamin preparation has
disorder and frequent acute viral diseases. He markedantiradiation effect due to its
has been prescribed a vitamin that affects antioxidantactivity?
photoreception processes by producing A. Tocopherol acetate
rhodopsin, the photosensitive pigment. B. Ergocalciferol
What vitamin is it? C. Riboflavin
A. Retinol acetate D. Cyanocobalamin
B. Tocopherol acetate E. Folic acid
C. Pyridoxine hydrochloride
D. Cyanocobalamin 48. What vitamin deficiency leads to both
E. Thiamine disorder of reproductive function and distrophy
of skeletal muscles?
43. There are various diseases that cause sharp A. Vitamin E
increase of active oxygen, leading tocell B. Vitamin A
membranes destruction. Antioxidantsare used to C. Vitamin K
prevent it from happening. Themost potent D. Vitamin D
natural antioxidant is: E. Vitamin B1
A. Alpha-tocopherol
B. Glycerol 49. Ionizing radiation or vitamin E
C. Vitamin D deficiencyaffect the cell by increasing
D. Fatty acids lysosomemembrane permeability.What are the
E. Glucose possibleconsequences of this pathology?
A. Partial or complete cell destruction
44. A pregnant woman with several B. Intensive protein synthesis
miscarriagesin anamnesis is prescribed atherapy C. Intensive energy production
that includes vitamin preparations.What vitamin D. Restoration of cytoplasmic membrane
facilitates carrying of apregnancy? E. Formation of maturation spindle
A. Alpha-tocopherol
B. Folic acid 50. Examination of a man who hadn’t been
C. Cyanocobalamin consuming fats but had been gettingenough
D. Pyridoxal phosphate carbohydrates and proteinsfor a long time
E. Rutin revealed dermatitis, poor wound healing, vision
impairment. Whatis the probable cause of
45. Parodontosis is treated by meansof metabolic disorder?
antioxidants. Which of the followingnatural A. Lack of linoleic acid, vitamins A, D, E, K
compounds is used as an antioxidant: B. Lack of palmitic acid
hange E hange E
XC di XC di
F- t F- t
PD

PD
or

or
!

!
W

W
O

O
N

N
Y

Y
U

U
B

B
to

to
ww

ww
om

om
C. Lack of vitamins PP, H
k

k
lic

lic
C

C
.c

.c
w

w
tr re tr re
.

.
ac ac
k e r- s o ft w a k e r- s o ft w a
D. Low caloric value of diet
E. Lack of oleic acid

51. For the prevention of atherosclerosis,


coronary heart disease, cerebrovascular accident,
the consumption of high-fat polyunsaturated fatty
acids is recommended. One of the following fatty
acids is:
A. Linolic
B. Oleic
C. Lauric
D. Palmitooleic
E. Stearic
hange E hange E
XC di XC di
F- t F- t
PD

PD
or

or
!

!
W

W
O

O
N

N
Y

Y
U

U
B

B
to

to
ww

ww
om

om
k

k
Basic laws of metabolism and energy
lic

lic
C

C
.c

.c
w

w
tr re tr re
.

.
ac ac
k e r- s o ft w a k e r- s o ft w a
1. The Krebs cycle plays an important role in E. They inhibit cytochrome B
the realization of the glucoplastic effect of
amino acids. This is due to the mandatory 6. A patient with poisoning with an insecticide-
transformation of a nitrogen-free residue into: rotenone was taken to the hospital. Which part
A. Oxaloacetate. of the mitochondrial electron transfer chain is
B. Malate. blocked by this substance?
C. Succinate. A. NADH coenzyme Q-reductase.
D. Fumarat. B. ATP synthetases.
E. Citrate. C. Coenzyme Q-cytochrome C-reductase.
D. Succinate-coenzyme Q-reductase.
2. At the patient of 57 years, suffering from a E. Cytochrome C-oxidase.
diabetes, the keto-acidosis has developed. The
biochemical basis of this condition is a decrease 7. Rotenone is known to inhibit respiratory
in the degree of utilization of acetyl-CoA. The chain. What complex of mitochondrial
disadvantage of which is the connection in cells respiratory chain is inhibited by this substance?
that is due? A. NADH-coenzymeQreductase
A. Oxaloacetate B. Cytochromeoxidase
B. 2-oxoglutarate C. CoenzymeQ-cytochromecreductase
C. Glutamate D. Succinate-coenzyme Qreductase
D. Aspartate E. Adenosine triphosphate synthetase
E. Succinate
8. In the intensive care unit in a serious
3. During the oxidation of carbohydrates, lipids, condition, unconscious, the patient was
a large amount of energy is formed, the bulk of admitted. Diagnosed overdose of barbiturates,
which is formed due to the oxidation of acetyl- which caused the phenomenon of tissue
CoA. How many ATP molecules are formed hypoxia. At what level was blocking the
when a single molecule of acetyl-CoA is electron transport?
completely oxidized? A. NADH-Coenzyme-Q-reductase
A. 12. B. Cytochrome oxidase
B. 8. C. Cytochrome b - cytochrome c1
C. 38. D. Ubiquinone
D. 24. E. ATP synthase
E. 36
9. Hyperthermia, bulimia, weight loss, which is
4. Prussic acid and cyanide are among the observed in patients with thyrotoxicosis,
strongest poisons. Depending on the dose, death associated with a violation:
occurs in a few seconds or minutes. The A. Conjugation of oxidation and
inhibition of the activity of which enzyme is the phosphorylation.
cause of death? B. Reactions of fat synthesis.
A. Cytochrome oxidase. C. Decay of ATP.
B. Acetyl holinesesterase. D. Reactions of the citric acid cycle
C. Catalase. E. Reactions of beta-oxidation of fatty acids.
D. Methemoglobin reductase.
E. ATP synthetases. 10. With thyrotoxicosis, the production of
thyroid hormones T3 and T4 increases, weight
5. Cyanide poisoning causes immediate death. loss develops, tachycardia, mental excitability
What is the mechanism of cyanide effect at the and so on. The mechanism of their action?
molecular level? A. separates oxidation and oxidative
A. They inhibit cytochromoxidase phosphorylation.
B. They bind substrates of tricarboxylic acid B. Activate substrate phosphorylation.
cycle C. Block the substrate by phosphorylation.
C. They block succinate dehydrogenase D. Block the respiratory chain.
D. They inactivate oxygene E. Oxidative phosphorylation is activated.
hange E hange E
XC di XC di
F- t F- t
PD

PD
or

or
!

!
W

W
O

O
N

N
Y

Y
U

U
B

B
to

to
ww

ww
om

om
11. A woman who is receiving treatment for cytochrome C. What is its function in a normal
k

k
lic

lic
C

C
.c

.c
w

w
tr re tr re
.

.
ac ac
k e r- s o ft w a k e r- s o ft w a
hyperthyroidism, there is an increase in body cell?
temperature. What is the basis of this A. Enzyme of respiratory chain of
phenomenon? electrontransport
A. Separation of oxidative phosphorylation B. Enzyme of tricarboxylic acid cycle
B. Decreased glucose utilization by tissues C. Enzyme of beta-oxidation of fatty acids
C. Reduced fat oxidation in the liver D. Component of H+ATP system
D. Violation of amino acid deamination E. Component of pyruvate-
E. Violation of glycogen synthesis dehydrogenasesystem

12. A 38-year-old woman complains of 16. Cyanide is a poison that causes instant death
increased sweating, heartbeat, and an increase in of the organism. What enzymes found in
temperature in the evening. The main exchange mitochondria are affected by cyanide?
is increased by 60%. The doctor diagnosed A. Cytochrome oxidase (aa3)
thyrotoxicosis. What properties of thyroxin lead B. Flavin enzymes
to increased heat production? C. Cytochrome 5
A. Separation of oxidative phosphorylation D. NAD+-dependent dehydrogenase
B. Increases conjugation of oxidation and E. Cytochrome P-450
phosphorylation
C. Reduces β-oxidation of fatty acids 17. Hydrocyanic acid and cyanides are the most
D. Reduces the deamination of amino acids violent poisons. According to the dose the death
E. Contributes to the accumulation of acetyl follows after a few seconds or minutes. The
CoA death is caused by the inhibited activity of the
following enzyme:
13. Cells were treated with a substance that A. Cytochrome oxidase
blocks the phosphorylation of nucleotides in the B. Acetylcholinesterase
mitochondria. The process of cell activity will C. ATP-synthetase
be violated in the first place? D. Catalase
A. Oxidative phosphorylation. E. Methemoglobin reductase
B. Glycolysis.
C. Integration of functional protein molecules. 18. Potassium cyanide that is a poison came into
D. Aerobic oxidation of glucose a patient’s organism and caused death a few
E. Synthesis of mitochondria proteins. minutes after it. The most probable cause of its
toxic effect was abnormal activity of:
14. In the presence of 2,4-dinitrophenol, the A. Cytochrome oxidase
oxidation of substrates can continue, but the B. Catalase
synthesis of ATP molecules is impossible. What C. ATP-synthetase
is the mechanism of its action? D. NADP − H-dehydrogenase
A. Separation of oxidation and phosphorylation E. Haemoglobin synthesis
in mitochondria
B. Activation of the enzyme ATPase 19. The resuscitation unit has admitted a patient
C. Transfer of substrates beyond mitochondria in grave condition. It is known that he had
D. Stimulation of hydrolysis of formed ATP mistakenly taken sodium fluoride which blocks
E. Inhibition of the enzyme cytochrome oxidase cytochrome oxidase. What type of hypoxia
developed in the patient?
15. Researches of the latest decades established A. Tissue
that immediate "executors"ofcell apoptosis are B. Hemic
special enzymes calledcaspases. Generation of C. Cardiovascular
one of themproceeds with participation of D. Hypoxic
E. Respiratory
hange E hange E
XC di XC di
F- t F- t
PD

PD
or

or
!

!
W

W
O

O
N

N
Y

Y
U

U
B

B
to

to
ww

ww
om

om
k

k
Carbohydrate metabolism and its regulation
lic

lic
C

C
.c

.c
w

w
tr re tr re
.

.
ac ac
k e r- s o ft w a k e r- s o ft w a
1. In a patient of 60 years the activity of the 6. Analysis of a patient’s saliva revealedhigh
main digestive enzyme of saliva is reduced. In concentration of lactate. This is mostprobably
this case, the primary hydrolysis of which caused by activation of thefollowing process:
organic compounds is disturbed? A. Anaerobic glucose breakdown
A. Carbohydrates. B. Aerobic glucose breakdown
B. Fats. C. Glycogen breakdown
C. Proteins. D. Carbohydrate hydrolysis
D. Cellulose. E. Glucose-lactate cycle
E. Lactose.
7. Clinical examination enabled to make a
2. With age, the activity of the parotid glands provisional diagnosis: stomach cancer. Gastric
decreases.The activity of what enzyme of juice contained lactic acid.What type of glucose
carbohydrate metabolism will decrease? catabolism turns upin the cancerous cells?
A. Amilase. A. Anaerobic glycolysis
B. Lysozyme. B. Pentose-phosphate cycle
C. Phosphatase. C. Gluconeogenesis
D. Hexokinase. D. Aerobic glycolysis
E. Maltase. E. Glucose-alanine cycle

3. A newborn develops dyspepsia after the milk 8. What process provides erythrocytes with the
feeding. When the milk is substituted by the required amount of energy in the form of ATP
glucose solution the dyspepsia symptoms for their vital activity?
disappear. The newborn has the subnormal A. Glycolysis.
activity of the following enzyme: B. Aerobic oxidation of glucose.
A. Lactase C. β-Oxidation of fatty acids.
B. Invertase D. Pentose phosphate cycle.
C. Maltase E. Tricarbonic acid cycle.
D. Amylase
E. Isomaltase 9. Human red blood cells do notcontain
mitochondria. What is themain pathway for
4. While determining power inputs of apatient’s ATP production inthese cells?
organism it was established thatthe respiratory A. Anaerobic glycolysis
coefficient equaled 1,0. This means that in the B. Aerobic glycolysis
cells of the patientthe following substances are C. Oxidative phosphorylation
mainlyoxidized: D. Creatine kinase reaction
A. Carbohydrates E. Cyclase reaction
B. Proteins
C. Fats 10. During consumption of biscuits, sweets in
D. Proteins and carbohydrates the mixed saliva temporarily increases the level
E. Carbohydrates and fats of lactate. The activation of which biochemical
process leads to this?
5. Аfter the transition to a mixed diet in a A. Anaerobic glycolysis
newborn child has arisen dyspepsia with B. Tissue breathing
diarrhea, meteorism developmental delay.The C. Aerobic glycolysis
biochemical basis of this pathology is D. Gluconeogenesis
insufficiency of: E. Microsomal oxidation
A. Saccharaseandisomaltase.
B. Lactaseandcellobiase. 11. During long-distance running, skeletal
C. Trypsinandchymotrypsin. muscle of a trained person uses glucose to
D. Lipaseandcreatinekinase. obtain the energy of ATP for muscle
E. Cellulase. contraction. Indicate the main process of
utilization of glucose in these conditions:
A. Aerobic glycolysis
hange E hange E
XC di XC di
F- t F- t
PD

PD
or

or
!

!
W

W
O

O
N

N
Y

Y
U

U
B

B
to

to
ww

ww
om

om
B. Anaerobic glycolysis D. Enolase
k

k
lic

lic
C

C
.c

.c
w

w
tr re tr re
.

.
ac ac
k e r- s o ft w a k e r- s o ft w a
C. Glycogenolysis E. Lactate dehydrogenase
D. Gluconeogenesis
E. Glycogenesis 17. Cytoplasm of the myocytes contains a lot of
dissolved metabolites resulting fromglucose
12. After restoration of blood circulationin oxidation. Name the metabolite that turns
damaged tissue accumulation oflactate comes to directly into lactate:
a stop and speed ofglucose consumption slows A. Pyruvate
down. Thesemetabolic changes are caused by B. Oxaloacetate
activationof the following process: C. Glycerophosphate
A. Aerobic glycolysis D. Glucose-6-phosphate
B. Anaerobic glycolysis E. Fructose-6-phosphate
C. Lipolysis
D. Gluconeogenesis 18. Treatment of many diseasesinvolves use of
E. Glycogen biosynthesis cocarboxylase (thiaminepyrophosphate) for
supplying cells withenergy. What metabolic
13. When blood circulation in the damaged process is activatedin this case?
tissue is restored, then lactateaccumulation A. Oxidizing decarboxylation of pyruvate
comes to a stop andglucose consumption B. Glutamate deamination
decelerates. These metabolic changes are caused C. Amino acids decarboxylation
by activation of the following process: D. Decarboxylation of biogenic amines
A. Aerobic glycolysis E. Detoxication of harmful substances inliver
B. Anaerobic glycolysis
C. Lipolysis 19. A worker of a chemical enterprise was taken
D. Gluconeogenesis to the hospital with signs of poisoning. In the
E. Glycogen biosynthesis hair of this woman found increased
concentration of arsenate, which blocks lipoic
14. A 32-year-old female patientsuffers from acid. Violation of which process is the most
gingivitis accompaniedby gum hypoxia. What likely cause of poisoning?
metabolite ofcarbohydrate metabolism is A. Oxidation decarboxylation of pyruvate.
produced inthe periodontium tissues more B. Microsomal oxidation.
actively inthis case? C. Restoration of methemoglobin.
A. Lactate D. Restoration of organic peroxides.
B. Ribose 5-phosphate E. Removal of superoxide ions.
C. Glycogen
D. Glucose 6-phosphate 20. Oxidative decarboxylation of pyruvic acid is
E. NADPH-H catalyzed by a multienzyme complex with
several functionally linked coenzymes. Name
15. Untrained people often have musclepain this complex:
after sprints as a result of lactateaccumulation. A. Thymidine diphosphate (TDP), flavin
This might be caused byintensification of the adenine dinucleotide (FAD), coenzyme A
following biochemicalprocess: (CoASH), nicotine amide adenine dinucleotide
A. Glycolysis (NAD), lipoic acid
B. Gluconeogenesis B. Flavin adenine dinucleotide (FAD),
C. Pentose phosphate pathway tetrahydrofolic acid, pyridoxal-5-phosphate,
D. Lipogenesis thymidine diphosphate (TDP), choline
E. Glycogenesis C. Nicotine amide adenine dinucleotide (NAD),
pyridoxal-5-phosphate, thymidine diphosphate
16. Anaerobic splitting of glucose to lactic acid (TDP), methylcobalamin, biotin
is regulated by the relevant enzymes. What D. Coenzyme A (CoASH), flavin adenine
enzyme is the main regulator of this process? dinucleotide (FAD), pyridoxal-5- phosphate,
A. Phosphofructokinase tetrahydrofolic acid,carnitine
B. Glucose-6-phosphate isomerase E. Lipoic acid, tetrahydrofolic acid, pyridoxal-
C. Aldolase 5-phosphate, methylcobalamin
hange E hange E
XC di XC di
F- t F- t
PD

PD
or

or
!

!
W

W
O

O
N

N
Y

Y
U

U
B

B
to

to
ww

ww
om

om
21. It has been found out that one of pesticide consumed a small amount of alcohol, resulting
k

k
lic

lic
C

C
.c

.c
w

w
tr re tr re
.

.
ac ac
k e r- s o ft w a k e r- s o ft w a
components is sodium arsenate that blocks in severe poisoning. Explain the cause of
lipoic acid. Which enzyme activityis impaired poisoning:
by this pesticide? A. Accumulation of acetaldehyde
A. Pyruvate dehydrogenase complex B. Allergic reaction
B. Microsomal oxidation C. Neuralgia disorders
C. Methemoglobin reductase D. Cardiovascular insufficiency
D. Glutathione peroxidase E. Violation of the function of the kidneys
E. Glutathione reductase
27. The biosynthesis of the purine ring occurs
22. In erythrocytes of the patient with hemolytic on ribose-5-phosphate by gradual build-up of
anemia, the activity of pyruvate kinase was nitrogen and carbon atoms and the closure of
significantly reduced. What metabolic process is the rings. The source of ribose-5-phosphate is
violated under these conditions? the following process:
A. Glycolysis A. Pentose phosphate cycle
B. Glycogenolysis B. Glycolysis
C. Gluconeogenesis C. Glyconeogenesis
D. Pentosophosphate pathway of glucose D. Gluconeogenesis
oxidation E. Glycogenolysis
E. Synthesis of glycogen
28. Due to the long-term use of sulfanilamide
23. Decreased ratio of adenylic nucleotides drugs in a young woman appeared signs of
ATP/ADP results in intensifiedglycolysis in hemolytic anemia due to hereditary violation of
parodentium tissues underhypoxia conditions. the synthesis of the enzyme pentosophosphate
What reaction is activatedin this case? pathway glucose-6-phosphate dehydrogenase,
A. Phosphofructokinase which provides formation in the body:
B. Aldolase A. NADРН.
C. Triosophosphate isomerase B. ATP.
D. Enolase C. NAD
E. Lactate dehydrogenase D. FAD
E. FMN.
24. In some anaerobic bacteria the pyruvate
produced by glycolysis is converted to the ethyl 29. 22 year old woman has been taking
alcohol (alcoholic fermentation). What is the sulfanilamides for a long time that led to
biological significance of this process? symptoms of hemolytic anaemia caused by
A. NAD+ replenishment hereditary disturbance of synthesis of glucose 6-
B. Lactate production phosphate dehydrogenase. This enzyme of
C. ADP production pentose-phosphate cycle is responsible for
D. Providing the cells with NADPH generation of:
E. ATP production A. NADP − H2
B. NAD
25. In medical practice for preventionof C. FAD
alcoholism, a drug that is an aldehyde D. FMN
dehydrogenase inhibitor is widely used. The E. ATP
increase of which metabolite in the blood causes
disgust to alcohol? 30. Sulfanilamides are applied as antimicrobal
A. Acetaldehyde agents in clinical practice. Sulfanilamide
B. Ethanol treatment, however, can result in hemolytic
C. Malone aldehyde anemia development in patients that suffer from
D. Propionic aldehyde genetic defect of the following enzyme of
E. Methanol pentose phosphatemetabolismin erythrocytes:
A. Glucose-6-phosphate dehydrogenase
26. The patient during the course of treatment B. Hexokinase
with a drug that blocks alcohol dehydrogenase, C. Transketolase
hange E hange E
XC di XC di
F- t F- t
PD

PD
or

or
!

!
W

W
O

O
N

N
Y

Y
U

U
B

B
to

to
ww

ww
om

om
D. Transaldolase deficiency in the galactose-1-
k

k
lic

lic
C

C
.c

.c
w

w
tr re tr re
.

.
ac ac
k e r- s o ft w a k e r- s o ft w a
E. Pyruvatekinase phosphate uridylyltransferase. What is the
pathological process in the child?
31. 38 year old patient takes aspirin and A. Galactosaemia
sulfanilamides. After their intake intensified B. Fructosemia
erythrocyte haemolysis is observed which is C. Hyperglycemia
caused by deficiency of glucose 6-phosphate D. Hypoglycemia
dehydrogenase. This pathology is caused by E. Hyperlactate acidemia
failure of the following coenzyme:
A. NADP − H 36. In the 2-year-old boy, an increase in the size
B. FAD − H2 of the liver and spleen, cataract is observed. The
C. Pyridoxal phosphate concentration of sugar is elevated in the blood,
D. FMN − H2 but the test of glucose tolerance is normal. An
E. Ubiquinone hereditary violation of the metabolism of which
substance is the cause of this condition?
32. The patient has a chronic inflammatory A. Galactose
process of the tonsils. Due to the what B. Fructose
biochemical process in the inflammation C. Glucose
centers, the concentration of NADPH, which is D. Maltose
necessary for the implementation of the E. Saccharose
mechanism of phagocytosis, is maintained?
A. Pentose phosphate pathway 37. Indicate, hereditary insufficiency of which
B. Cori Cycle enzyme is the cause of vomiting and diarrhea
C. Krebs cycle after taking the fruit juices in a 9-month-old
D. Ornithine cycle child whose fructose intake has led to
E. Glycolysis hypoglycemia?
A. Fructose-1-phosphataldolase.
33. A 7-year-old child has symptoms of B. Phosphofructokinase
hemolytic anemia. In the biochemical analysis C. Hexokinase.
of erythrocytes, a reduced concentration of D. Fructose-1,6-diphosphatase.
NADPH and reduced glutathione has been E. Fructokinase.
established. The deficiency of which enzyme in
this case causes biochemical changes and 38. In the 8-month-old child, vomiting and
clinical manifestations? diarrhea are observed after taking fruit juices.
A. Glucose-6-phosphate dehydrogenase Fructose intake led to hypoglycemia. Hereditary
B. Hexokinase insufficiency of which enzyme is the cause of
C. Fructokinase these disorders?
D. Pyruvate kinase A. Fructose-1-phosphataldolase
E. Lactate dehydrogenase B. Fructokinase
C. Hexokinase
34. A child’s blood presents high content of D. Phosphofructokinase
galactose, glucose concentration is low. There E. Fructose-1,6-diphosphatase
are such presentations as cataract, mental
deficiency, adipose degeneration of liver.What 39. The excess concentration of glucose in the
disease is it? oral solution in diabetes leads to development:
A. Galactosemia A. Multiple caries
B. Diabetes mellitus B. Hyperplasia of enamel
C. Lactosemia C. Hypoplasia of enamel
D. Steroid diabetes D. Fluorosis
E. Fructosemia E. Intensifiedcalcification of enamel

35. A sick child has a delayed mental 40. In the biochemical study of the patient's
development, enlarged liver, and decreased blood, hyperglycemia, hyperketonemia, glucose
vision. The physician binds these symptoms to and ketone bodies in urine were detected. On
hange E hange E
XC di XC di
F- t F- t
PD

PD
or

or
!

!
W

W
O

O
N

N
Y

Y
U

U
B

B
to

to
ww

ww
om

om
the electrocardiogram diffuse changes in the A. Decrease of proteosynthesis
k

k
lic

lic
C

C
.c

.c
w

w
tr re tr re
.

.
ac ac
k e r- s o ft w a k e r- s o ft w a
myocardium was detected. She complains of dry B. Increase of lipolysis
mouth, thirst, frequent urination, general C. Acceleration of gluconeogenesis
weakness. The patient suffers from: D. Reduction of lipolysis
A. Diabetes mellitus. E. Increase of catabolism
B. Alimentary hyperglycemia.
C. Acute pancreatitis. 45. Diabetes mellitus causes ketosis as a result
D. Ischemic heart disease. of activated oxidation of fatty acids. What
E. Non-diabetes mellitus. disorders of acid-base equilibrium may be
caused by excessive accumulation of ketone
41. A 42 year old woman diagnosed with bodies in blood?
diabetes mellitus was admitted the A. Metabolic acidosis
endocrinological department with complaints of B. Metabolic alcalosis
thirst, excessive appetite. What pathological C. Any changes woun’t happen
components are revealed in course of laboratory D. Respiratory acidosis
examination of the patient’s urine? E. Respiratory alcalosis
A. Glucose, ketone bodies
B. Protein, aminoacids 46. A patient is ill with diabetes mellitus that is
C. Protein, creatine accompanied by hyperglycemia of over 7,2
D. Bilirubin, urobilin millimole/l on an empty stomach. The level of
E. Blood what blood plasma protein allows to estimate
the glycemia rate retrospectively (4-8 weeks
42. A patient with insulin-dependent diabetes before examination)?
mellitus has been injected with insulin. After A. Glycated hemoglobin
some time at the patient has developed B. Albumin
weakness, irritability, and increased sweating. C. Fibrinogen
What is the main mechanism of development of D. C-reactive protein
hypoglycemic coma? E. Ceruloplasmin
A. Carbohydrate starvation of the brain.
B. Decrease of gluconeogenesis. 47. A 62-year-old female patient has developed
C. Increaseof glycogenolysis. a cataract (lenticular opacity) secondary to the
D. Increase of ketogenesis. diabetes mellitus. What type of protein
E. Increase of lipogenesis. modification is observed in case of diabetic
cataract?
43. Examination of a 56-year-oldfemale patient A. Glycosylation
with a history of type 1 diabetes revealed a B. Phosphorylation
disorder ofprotein metabolism that is C. ADP-ribosylation
manifestedby aminoacidemia in the laboratory D. Methylation
blood test values, and clinically by the delayed E. Limited proteolysis
wound healing and decreased synthesis of
antibodies. Which ofthe following mechanisms 48. After recovering from epidemic parotiditis a
causes the development of aminoacidemia? patient began to put off weight, he was
A. Increased proteolysis permanently thirsty, drank a lot of water, had
B. Albuminosis frequent urination, voracious appetite. Now he
C. Decrease in the concentration ofamino acids has complaints of skin itch, weakness,
in blood furunculosis. His blood contains: glucose - 16
D. Increase in the oncotic pressure inthe blood mmole/L, ketone bodies - 100 mcmole/L;
plasma glucosuria. What disease has developed?
E. Increase in low-density lipoproteinlevel A. Insulin-dependent diabetes
B. Insulin-independent diabetes
44. It is known that in patients with diabetes C. Steroid diabetes
mellitus often found inflammatory processes, D. Diabetes insipidus
reduced regeneration, and healing of wounds E. Malnutrition diabetes
decreases. The reason for this is:
hange E hange E
XC di XC di
F- t F- t
PD

PD
or

or
!

!
W

W
O

O
N

N
Y

Y
U

U
B

B
to

to
ww

ww
om

om
49. Patient with diabetes mellitus experienced D. Itsenko-Kushing's disease
k

k
lic

lic
C

C
.c

.c
w

w
tr re tr re
.

.
ac ac
k e r- s o ft w a k e r- s o ft w a
loss of consciousness and convulsions after an E. Akromegalia
injection of insulin. What might be the result of
biochemical blood analysis for concentration of 54. In a patient 15 years of age, glucose
sugar? concentration is 4,8 mmol/l, an hour after the
A. 1,5 mmol/L sugar intake - 9.0 mmol/l, after 2 hours - 7.0
B. 8,0 mmol/L mmol/l, after 3 hours - 4.8 mmol/l . These
C. 10,0 mmol/L indices are characteristic of such a disease:
D. 3,3 mmol/L A. Hidden diabetes mellitus
E. 5,5 mmol/L B. Type I diabetes
C. Type II diabetes
50. A 46-year-old patient complains of dry D. Itsenko-Kushing's disease
mouth, thirst, urination, general weakness. In E. –
the blood: hyperglycemia, hyperketonemia. In
the urine: glucose, ketone body. On the ECG: 55. A nurse accidentally injected a nearly
diffuse changes in myocardium. What is the double dose of insulin to a patient with diabetes
most probable diagnosis? mellitus. The patient lapsed into a hypoglycemic
A. Diabetes mellitus coma. What drug should be injected in order to
B. Alimentary hyperglycemia help him out of coma?
C. Acute pancreatitis A. Glucose
D. Non-diabetes mellitus B. Lidase
E. Ischaemic heart disease C. Insulin
D. Somatotropin
51. After the epidemic parotitis, the patient loses E. Noradrenaline
weight, constantly feels thirst, drinks a lot of
water, indicates frequent urination, increased 56. For type II diabetes, the characteristic
appetite, itching, weakness, furunculosis. In the features are hyperglycemia, hypochloremia.
blood: glucose - 16 mmol/l, ketone bodies - 100 Which of the following processes is activated in
μmol/l; glucosuria What disease has developed the first place?
in the patient? A. Gluconeogenesis.
A. Insulin-dependent diabetes mellitus B. Glycolysis.
B. Non-dependent diabetes mellitus C. Glycogenolysis.
C. Steroidal diabetes D. Glucose reabsorption.
D. Non-diabetes mellitus E. Transport of glucose to the cell.
E. Malnutrition-related diabetes mellitus
57. A 57 year old patient with diabetes mellitus
52. A 38-year-old patient was delivered to the was developed ketoacedosis. Biochemical base
reanimation unit with unconscious. Reflexes are of this condition is smaller extent of acetyl-
absent. Blood sugar is 2.1 mmol/L. In CoAutilization.What cell compound deficit
anamnesis - diabetes since 18 years of age. causes this effect?
What coma does a patient have? A. Oxaloacetate
A. Hypoglycemic B. 2-oxoglutarate
B. Ketoacidotic C. Glutamate
C. Lactacidotic D. Aspartate
D. Hyperosmolar E. Succinate
E. Hyperglycemic
58. A patient with diagnose of diabetes mellitus,
53. In the patient's blood, glucose is 5.6 in the morning received an assigned dose of
mmol/L, 1 hour after the sugar intake - 13.8 insulin of prolonged action. He missed the next
mmol/l, and after 3 hours - 9.2 mmol/l. What meal, and soon felt weakness, headache,
pathology is characterized by such indicators? dizziness, greediness, trembling of the body,
A. Hidden form of diabetes mellitus convulsions, hunger, and the phenomenon of
B. Healthy person hypoglycemia. The use of glucose does not
C. Thyrotoxicosis
hange E hange E
XC di XC di
F- t F- t
PD

PD
or

or
!

!
W

W
O

O
N

N
Y

Y
U

U
B

B
to

to
ww

ww
om

om
improve the condition. What drug should be D. Low (absent) activity of glucose tr 6-
k

k
lic

lic
C

C
.c

.c
w

w
tr re re
.

.
ac ac
k e r- s o ft w a k e r- s o ft w a
entered to alleviate this condition? phosphatase
A. Adrenaline E. Deficit of a gene that is responsible for
B. Triamcinolone synthesis of glucose 1-phosphaturidine
C. Noradrenaline transferase
D. Prednisolone
E. Hydrocortisone 63. After the introduction of adrenaline in a
patient with persistent hypoglycemia, blood test
59. A patient for 5 years suffering from has not changed significantly. In such
diabetes. As a result of a violation of a diet, he circumstances, there is a possibility of
developed a coma. An ambulance doctor gave disturbances in the liver. What function of the
him glucose. The condition of the patient has liver is changed?
improved. What kind of coma was in the A. Glycogendeposing.
patient? B. Glycolytic.
A. Hypoglycemic C. Excretory.
B. Acidotic D. Ketogenic.
C. Hyperglycemic E. Cholesterol-forming.
D. Hepatic
E. Hypothyroid 64. A 2-year-old child has been diagnosed with
Girke's disease, which is manifested by severe
60. Patients with Itsenko-Cushing's syndrome hypoglycemia. The reason for this condition is
are observed with a hyperglycemia and the absence of the enzyme glucose-6-
glucosuria. Synthesis and secretion of which phosphatase. With the violation of which
hormone is increased in this patient? process is this pathology involved?
A. Cortisol A. Mobilization of glycogen.
B. Adrenaline B. Gluconeogenesis.
C. Glucagone C. Glycolysis.
D. Thyroxine D. Ketogenesis.
E. Aldosterone E. Synthesis of glycogen.

61. Pancreas is known as a mixed gland. 65. The glycogen that came from the food was
Endocrine functions include production of digested in the gastrointestinal tract. Which final
insulin by beta cells. This hormone affects the product was formed as a result of this process?
metabolism of carbohydrates. What is its effect A. Glucose
upon the activity of glycogen phosphorylase B. Lactate
(GP) and glycogen synthase (GS)? C. Lactose
A. It inhibits GP and activates GS D. Galactose
B. It activates both GP and GS E. Fructose
C. It inhibits both GP and GS
D. It activates GP and inhibits GS 66. In the human diet there are a large number
E. It does not affect the activity of GP and GS of carbohydrates. The number of which
structures will increase in cytoplasm of
62. Medical ambulance delivered a 2 year old hepatocytes?
girl to the children’s department.Objectively: A. Glycogen granules
the child is languid, apathetic. Liver is enlarged B. Drops of fat
and liver biopsy revealed a significant excess of C. The lysosomes
glycogene. Glucose concentration in the blood D. Free ribosomes
streamis below normal. What is the cause of low E. Inclusion of lipofuscine
glucose concentration?
A. Low (absent) activity of glycogene 67. Andersen's disease belongs to a group of
phosphorylase in liver hereditary diseases that develop due to the
B. Low (absent) activity of hexokinase congenital malformation of the synthesis of
C. High activity of glycogen synthetase certain enzymes of glycogenolysis. The
hange E hange E
XC di XC di
F- t F- t
PD

PD
or

or
!

!
W

W
O

O
N

N
Y

Y
U

U
B

B
to

to
ww

ww
om

om
inadequacy of which enzyme is the molecular B. Gluconeogenesis in the muscles.
k

k
lic

lic
C

C
.c

.c
w

w
tr re tr re
.

.
ac ac
k e r- s o ft w a k e r- s o ft w a
basis of this glycogenose? C. Glycogenolysis.
A. Amilo (1-4 → 1-6) transglycosidases. D. Decarboxylation.
B. Glycogen synthase. E. Synthesis of higher fatty acids.
C. Glucose-6-phosphatase.
D. Lysosomal glycosidase. 73. In a patient undergoing a course of medical
E. Phosphofructokinase. starvation, the normal level of glucose in the
blood is maintained mainly due to
68. Characteristic sign of glycogenosis is gluconeogenesis. From what amino acids at the
muscle pain during physical work. Blood same time in the human liver most actively
examination reveals usually hypoglycemia. This synthesized glucose?
pathology is caused by congenital deficiency of A. Alanine
the following enzyme: B. Lysine
A. Glycogen phosphorylase C. Valine
B. Glucose 6-phosphate dehydrogenase D. Glutamic acid
C. Alpha amylase E. Leucine
D. Gamma amylase
E. Lysosomal glycosidase 74. The gluconeogenesis is activated in the liver
after intensive physical trainings. What
69. During starvation normal rate ofglucose is substance is utilized in gluconeogenesis first of
maintained by means ofactivation of all in this case:
gluconeogenesis. Whatsubstance can be used as A. Lactate
a substrate forthis process? B. Pyruvate
A. Alanine C. Glucose
B. Ammonia D. Glutamate
C. Adenine E. Alanine
D. Urea
E. Guanine 75. During intensive physical work, muscle
tissue accumulates lactic acid, which diffuses
70. It is known that human carbohydrate into the blood and is absorbed by the liver and
reserves rapidly disappear as a result of heart. What process ensures the recovery of
prolonged fasting. Which of the metabolic glycogen stores in the muscles?
processes restores glucose in the blood? A. Cori Cycle
A. Gluconeogenesis. B. Citric acid cycle
B. Aerobic oxidation of glucose. C. Ornithine cycle
C. Glycolysis. D. Cycle of tricarboxylic acids
D. Glycogenolysis. E. Pentose phosphate pathway
E. Pentose phosphate pathway.
76. The patient was diagnosed with beri-beri.
71. Hyperglycemia develops due to chronic What is the enzyme's activity affected by the
overdose of glucocorticoids in a patient. Specify patient?
the process of carbohydrate metabolism, due to A. Pyruvate dehydrogenase.
which the concentration of glucose increases: B. Citrate synthase.
A. Gluconeogenesis. C. Malate dehydrogenase.
B. Aerobic oxidation of glucose. D. Succinate dehydrogenase.
C. Glycogenolysis. E. Fumarase.
D. Glycogenenesis.
E. Pentose phosphate cycle. 77. If insufficient thiamine - vitamin B1 occurs
disease beriberi (polyneuritis) and disturbed
72. During the fasting the muscle proteins break carbohydrate metabolism. What metabolite thus
down into free amino acids. In which process accumulates in the blood?
will most likely be used amino acids under such A. Pyruvate.
conditions? B. Lactate.
A. Gluconeogenesis in the liver. C. Succinate.
hange E hange E
XC di XC di
F- t F- t
PD

PD
or

or
!

!
W

W
O

O
N

N
Y

Y
U

U
B

B
to

to
ww

ww
om

om
D. Citrate. 82. Glycogen polysaccharide is synthesizedfrom
k

k
lic

lic
C

C
.c

.c
w

w
tr re tr re
.

.
ac ac
k e r- s o ft w a k e r- s o ft w a
E. Malat. the active form of glucose. Theimmediate donor
of glucose residues duringthe glycogenesis is:
78. Some students developed myodynia after A. UDP-glucose
continuous physical activity duringphysical B. Glucose-1-phosphate
education. The reason for suchcondition was C. ADP-glucose
accumulation of lactic acidin the skeletal D. Glucose-6-phosphate
muscles. It was generatedin the students’ bodies E. Glucose-3-phosphate
after activation ofthe following process:
A. Glycolysis 83. A child with point mutation presents with
B. Gluconeogenesis absence of glucose-6-phosphatase,
C. Lipolysis hypoglycemia, and hepatomegaly. What
D. Pentose-phosphate cycle pathology are these signs characteristic of?
E. Glyconeogenesis A. Von Gierke’s disease (Glycogen storage
disease type I)
79. It is known that the pentosephosphate B. Cori’s disease (Glycogen storage disease
pathway occuring in the adipocytesof adipose type III)
tissue acts as a cycle.What is the main function C. Addison’s disease (Primary adrenal
of this cycle inthe adipose tissue? insufficiency)
A. NADPH2 generation D. Parkinson’s disease
B. Ribose-phosphate production E. McArdle’s disease (Glycogen storage disease
C. Xenobiotic detoxification type V)
D. Energy generation
E. Glucose oxidation to end products 84. A child has a history ofhepatomegaly,
hypoglycemia, seizures,especially on an empty
80. Fructosuria is known to be connected with stomach andin stressful situations. The child is
inherited deficiency of fructose-1-phosphate diagnosedwith Gierke disease. This diseaseis
aldolase.What product of fructose metabolism caused by the genetic defect ofthe following
will accumulate in the organism resulting in enzyme:
toxicaction? A. Glucose-6-phosphatase
A. Fructose-1-phosphate B. Amyloid-1,6-glycosidase
B. Glucose-1-phosphate C. Phosphoglucomutase
C. Glucose-6-phosphate D. Glycogen phosphorylase
D. Fructose-1,6-biphosphate E. Glucokinase
E. Fructose-6-phosphate
85. In patients with glycogenosis, that is von
81. The genetic defect of pyruvatecarboxylase Gierke’s disease, the conversion of glucose-6-
deficiency is the cause of delayed physical and phosphate in to glucose is inhibited, which is
mental development and early death in children. accompanied by the improper breakdown of
This defectis characterized by lacticemia, glycogen in the liver. The cause of this
lactaciduria, disorder of a number of metabolic condition is the following enzyme deficiency:
path ways. In particular, the following process is A. Glucose-6-phosphatase
inhibited: B. Glycogen phosphorylase
A. Citric acid cycle and gluconeogenesis C. Glucose-6-phosphate dehydrogenase
B. Glycolysis and glycogenolysis D. Phosphofructokinase
C. Glycogenesis and glycogenolysis E. Phosphoglucomutase
D. Lipolysis and lipogenesis
E. Pentose phosphate pathway andglycolysis
hange E hange E
XC di XC di
F- t F- t
PD

PD
or

or
!

!
W

W
O

O
N

N
Y

Y
U

U
B

B
to

to
ww

ww
om

om
k

k
Metabolism of lipids and its regulation
lic

lic
C

C
.c

.c
w

w
tr re tr re
.

.
ac ac
k e r- s o ft w a k e r- s o ft w a
1. Examination of a patient revealed that dental E. Electrolytes
hypoplasia was caused by hypovitaminosis of
vitamins A and D. These vitamins were 6. A patient was prescribed a drug with apparent
administered perorally but they didn’t have any lipophilic properties. What is the main
medicinal effect. What is the probable cause of mechanism of its absorption?
disturbed vitamin assimilation? A. Passive diffusion
A. Bile acid deficiency B. Active transporting
B. Phospholipase A2 deficiency C. Binding with transport proteins
C. Cholesterolesterase deficiency D. Pinocytosis
D. Colipase deficiency E. Filtration
E. Pancreatic lipase deficiency
7. Due to the blockage of the common bile duct
2. Examination of a man who hadn’t been (which was radiographically confirmed), the
consuming fats but had been getting enough biliary flow to the duodenum was stopped. We
carbohydrates and proteins for long time should expect the impairment of:
revealed dermatitis, poor wound healing, vision A. Fat emulsification
impairment. Whatis the probable cause of B. Protein absorption
metabolic disorder? C. Carbohydrate hydrolysis
A. Lack of linoleic acid, vitamins A, D, E, K D. Secretion of hydrochloric acid
B. Lack of palmitic acid E. Salivation inhibition
C. Lack of vitamins PP, H
D. Low caloric value of diet 8. After eating fatty foods, the patient has
E. Lack of oleic acid nausea and heartburn, steatorrhea occurs. The
reason for such a state can be:
3. In humans, the absorption of products of the A. Lack of bile
hydrolysis of fats is impaired. The reason for B. Increased lipase secretion
this may be a deficiency in the cavity of the C. Violation of trypsin synthesis
small intestine: D. Lack of amylase
A. Bile acids E. Impaired phospholipase synthesis
B. Lipolytic enzymes
C. Bile pigments 9. A coprological study found that the feces are
D. Sodium ions discolored, there are found drops of neutral fat.
E. Fat-soluble vitamins The most likely cause of this is a violation:
A. Flow of bile into the intestine
4. A patient has normally colored stool B. pH of gastric juice
including a large amount of free fatty acids. The C. Secretions of pancreatic juice
reason for this is a disturbance of the following D. Secretion of intestinal juice
process: E. Absorption processes in the intestine
A. Fat absorption
B. Fat hydrolysis 10. The respiratory ratio of the patient is 0.7.
C. Biliary excretion This indicates that in human cells prevails:
D. Bilification A. Oxidation of fats
E. Lipase secretion B. Oxidation of carbohydrates
C. Oxidation of proteins
5. A 65-year-old patient suffers from D. Mixed oxidation of fats and carbohydrates
cholelithiasis. Recently, there were signs of E. Mixed oxidation of fats and proteins
achilic syndrome due to obturation of the biliary
tract. Which components of food will be 11. A sportsman was recommended to take a
mastered the most? preparation with carnitine in order to improve
A. Fats his achievements. What process is activated by
B.Carbohydrates carnitine to the most extent?
C. Proteins A. Transporting of fatty acids to the
D. Nucleic acids mitochondria
hange E hange E
XC di XC di
F- t F- t
PD

PD
or

or
!

!
W

W
O

O
N

N
Y

Y
U

U
B

B
to

to
ww

ww
om

om
B. Lipid synthesis B. Pancreatic amylase
k

k
lic

lic
C

C
.c

.c
w

w
tr re tr re
.

.
ac ac
k e r- s o ft w a k e r- s o ft w a
C. Synthesis of ketone bodies C. Pancreatic proteases
D. Synthesis of steroid hormones D. Bile lipase
E. Tissue respiration E. Gastric proteases

12. One of the factors that cause obesity is the 17. Obesity is a common disease. The aim of its
inhibition of fatty acids oxidation due to: treatment is to lower content of neutral fats in
A. Low level of carnitine the body. What hormone-sensitive enzyme is
B. Impaired phospholipid synthesis the most important for intracellular lipolysis?
C. Excessive consumption of fatty foods A. Triacylglycerol lipase
D. Choline deficiency B. Protein kinase
E. Lack of carbohydrates in the diet C. Adenylate cyclase
D. Diacylglycerol lipase
13. A patient with a high degree of obesity in E. Monoacylglycerol lipase
the quality of a dietary supplement is
recommended carnitine to improve the 18. Disorderthe splitting of the lipids in the
"burning" of fat. What is the direct involvement small intestine is due to a disorder of lipase
of carnitine in the process of oxidation of fats? activity. Which of the following factors
A. Transport of fatty acids from cytosol to activates the lipase?
mitochondria A. Bile acids
B. Activation of fatty acids B. Hydrochloric acid
C. Involved in one of the reactions of beta- C. Enterokinase
oxidation of fatty acids D. Pepsin
D. Transport of fatty acids from fat depots to E. Solts of Na+
tissues
E. Activation of intracellular lipolysis 19. A man has 35 years of pheochromocytoma.
In the blood there is an elevated level of
14. A 1-year-old child entered the clinic with adrenaline and norepinephrine, the
signs of damage to the muscles of the concentration of free fatty acids increased in 11-
extremities and trunk. After the examination fold. Indicate which activation of any enzyme
revealed a deficit of carnitine in the muscles. under the influence of adrenaline increases
The biochemical basis of this pathology is the lipolysis?
disruption of the process: A. TAG-lipase.
A. Transport of fatty acids in mitochondria B. Lipoprotein lipase.
B. Regulation of Ca2+level in mitochondria C. Phospholipases A2.
C. Substrate phosphorylation D. Phospholipase C.
D. Utilization of lactic acid E. Cholesterol esterase
E. Oxidizing phosphorylation
20. The patient, who was in the clinic about
15. Emotional stress causes activation of pneumonia complicated by pleurisy, was
hormone-sensitive triglyceride lipase in the included in the complex therapy of
adipocytes. What secondary mediator takes part prednisolone. The anti-inflammatory action of
in this process? this synthetic glucocorticoid is associated with
A. Cyclic adenosine monophosphate blocking the release of arachidonic acid by
B. Cyclic guanosine monophosphate inhibition:
C. Adenosine monophosphate A. A2 phospholipases
D. Diacylglycerol B. Lipoxygenase
E. Ions of Ca2+ C. Phospholipases C.
D. Peroxidase
16. A significant amount of undigested fat was E. Cyclooxygenase
found in the patient's coprogram.The violation
of which enzymes secretion most likely occurs 21. In the laboratory examination of blood of a
in this person? person who was bitten by a change, hemolysis
A. Pancreatic lipase of erythrocytes, hemoglobinuria was revealed.
hange E hange E
XC di XC di
F- t F- t
PD

PD
or

or
!

!
W

W
O

O
N

N
Y

Y
U

U
B

B
to

to
ww

ww
om

om
The action of a mixed poison is due to the C. Interferons
k

k
lic

lic
C

C
.c

.c
w

w
tr re tr re
.

.
ac ac
k e r- s o ft w a k e r- s o ft w a
presence of an enzyme in it: D. Catecholamins
A. Phospholipase A2 E. Corticosteroids
B. Phospholipase D
C. Phospholipase S. 27. The drug "Geptral", which is used in liver
D. Phospholipase A1 diseases contains S-adenosylmethionine. This
E. Sphingomyelinase active amino acid is involved in the synthesis
of:
22. Patients with pain syndrome in the joints A. Phospholipids
permanently appoint aspirin. Which of the B. Fatty acids
enzymes that he enfolds? C. Triacylglycerols
A. Phospholipase A2 D. Cholesterol
B. Cyclooxygenase E. Heme
C. Phospholipase D
D. Lipoxygenase 28. It is known that a part of carbon dioxide is
E.PhospholipaseC used in the body in the biosynthesis of fatty
acids, urea, gluconeogenesis, and the like. What
23. Thromboxanes belong to the bioregulators kind of vitamin forms a CO2-transporting form
of cellular functions of lipid nature. The source for these reactions?
for the synthesis of these compounds is: A. Biotine
A. Arachidonic acid B. Timine
B. Stearic acid C. Riboflavin
C. Palmitic acid D. Nicotinamide
D. Phosphatidic acid E. Retinol
E. Palmiotoleic acid
29. A young man of 25 years consumes an
24. Methyl groups (-CH3) are used in the body excessive amount of carbohydrates (600 g per
for synthesis such important compounds as day) that exceeds its energy needs. What
creatine, choline, adrenaline, and others. The process will be activated in the human organism
source of these groups is one of the essential in this case?
amino acids, namely: A. Lypogenesis
A. Methionine B. Glycolysis
B. Valine C. Lypolis
C. Leicine D. Gluconeogenesis
D. Isoleucine E. Oxidation of fatty acids
E. Tryptophan
30. Disorders of the processes of mielinisation
25. At deficiency of biotin, there is a disorder of neuronal fibers' leads to neurological disorders
higher fatty acids synthesis. Which of these and mental retardation. Such symptoms are
metabolites can be affected by this? characteric for hereditary and acquired
A. Malonyl-CoA metabolic disturbances:
B. Succinyl-CoA A. Sphingolipids
C. Pyruvate B. Neutral fat
D. Alanine C. Higher fatty acids
E. Serotonin D. Cholesterol
E. Phosphatidic acid
26. Deficiency in the body of linoleic and
linoleic acids leads to skin damage, hair loss, 31. The mother asked the doctor about the bad
delayed healing of wounds, thrombocytopenia, health of the child - the lack of appetite, poor
and reduced probability of infection. Disorders sleep, irritability. At the biochemical
the synthesis of which substances most likely examination in the blood, the absence of the
causes these symptoms? enzyme glucocerebrosidase was detected. For
A. Eicosanoids which pathology is this characteristic?
B. Interleukins A. Gaucher’s disease
hange E hange E
XC di XC di
F- t F- t
PD

PD
or

or
!

!
W

W
O

O
N

N
Y

Y
U

U
B

B
to

to
ww

ww
om

om
B. Tay-Sachs disease A. Choline
k

k
lic

lic
C

C
.c

.c
w

w
tr re tr re
.

.
ac ac
k e r- s o ft w a k e r- s o ft w a
C. Niemann-Pika's disease B. DOPA
D. Girke's disease C. Cholesterol
E. Pompe’s disease D. Acetoacetate
E. Linolic acid
32. A patient is diagnosed with
glucocerebrosidelipidosis (Gaucher’s disease) 37. Synthesis of phospholipids is
that manifests as splenomegaly, liver disorderedunder the liver fat infiltration.
enlargement, affected bone tissue, and Indicate which of the following substances can
neuropathies. What enzyme of complex lipid enhance the process of methylation during
catabolism is deficient, thus causing this phospholipids synthesis?
disease? A. Methionine
A. Sphingomyelinase B. Ascorbic acid
B. β-galactosidase C. Glucose
C. Glucocerebrosidase D. Glycerin
D. Hexosaminidase E. Citrate
E. Hyaluronidase
38. Examination of cell culture gotfrom a
33. At utilization of arachidonic acid by patient with lysosomal pathology revealed
cyclooxygenase pathways, biologically active accumulation of great quantity of lipids in the
substances are formed. Indicate them: lysosomes. What of the following diseases is
A. Prostaglandins this disturbance typical for?
B. Thyroxine A. Tay-Sachs disease
C. Biogenic amines B. Gout
D. Somatomedin C. Phenylketonuria
E. Insulin-like growth factors D.Wilson disease
E. Galactosemia
34. The patient appointed a lipotropic drug a
donor of methyl groups, to prevent a fatty liver 39. An experimental animal has been
distrophia. This is sensible: givenexcessive amount of carbon-
A. S-Adenosylmethionine labeledglucose for a week. What compound
B. Cholesterol canthe label be found in?
C. Bilirubin A. Palmitic acid
D. Valine B. Methionine
E. Glucose C. Vitamin A
D. Choline
35. Steatosis occurs as a result of the E. Arachidonic acid
accumulation of triacylglycerols in hepatocytes.
One of the mechanisms of development of this 40. A 6 years old child was delivered to
disease is reduction of utilization neutral fat ahospital. Examination revealed that thechild
LDL. What lipotropic substances prevent the couldn’t fix his eyes, didn’t keephis eyes on
development of steatosis? toys, eye ground had thecherry-red spot sign.
A. Methionine, BC, B12 Laboratory analyses showed that brain, liver and
B. Arginine, B2, B3 spleen had highrate of ganglioside glycometide.
C. Alanine, B1, PP What congenital disease is the child ill with?
D. Valine, B3, B2 A. Tay-Sachs disease
E. Isoleucine, B1, B2 B.Wilson’s syndrome
C. Turner’s syndrome
36. In an experimental animal, receiving non- D. Niemann-Pick disease
protein diet, fatty infiltration of the liver has E. MacArdle disease
developed due to the deficiency of the
methylating agents. The formation of which
metabolite is disturbed in the experimental
animal?
hange E hange E
XC di XC di
F- t F- t
PD

PD
or

or
!

!
W

W
O

O
N

N
Y

Y
U

U
B

B
to

to
41. A dry cleaner’s worker has been found to
ww

ww
om

om
46. The doctor gave the woman tr a
k

k
lic

lic
C

C
.c

.c
w

w
tr re re
.

.
ac ac
k e r- s o ft w a k e r- s o ft w a
have hepatic steatosis. This pathology can be recommendation to continuethe low-calorie diet.
caused by the disruption of synthesis of the She decided to get the same amount of calories,
following substance: but to replace carbohydrates on fats. Which of
A. Phosphatidylcholine the following lipoprotein fractions will bee
B. Tristearin levated as a result of this diet?
C. Urea A. Chylomicrons
D. Phosphatidic acid B. VLDL
E. Cholic acid C. LDL
D. LPID
42. The key reaction of fatty acid synthesis is E. HDL
production of malonyl-CoA. What metabolite is
the source of malonyl-CoA synthesis? 47. For the prevention of atherosclerosis,
A. Acetyl-CoA coronary heart disease, cerebrovascular
B. Succinyl-CoA accident, the consumption of high-fat
C. Acyl-CoA polyunsaturated fatty acids is recommended.
D. Malonate One of the following fatty acids is:
E. Citrate A. Linolic
B. Oleic
43. A 2-year-old child presents with acute C. Lauric
psychomotor retardation, vision and hearing D. Palmitooleic
impairment, sharp enlargement of the liver and E. Stearic
spleen. The child is diagnosed with hereditary
Niemann-Pick disease. What genetic defect is 48. Hereditary hyperlipoproteinemia of type I is
the cause of this disease? duetoin sufficiency of the lipoproteinlipase.
A. Sphingomyelinase deficiency Increasing of which transport forms of lipid in
B. Glucose 6-phosphatase deficiency plasma is characteric even on an empty
C. Amylo-1,6-glucosidase deficiency stomach?
D. Acid lipase deficiency A. Lipoproteinsoflowdensity
E. Xanthine oxidase deficiency B. Chylomicrons
C. Lipoproteinsofverylowdensity
44. A 3-year-oldgirlwithmentalretardation D. High-densitylipoproteins
hasbeendiagnosedwithsphingomyelin lipidosis E. Modifiedlipoproteins
(Niemann-Pickdisease). Inthis
conditionsynthesisofthefollowing substance 49. After 4 hours taking of fatty food is found
isdisrupted: that patient blood plasma is muddy. The most
A. Sphingomyelinase probable reason for this state is increasing the
B. Glycosyltransferase concentration in plasma:
C. Sphingosine A. Chilomicrons
D. Ceramides B. VLDL
E. Gangliosides C. LDL
D. Cholesterol
45. Modern antiatherosclerotic drugs are used E. Phospholipids
for the preventions and treatment of
atherosclerosis. Such drugs as gemfibrozil and 50. In the diseased child is established
fenfibrate is inhibiting cholesterol biosynthesis hyperlipoproteinemia, inherited. Genetic defect
by enzyme digestion: of what synthesis enzyme causes this
A. β-HMG-reductase phenomenon?
B. Hexokinase A. Lipoproteinlipase
C. Glucose-6-phosphatase B. Glycosidase
D. Acyltransferase C. Proteinase
E. Acyl-CoA-cholesterolacyltransferase D. Hemesynthetase
E. Fenylanininehydroxylase
hange E hange E
XC di XC di
F- t F- t
PD

PD
or

or
!

!
W

W
O

O
N

N
Y

Y
U

U
B

B
to

to
ww

ww
om

om
51. Rabbits were fed with cholesterol
k

k
lic

lic
C

C
.c

.c
w

w
tr re tr re
.

.
ac a
k e r- s o ft w a cke a
supplementation. After 5 months, 56. During examination of a teenagerwith r- s o ft w

atherosclerotic changes were detected in the xanthomatosis the family history


aorta. Name the main cause of atherogenesis in ofhypercholesterolemia is revealed.
this case: Whattransportable lipids are increased
A. Exogenous hypercholesterinemia inconcentration in case of such a disease?
B. Overeating A. Low-density lipoproteins
C. Hypodynamia B. Chylomicrons
D. Endogenous hypercholesterinemia C. Very low-density lipoproteins
E. Stress D. High-density lipoproteins
E. Intermediate-density lipoproteins
52. In the examination, was established the
female has insufficient activity of 57. Inthebloodofpatientswithdiabetesmellitus,
lipoproteinlipase which hydrolizes the increasesthecontentoffreefattyacids (HFAs)
cholymocrons triglycerides on the surface of the areobserved. Thereasonforthismaybe:
endothelium adipose tissue. What biochemical A.
disorders should wait? Increasingtheactivityofadipocytestriglyceridelip
A. Hyperlipoproteinemia type I ase
B. Hyperlipoproteinemia II A type B. Accumulationinthecytosolof palmitic-CoA
C. Hyperlipoproteinemia of type III C. Activationofketone bodies utilization
D. Hyperlipoproteinemia of type IV D. Activationofthesynthesisofapolipoproteins
E. Hyperlipoproteinemia II B type A-1, А-2, А-4
E. Reducedplasma phosphatidylcholine-
53. A 58-year-old patient suffers from the cholesteine acyltransferaseactivity
cerebral atherosclerosis. Examination revealed
hyperlipoidemia. What class of lipoproteins will 58. In the case of diabetes mellitus and
most probably show increase in concentration in starvation in the blood, the amount of acetone
this patient’s blood serum? bodies used in the quality of energy material
A. Low-density lipoproteins increases. Name the substance from which they
B. High-density lipoproteins are synthesized:
C. Fatty acid complexes with albumins A. Acetyl-CoA
D. Chylomicrons B. Succinyl-CoA
E. Cholesterol C. Citrate
D. Malate
54. A patient underwent a course oftreatment E. Ketoglutarate
for atherosclerosis. Laboratorytests revealed an
increase in the antiatherogeniclipoprotein 59. A 67-year-old man consumes eggs, pork fat,
fraction in theblood plasma. The treatment butter, milk and meat. Blood test results:
efficacy isconfirmed by the increase in: cholesterol - 12,3 mmol/l, total lipids - 8,2 g/l,
A. HDL increased low-density lipoprotein fraction
B. VLDL (LDL). What type of hyperlipoproteinemia is
C. IDL observed in the patient?
D. LDL A. Hyperlipoproteinemia type IIa
E. Chylomicrons B. Hyperlipoproteinemia type I
C. Hyperlipoproteinemia type IIb
55. During the examination of the patient, the D. Hyperlipoproteinemia type IV
increase in blood serum in low density E. Cholesterol, hyperlipoproteinemia
lipoproteins was detected. What disease can be
foreseen in this patient? 60. Cholesterol content in blood serum of a 12-
A. Atherosclerosis year-old boy is 25 mmol/l. Anamnesis states
B. Kidney damage hereditary familial hypercholesterolemia caused
C. Acute pancreatitis by synthesis disruption of receptor-related
D. Gastritis proteins for:
E. Lung inflammation A. Low-density lipoproteins
hange E hange E
XC di XC di
F- t F- t
PD

PD
or

or
!

!
W

W
O

O
N

N
Y

Y
U

U
B

B
to

to
ww

ww
om

om
B. High-density lipoproteins 61. Increased HDL levels decrease the risk tr of
k

k
lic

lic
C

C
.c

.c
w

w
tr re re
.

.
ac ac
k e r- s o ft w a k e r- s o ft w a
C. Chylomicrons atherosclerosis. What is the mechanism of HDL
D. Very low-density lipoproteins antiatherogenic action?
E. Middle-density lipoproteins A. They remove cholesterol from tissues
B. They supply tissues with cholesterol
C. They are involved in the breakdown of
cholesterol
D. They activate the conversion of cholesterol
to bile acids
E. They promote absorption of cholesterol in the
intestine
hange E hange E
XC di XC di
F- t F- t
PD

PD
or

or
!

!
W

W
O

O
N

N
Y

Y
U

U
B

B
to

to
ww

ww
om

om
k

k
Amino- acid metabolism and its regulation
lic

lic
C

C
.c

.c
w

w
tr re tr re
.

.
ac ac
k e r- s o ft w a k e r- s o ft w a
1. A 36-year-old female patientwho has been A. Gastrin
limiting the number offoodstuffs in her diet for B. Secretin
3 monthspresents with a decrease in body C. Cholecystokinin
weight, deterioration of physical andmental D. Somatostatin
health, face edemata. Thesechanges may be E. Neurotensin
caused by the deficiencyof the following
nutrients: 6. Young man, due to the irritation of the carotic
A. Proteins plexus by the inflammatory process (solarium),
B. Vitamins the functional activity of the glands of the
C. Fats stomach is increased, which is expressed, in
D. Carbohydrates particular, in the increase of the product of
E. Micronutrients chloride acid. Which of the following
substances causes hypochlorhydria in this case?
2. In the daily diet of an adult healthy person A. Gastrin
should be fats, proteins, carbohydrates, B. Gastrointing peptide
vitamins, mineral salts and water. Specify the C. Urologist
amount of protein per day, which provides D. Glukagon
normal functioning of the body. E. Kalikrein
A. 100-120 g
B. 50-60 g 7. The pyloric part of the stomach has been
C.10-20 g removed. Reducing the secretion of which
D. 70-80 g hormone should be seen first of all?
E. 40-50 g A. Gastrin
B. Gistamin
3. A 30 years old patient with acute C. Secretin
inflammation of the liver (pancreatitis), D. Kholetsistokinin
violations of cavity digestion of proteins were E. Stinging digestive peptide
detected. This may be due to insufficient
synthesis and secretion of the gland of such an 8. In order to determine the maximum secretion
enzyme: of hydrochloric acid in gastric juice, a solution
A. Trypsin of histaminewas given to 42-year-old patient.
B. Pepsin This led to increasing of the secretion of the
C. Lipase parietal gland of such component of juice as:
D. Dipeptidase A. Bicarbonates
E. Amilaza B. Trypsinogen
C. Lipase
4. The patient was brought to the hospital with D. Amilaza
burns of skin. To clean the wounds from dead E. Slime
tissues and mucus, the doctor prescribed an
enzyme preparation for local treatment. Name 9. In the acute experiment, a weak solution of
it: hydrochloric acid was introduced into the cavity
A. Tripsin of the duodenum of animal. Will it increase the
B. Pansinorm secrecy of the gastrointestinal hormone?
C. Asparaginase A. Sekretin
D. Pepsin B. Gastrin
E. Streptokinase C. Motilin
D. Neurotensin
5. A patient with gastric juice hypersecretion E. Gistamin
has been recommended to exclude from the
dietric broths and vegetable in fused water. A 10. Transformation of proteins in the stomach is
doctor recommended it, because these food the initial stage of protein digestion in the
products stimulate production of the following human digestive tract. What are the enzymes
hormone: involved in digestive proteins in the stomach:
hange E hange E
XC di XC di
F- t F- t
PD

PD
or

or
!

!
W

W
O

O
N

N
Y

Y
U

U
B

B
to

to
ww

ww
om

om
A. Pepsin and gastriksin noradrenaline and serotonine in the
k

k
lic

lic
C

C
.c

.c
w

w
tr re tr re
.

.
ac ac
k e r- s o ft w a k e r- s o ft w a
B. Trypsin and Cathepsin mitochondrions of cerebral neurons. What
C. Chymotropins and lysozyme enzyme participates in this process?
D. Enteropeptidase and elastase A. Monoamine oxidase
E. Carboxypeptidase and aminopeptidase B. Transaminase
C. Decarboxylase
11. A newborn child suffers from milkcurdling D. Peptidase
in stomach, this means thatsoluble milk proteins E. Lyase
(caseins) transformto insoluble proteins
(paracaseins) bymeans of calcium ions and a 16. It is known that in the metabolism of
certainenzyme. What enzyme takes part in catecholamine mediators a special role belongs
thisprocess? to the enzyme monoamine oxidase (MAO).
A. Renin How does this enzyme inactivate mediators
B. Pepsin (noradrenaline, adrenaline, dopamine)?
C. Gastrin A. The oxidative deamination
D. Secretin B. Addition of an amino group
E. Lipase C. Removal of the metal group
D. Carboxylation
12. A patient has been prescribed E. Hydrolisis
pyridoxalphosphate. What processes are
correctedwith this drug? 17. During the catabolism of histidine, a
A. Transamination and decarboxylation biogenic amine is formed, which has a powerful
ofaminoacids vasodilating effect. Name it.
B. Oxidative decarboxylation of keto acids A. Histamine.
C. Deaminization of amino acids B. DOPA.
D. Synthesis of purine and pyrimidine bases C. Dopamine.
E. Protein synthesis D. Serotonin.
E. Noradrenalin.
13. To the hospital delivered 7 years-old
childwith allergic shock, which developed after 18. When decarboxylating glutamate in the
she was stomped by bee. In the blood, the central nervous system, a mediator of inhibition
concentration of histamine is increased.The is formed. Name it.
result of which reaction formed this amine? A. GAMK.
A. Decarboxylation B. Glutathione.
B. Hydroxylation C. Histamine.
C. Degradation D. Serotonin.
D. Deamination E. Asparagine.
E. Restoration
19. Biogenic amines are used in psychiatry for
14. The allergen was administered to the patient, the treatment of a number of diseases of the
tested for hypersensitivity, under the skin,and central nervous system. Specify the drug of this
than saw reddening, edema, and even after the group, which is a mediator of inhibition:
action of histamine. As a result of which the A. γ-aminobutyric acid.
amino acid conversion of histidine forms this B. Dopamine.
biogenic amine? C. Histamine.
A. Decarboxylation D. Serotonin.
B. Methylation E. Taurin.
C. Phosphorylation
D. Isolation 20. A child 9 months old eats artificial sweeties
E. Depositing that are not balanced by the vitamin B6. A child
have a pelagic dermatitis, convulsions, anemia.
15. Pharmacological effects of The development of a cramps may be related
antidepressantsare connected with inhibition of with a violation of forming of:
an enzyme catalyzing biogenic amines A. GAMK
hange E hange E
XC di XC di
F- t F- t
PD

PD
or

or
!

!
W

W
O

O
N

N
Y

Y
U

U
B

B
to

to
ww

ww
om

om
B. Gistamin E. Cysteine, glucine, proline
k

k
lic

lic
C

C
.c

.c
w

w
tr re tr re
.

.
ac ac
k e r- s o ft w a k e r- s o ft w a
C. Serotonin
D. DOPA 26. There are several ways to dispose of
E. Dopamin ammonia in the body, but for specific organs are
specific. What is the way to neutralize this toxic
21. What neurotransmitter in the brain tissue substance is typical for brain cells?
can be synthesized from the product of A. Formation of glutamine.
reamination of alpha-ketoglutaric acid? B. Formation NH 4 +.
A. GAMK C. Formation of asparagine.
B. Tryptamine D. Formation of Creatine.
C. Dopamine E. Formation of urea.
D. Serotonin
E. Noradrenalin 27. Ammonia is a very poisonous substance,
especially for the nervous system. What
22. In a patient with a diagnosis of malignant substance is particularly active in ammonia
carcinoma, the severely increased amount of disinfection in brain tissues?
serotonin in the blood.Choose the amino acid A. Glutamic acid.
from which this compound is formed: B. Lizin.
A. Tryptophan C. Proline.
B. Allanin D. Hystidine.
C. Leicin E. Alanin.
D. Treonin
E. Methionin 28. It is known that accumulation of ammonia is
the main cause of cerebral coma in hepatic
23. Hospitalized patient with diagnosis of insufficiency. What is a free amino acid plays a
intestinal carcinoids. Analizes showed an primary role in the use of this toxic substance in
increasing secretion of serotonin. It is known the brain?
that this substance is formed from an amino acid A. Glutamic acid.
tryptophan. What biomechanical mechanism is B. Alanin.
the basis of this process? C. Histidine.
A. Decarboxylation D. Tryptophan.
B. Deamination E. Cysteine.
C. Microsomal oxidation
D. Transamination 29. After an injury to the brain, the patient has
E. Formation of paired compounds an increased ammonia formation. What amino
acid is involved in removing ammonia from this
24. Methyl groups (-CH3) are used in the body tissue?
to synthesize such important compounds as A. Glutaminov.
creatine, choline, adrenaline, and others. The B. Valin.
source of these groups is one of the essential C. Lizin.
amino acids, namely: D. Tyrosin
A. Methionin E. Tryptophan
B. Valin
C. Leicin 30. The patient entered the clinic with a
D. Isoleucine concussion of the brain. On the background of
E. Tryptophan neurological symptoms, the concentration of
ammonia in the blood increases. What substance
25. Cationic glycoproteins are the main should be used to neutralize this substance in
components of salivation of parotid glands. the brain tissue?
What amino acids cause their positive charge? A. Glutamic acid
A. Lizin, arginine, histystine B. Gistamin
B. Aspartate, glutamate, glycine C. Ascorbic acid
C. Aspartate, arginine, glutamate D. Serotonin
D. Glutamate, valine, leucine E. Nicotinic acid
hange E hange E
XC di XC di
F- t F- t
PD

PD
or

or
!

!
W

W
O

O
N

N
Y

Y
U

U
B

B
to

to
ww

ww
om

om
31. After the operation on the intestines, the 35. In a 2 year old child, there is a Intellectual
k

k
lic

lic
C

C
.c

.c
w

w
tr re tr re
.

.
ac ac
k e r- s o ft w a k e r- s o ft w a
patient appeared symptoms of poisoning with disability, intolerance of protein foods, severe
ammonia according to the type of stomach hyperammonia as the result of reduced level of
coma. What mechanism of action of ammonia urea in the plasma of blood, which is connected
on the energy supply of the central nervous with birth deficiency of such mythohondrial
system? enzyme as:
A. Braking (slowdown) CTK due to the binding A. Carbamoyl phosphate synthase
of alpha ketoglutarate B. Citrate synthase
B. Braking of glycolysis C. Succinate dehydrogenase
C. Braking of beta-oxidation of fatty acids D. Malatedehydrogenase
D. Inactivation of the enzymes of respiratory E. Monoamine oxidase
chain
E. Dissociation of oxidative phosphorylation 36. In the biosynthesis of urea in the liver, the
formation of ornithine and urea is stimulated.
32. In a newborn child there is a decrease in What amino acid is the intermediate product of
intensity of sucking, frequent vomiting, this synthesis?
hypotension. In urine and blood, the A. Аrginine
concentration of citrulline is significantly B. Leicinc.
increased. What metabolic process is affected? C. Сitrate.
A. Ornithine cycle D. Valin.
В. Glycolysis E. Tryptophan.
C. Core's cycle
D. СТК 37. In the urine of the newborn, determined by
E. Gluconeogenesis citrulline and high levels of ammonia. Indicate
which substance is most likely to be inflicted on
33. In a 3 year old child, which outlived the a baby?
severe viral infection, the doctor has indicated A. Urea.
re-vomiting, unconsciousness, convulsions. The B. Bilirubin.
hyperammonemia was detected during the C. Creatine.
analysis. What can be connected with changing D. Creatinine.
of the biochemical indicators of blood in this E. Uric acid.
child?
A. Violation of ammonia neutralization in the 38. In a patient with reduced singular function
ornithine cycle of the kidneys, an unpleasant odor is noted from
B. Activation of the processes of the mouth. Increased excretion of the salivary
decarboxylation of amino acids glands of what substance is the cause of this?
C. Violation of neutralizing of biogenic amines A. Urea
D. Strengthening of decay of proteins in the B. α-amylase.
intestines C. Mutsina.
E. Depressing the activity of transamination D. Lysozyme.
enzymes E. Fosfatasi.

34. The main part of nitrogen is taking out of 39. A patient in an unconscious state is
organism as element of urea. Reducing the delivered by an ambulance brigade to the
activity of what liver enzyme leads to inhibition hospital. Objectively: reflexes are absent,
of the synthesis of urea and increasing the seizures occur periodically, breathing is
accumulation of ammonia in blood and tissues? unequal. After a laboratory examination, the
A. Carbamoyl phosphate synthase liver was diagnosed with a coma. The
B. Aspartate aminotransferase accumulation of which metabolites in the blood
C. Urease is essential for the appearance of central nervous
D. Amilaza system disorders?
E. Pepsin A. Ammonia.
B. Bilirubin.
C. Gistamin.
hange E hange E
XC di XC di
F- t F- t
PD

PD
or

or
!

!
W

W
O

O
N

N
Y

Y
U

U
B

B
to

to
ww

ww
om

om
D. Glutamin. D. Methionine
k

k
lic

lic
C

C
.c

.c
w

w
tr re tr re
.

.
ac ac
k e r- s o ft w a k e r- s o ft w a
E. Urea. E. Glutamine

40. In a boy of 4 years after suffering from 45. A 46-year-old patient suffers from
severe viral hepatitis, vomiting, episodes of progressing Duchenne muscular dystrophy.
nephropathy, seizures are observed. In the blood Changes of the following blood enzyme level
there is hyperammonia. Violation of which of may diagnose this condition:
the biochemical process in the liver caused such A. Creatine phosphokinase
a condition of the patient? B. Lactate dehydrogenase
A. Disposal of ammonia. C. Pyruvate dehydrogenase
B. Decarboxylation of amino acids. D. Glutamate dehydrogenase
C. Disposal of Biogenic Amines. E. Adenylate kinase
D. Synthesis of alphabets.
E. Gluconeogenesis. 46. It was found that the cause of dizziness,
memory impairment, and periodic judgment in
41. Examination of a 2 year-old child with renal the patient is a violation of the decarboxylation
insufficiency revealed hyperoxaluria that caused of glutamic acid. What is the product of this
depositing of calcium oxalate stones in the reaction:
kidneys. The reason for this condition is a A. GAMK.
disturbance of metabolism of the following B. PAUL
aminoacid: C. TDP
A. Glycine D. ATP.
B. Lysine E. TGFK.
C. Methionine
D. Arginine 47. A 24 year-old patient that suffers from
E. Histidine epilepsy was administered glutamic acid. What
product of its decarboxylation caused the
42. Antioxidant enzymes inhibit lipid therapeutic effect?
peroxidation processes. Decrease of glutathione A. GABA
peroxidase activity is caused by deficiency of B. Histamine monooxygenase
the following microelement: C. Serotonin
A. Selenium D. Dopamine
B. Molybdenum E. Taurine
C. Cobalt
D. Manganese 48. A baby has epileptical convulsions that are
E. Copper caused by vitamin B6 defficiency. It is due to
the decrease of GABA in the neural tissue.
43. Blood vessels endothelium has a What enzyme activity is decrased in this case?
significantly high metabolic activity as it A. Glutamate decarboxylase
synthesizes a variety of vasoactive substances. B. Alanine aminotransferase
What powerful vasodilatator is synthesized from C. Glutamate degydrogenase
L-arginine? D. Pyridoxal kinase
A. Nitric oxide E. Glutamate synthetase
B. Histamine
C. Bradykinin 49. In a patient with a sharp increase in
D. Acetylcholine serotonin level, the diagnosis of "malignant
E. Epinephrine carcinoid" is observed in the blood. Choose the
amino acid from which the given biogenic
44. Nitric oxide is a powerful vasodilatator and amine can be formed.
blood pressure regulator. In human organism it A. Tryptophan.
is synthesized from: B. Alanine.
A. Arginine C. Leicin.
B. Proline D. Methionine
C. Lysine E. Threonine.
hange E hange E
XC di XC di
F- t F- t
PD

PD
or

or
!

!
W

W
O

O
N

N
Y

Y
U

U
B

B
to

to
50. А patient suffering from pellagra was
ww

ww
om

om
55. A patient has the initial stage of gingivitis.
k

k
lic

lic
C

C
.c

.c
w

w
tr re tr re
.

.
ac ac
k e r- s o ft w a k e r- s o ft w a
examined. It was found that his diet included Dilatation of the microcirculatory vessels
almost no meat and consisted mostly of corn. caused gum hyperaemia. What substance
His condition was caused by the deficiency of produced by mast cells caused the changes
the following substance in corn: A. Tryptophan mentioned above?
B. Tyrosine A. Histamine
C. Proline B. Epinephrine
D. Alanine C. Substance P
E. Histidine D. Endorphins
E. Actylcholine
51. A patient that is suffering from congenital
Hartnup disease has pellagra-like dermatosis 56. A cook burned his hand incautiously. What
and mental retardation due to the deficiency of substance has caused the reddening, swelling
nicotinic acid. The reason for this condition is and pain of the damaged area?
the violation of the following process: A. Histamine
A. Tryptophan absorption and reabsorption in B. Thiamine
the kidneys C. Glutamine
B. Phenylalanine transamination D. Lysine
C. Tryptophan decarboxylation E. Galactosamine
D. Methionine absorption and reabsorption in
the kidneys 57. L-DOPA is the precursor chemical of
E. Cysteine absorption and reabsorption dopamine. It is prescribed for Parkinson’s
disease treatment. What aminoacid is used for
52. A woman was bitten by a wasp. Shortly its biosynthesis?
after the bite a painful and itching blister was A. Tyrosine
formed. After some more time hives and B. Alanine
dyspnea developed. What factor was the reason C. Cysteine
for it? D. Histidine
A. Histamine E. Tryptophan
B. Hagemann factor
C. Lysosomal enzymes 58. A patient suffering from Parkinson’s disease
D. Norepinephrine has tremor of both hands. This symptom is
E. Epinephrine caused by the deficiency of the following
neurotransmitter in the striatopallidal structures:
53. Shortly after the dental treatment the patient A. Dopamine
has developed red itching blemishes on the B. GABA
facial skin and oral mucosa. Urticaria was C. Substance P
diagnosed. Vasodilatation and itching were D. Epinephrine
caused by the following bioactive substance: E. Serotonin
A. Histamine
B. Prostaglandin Е2 59. An 84 year-old patient suffers from the
C. Leucotriene В4 Parkinson’s disease. One of its pathogenetic
D. Interleukin-1 causes is the defficiency of a neurotransmitter in
E. Bradykinin certain neural structures. What neurotransmitter
is it?
54. The patient has an allergic reaction, which is A. Dopamine
accompanied by itching, swelling and reddening B. Epinephrine
of the skin. Concentration of which biogenic C. Norepinephrine
amine has increased in tissues? D. Histamine
A. Histamine. E. Acetylcholine
B. Gamma-aminobutyric acid.
C. Dopamine. 60. At the examination in the clinic, the man
D. Serotonin. was diagnosed with acute radiation sickness.
E. Triptamin. Laboratory has established a sharp decrease in
hange E hange E
XC di XC di
F- t F- t
PD

PD
or

or
!

!
W

W
O

O
N

N
Y

Y
U

U
B

B
to

to
ww

ww
om

om
serotonin in platelets. The metabolism of a D. Gout.
k

k
lic

lic
C

C
.c

.c
w

w
tr re tr re
.

.
ac ac
k e r- s o ft w a k e r- s o ft w a
substance is a possible cause of this condition? E. Fenilketonuria.
A. 5-oxytriptophane.
B. Tyrosine. 65. What is the most probable diagnosis in a
C. Histidine. child of infancy, in which there is darkening of
D. Phenylalanine. sclera, mucous membranes, auricles, separated
E. Serine. urine darkens in the air, in the blood and urine
found homogensic acid?
61. A 20-year-old woman came to the doctor A. Alkaptonuria.
with complaints of general weight loss, loss of B. Albinism.
appetite, weakness, skin discoloration C. Hemolytic anemia.
resembling bronze tan. In addtition to D. Porphyria.
hyperpigmentation, examination in the hospital E. Cystinuria.
revealed bilateral adrenal tuberculosis. What
substance leads to skin hyperpigmentation, 66. A baby has coloured sclera and mucouses
when accumulated excessively? and her urine is darkening when exposed to air.
A. Melanin Homogentisic acid was found in urine and blood
B. Bilirubin samples. What is the reason for this condition?
C. Hemozoin A Alcaptonuria
D. Lipofuscin B. Albinism
E. Adrenochrome C. Galactosemia
D. Cystinuria
62. A person consumed methionine-poor E. Histidinemia
products for a long period of time. As a result
neural and endocrine disorders have developed. 67. Dark stains on baby’s diapers indicate
They may be caused by the disturbance of the homogentisic acid synthesis. What substance’s
following substance synthesis: metabolism is disturbed?
A. Adrenaline A. Tyrosine
B. Pyruvate B. Galactose
C. Thyronine C. Methionine
D. Fatty acids D. Cholesterol
E. Glucagon E. Tryptophan

63. A 12-year-old boy in the urine revealed high 68. Excretion of homogentisic acid with the
levels of all amino acids in the aliphatic row. In urine is the reason for the condition called
this case, the highest excretion of cysteine and alcaptonuria. Disturbance of which amino acid
cysteine was noted. In addition, ultrasound of metabolism causes this condition?
the kidneys showed the presence of stones in A. Tyrosine
them. Choose a possible pathology. B. Phenylalanine
A. Cystinuria. C. Alanine
B. Alpathururia. D. Methionine
C. Cystitis E. Asparagine
D. Phenylketonuria.
E. Hartnup's disease. 69. A 5-year old child has darkened urine but no
bile enzymes were found in it. The child is
64. Parents of the 3-year-old child drew diagnosed with alcaptonuria. Which enzyme
attention to the fact that the child's urine deficiency is the reason for it?
becomes dark when restrained. Objectively: the A. Homogentisic acid oxydase
body temperature is normal, the skin is pink, B. Phenylalanine hydroxylase
clean, the stove is not elevated. What disease is C. Tyrosinase
accompanied by such manifestations. D. Oxyphenylpyruvate oxydase
A. Alkaptonuria. E. Phenylpyruvate decarboxylase
B. Hemolytic anemia.
C. Itsenko-Cushing syndrome.
hange E hange E
XC di XC di
F- t F- t
PD

PD
or

or
!

!
W

W
O

O
N

N
Y

Y
U

U
B

B
to

to
ww

ww
om

om
70. What kind of treatment is needed in a child 75. Excess of phenylpyruvate and phenylacetate
k

k
lic

lic
C

C
.c

.c
w

w
tr re tr re
.

.
ac ac
k e r- s o ft w a k e r- s o ft w a
with phenylketonuria, in whose blood there is was detected in the urine sample of a 6 day-old
an increased amount of phenylpyruvic acid? baby. Which amino acid metabolism is
A. Diet therapy. disturbed?
B. Antibacterial therapy. A. Phenylalanine
C. Vitamin therapy. B. Tryptophan
D. Hormonotherapy. C. Methionine
E. Fermentotherapy. D. Histidine
E. Arginine
71. A 9-year-old child has a mental and physical
lag. In the biochemical analysis of blood, an 76. A 1.5 year-old child is mentally and
increased amount of phenylalanine has been physically retarded, hair and skin are whitened,
detected. Blocking of an enzyme can lead to this the level of catecholamines in the blood is
condition? decreased. After adding few drops of 5%
A. Phenylalanine-4-monooxygenase. solution of ferrum trichloracetate to the urine it
B. Aspartate aminotransferases. changes its colour to olive green. This
C. Glutamintransaminase. symptoms are typical for the following
D. Glutamate-carboxylase. condition:
E. Oxidazes of homogenetic acetic acid. A. Phenylketonuria
B. Alcaptonuria
72. In a sick child, a raised level of C. Tyrosinosis
phenylpyruvate was detected in the urine (in the D. Albinism
norm practically absent). The amount of E. Xanthinuria
phenylalanine in the blood is 350 mg/l (about 15
mg/l). Which disease is characterized by the 77. A child with mental and physical
above symptoms? development retardation was delivered to the
A. Fenilketonuria. hospital. Phenylpyruvate was found in the urine.
B. Albin. Disturbance of which process is the reason for
C. Alpathonuria. this pathology?
D. Gout A. Amino acid metabolism
E. Tyrosinosis. B. Lipid metabolism
C. Carbohydrate metabolism
73. A certain type of congenital pathology is D. Water-salt balance regulation
accompanied with the inhibition of tyrosine E. Calcium and phosphate homeostasis
formation from phenylalanine. Biochemical
indicator of this condition is depositing of the 78. A patient with complaints about intolerance
following acid in the organism: to solar radiation was contacted by a doctor.
A. Phenylpyruvate There is burn skin and visual impairment.
B. Citric acid Preliminary diagnosis of albinism. Disturbance
C. Pyruvate of the exchange of amino acids observed in this
D. Lactic acid patient?
E. Glutamine A. Tyrosine.
B. Proline
74. A 6 month-old baby has mental and physical C. Lizin.
development retardation, seizures, pale D. Alanin.
eczematous skin, light hair, blue eyes. What E. Tryptophan
substance presence in blood and urine can help
to make a diagnosis in this case? 79. Albinoses has increased sensitivity to
A. Phenylpyruvate sunlight therefore they get burns instead of tan.
B. Tryptophan Which amino acid metabolism disturbance is
C. Histidine the reason for the condition?
D. Leucine A. Phenylalanine
E. Valine B. Methionine
C. Tryptophan
hange E hange E
XC di XC di
F- t F- t
PD

PD
or

or
!

!
W

W
O

O
N

N
Y

Y
U

U
B

B
to

to
ww

ww
om

om
D. Glutamine a specific smell of maple syrup. Which enzyme
k

k
lic

lic
C

C
.c

.c
w

w
tr re tr re
.

.
ac ac
k e r- s o ft w a k e r- s o ft w a
E. Histidine deficiency is typical for this condition?
A. Amino acid dehydrogenase
80. The patient is 12 years old, he has a general B. Aminotranspherase
weakness, dizziness, fatigability, lag in mental C. Glucose-6-phosphatase
development. In the laboratory examination, D. Phosphofructokinase
high concentrations of valine, isoleucine, E. Phosphofructomutase
leucine in blood and urine were detected. Urine
specific odor. What disease does this happen? 85. A hospital admitted a patient with
A. Disease of maple syrup. complaints about abdominal swelling, diarrhea,
B. Basetic disease. meteorism after consumption of food rich in
C. Hystidominemia. proteins. It is indicative of disturbed protein
D. Tyrosinosis. digestion and their intensified decaying. What
E. Addison's Illness. substance is the product of this process in the
bowels?
81. A 13 year-old boy is complaining about A. Bilirubin
general weakness, dizziness, fatigue. Mental B. Indole
retardation is detected. The examination C. Cadaverine
revealed high concentrations of valine, D. Agmatine
isoleucine, and leucine in blood and urine. The E. Putrescine
urine has a specific smell. What is the most
possible diagnosis? 86. A 60 year-old man suffers from chronic
A. Maple syrup urine disease intestinal obstruction. The processes of protein
B. Addison’s disease decomposition in the large intestine are
C. Tyrosinosis intensified. What is the evidence of these
D. Histidinemia processes?
E. Grave’s disease A. Indicanuria
B. Bilirubinuria
82. The child's urine had a characteristic smell C. Hyperuricosuria
of maple syrup. In the laboratory study, high D. Creatinuria
levels of leucine, valine, isoleucine and their E. Glucosuria
keto-derivatives were found in blood and urine.
What kind of enzyme is inadequate for this 87. A patient has been operated due to acute
disease? abdomen. His urine is brown, concentration of
A. Dehydrogenases of branched-chain amino indican is higher than 93 mmol/day. What does
acids. it indicate?
B. Aminotransferases. A. High intensity of protein decomposition
C. Glucose-6-phosphatase. processes in the intestine
D. Phosphorutokinase. B. Decreased activity of the urea cycle enzymes
E. Phosphofructomatase. C. Acceleration of aromatic amino acids
desamination
83. A baby refuses breastfeeding, it is nervous, D. Disturbance of the kidney absorption
its respiration is arrhythmic and urine has a E. Decrease of ammonia detoxification
specific smell of maple syrup. What congenital
enzymopathy is the reason for this condition? 88. A 43 year-old woman has been operated due
A. Keto acid dehydrogenase to acute abdomen. Her urine is brown and the
B. Glucose-6-phospate dehydrogenase concentration of indican in the blood has
C. Glycerol kinase increased rapidly. What does it indicate?
D. Aspartate aminotranspherase A.High intensity of protein decomposition
E. UDP-glucoronosyltranspherase B. Acceleration of amino acids desamination
C. Supression of glomerular filtration
84. A lab test revealed increased concentration D. Decreased intensity of the urea cycle
of leucine, valine, isoleucine and ketone E. Inhibition of gluconeogenesis
derivatives in the blood and urine. The urine has
hange E hange E
XC di XC di
F- t F- t
PD

PD
or

or
!

!
W

W
O

O
N

N
Y

Y
U

U
B

B
to

to
ww

ww
om

om
89. Indican is a byproduct of certain amino acid A. Tryptophan
k

k
lic

lic
C

C
.c

.c
w

w
tr re tr re
.

.
ac ac
k e r- s o ft w a k e r- s o ft w a
decomposition in the large intestine. Its B. Valine
excretion with the urine is used to indicate the C. Glycine
functional condition of the liver. What is this D. Serine
amino acid? E. Cysteine
hange E hange E
XC di XC di
F- t F- t
PD

PD
or

or
!

!
W

W
O

O
N

N
Y

Y
U

U
B

B
to

to
ww

ww
om

om
k

k
lic

lic
C

C
.c

.c
w

w
tr re tr re
.

.
ac ac
k e r- s o ft w a k e r- s o ft w a

Ministry of Public Health of Ukraine

O. O. BOGOMOLETS NATIONAL MEDICAL UNIVERSITY

Department of Bioorganic and Biological Chemistry

LIST OF TEST QUESTIONS

for preparation of Content module № 3

" Biochemistry of tissues and physiological


functions "
FOR STUDENTS OF THE 2ST YEAR OF STUDY
OF MEDICAL and STOMATOLOGICAL FACULTIES

Kyiv-2019
1
hange E hange E
XC di XC di
F- t F- t
PD

PD
or

or
!

!
W

W
O

O
N

N
Y

Y
U

U
B

B
to

to
ww

ww
om

om
k

k
lic

lic
C

C
.c

.c
w

w
tr tr
re
Content re
.

.
ac ac
k e r- s o ft w a k e r- s o ft w a

Fundamentals of Molecular Biology and Genetics ................................... 3

Biochemistry of intercellular communications:

hormones of protein-peptide nature .................................... 14

hormones of steroid and thyroid nature .............................. 22

Functional and clinical biochemistry of organs and tissues:

Biochemistry and pathobiochemistry of blood ......................................... 29

Biochemistry of immune system................................................................ 46

Biochemistry of liver .................................................................................. 51

Pathologic urine components .................................................................... 59

Biochemistry of nervous tissue.................................................................. 65

Biochemistry of muscle tissue ................................................................... 68

Biochemistry of connective tissue ............................................................. 71

Biochemistry of tooth and saliva (for stomat. faculty) ............................. 74

2
hange E hange E
XC di XC di
F- t F- t
PD

PD
or

or
!

!
W

W
O

O
N

N
Y

Y
U

U
B

B
to

to
ww

ww
om

om
k

k
lic

lic
C

C
.c

.c
Fundamentals of Molecular Biology and Genetics
w

w
tr re tr re
.

.
ac ac
k e r- s o ft w a k e r- s o ft w a

1. Gastroenterologists were assigned allopurinol, C. Adenine phosphoribosiltransferase


which inhibits the synthesis of uric acid by D. Hypoxanthine phosphoribosiltransferase
inactivating such an enzyme: E. Guanine deaminase
A. Xanthine oxidase
B. Deaminase 6. Allopurinol is a competitor of xanthine
C. Hyaluronidase
oxidase, who was prescribed for urolithiasis
D. Lactate dehydrogenase
E. Transaminase after the examination. The basis for this was the
chemical analysis of kidney stones, which
2. On the basis of laboratory analysis, the consisted mainly of:
patient was diagnosed with gout. Based on A. Urat sodium
laboratory analysis, the patient confirmed the B. Dihydrate calcium oxalate
diagnosis of gout. What was the analysis for the C. Monohydrate calcium oxalate
diagnosis? D. Phosphate calcium
A. Determination of uric acid in the blood and E. Calcium Sulphate
urine
B. Determination of urinary creatinine 7. A 48 year old patient complained about
C. Determination of residual nitrogen in the intense pain, slight swelling and reddening of
blood skin over the joints, temperature rise up to
D. The determination of urea in the blood and 38oC. Blood analysis revealed high
urine concentration of urates. This condition might be
E. Determination of urine ammonia caused by disturbed metabolism of:
A. Purines
3. A patient suffering from gout was prescribed B. Collagen
allopurinol.What pharmacological property of C. Cholesterol
allopurinol provides the rapeutic effect in this D. Pyrimidines
case? E. Carbohydrates
A. Competitive inhibition of xanthine oxidase
B. Acceleration of nitrogen-containing 8. A 46 year old patient applied to a doctor
substances excretion complaining about joint pain that becomes
C. Acceleration of pyrimidine nucleotides stronger the day before weather changes. Blood
catabolism examination revealed strengthened
D. Deceleration of pyrimidine nucleotides concentration of uric acid. The most probable
salvage cause of the disease is the intensified
E. Acceleration of nucleic acids synthesis disintegration of the following substance:
A. Adenosine monophosphate
4. Blood of a 12 year old boy presents low B. Cytidine monophosphate
concentration of uric acid and accumulation of C. Uridine triphosphate
xanthine and hypoxanthine. This child has D. Uridine monophosphate
genetic defect of the following enzyme: E. Thymidine monophosphate
A. Xanthine oxidase
B. Arginase 9. A patient has increased content of uric acid in
C. Urease his blood that is clinically presented by pain
D. Ornithine carbamoyltransferase syndrome as a result of urate deposition in the
E. Glycerylkinase joints. What process does this acid result from?
A. Lysis of purine nucleotides
5. A 42-year-old man suffering from gout has B. Lysis of pyrimidine nucleotides
increased level of urinary acid in blood. C. Heme catabolism
Allopurinol was prescribed to decrease the level D. Proteolysis
of urinary acid. Competitive inhibitor of what E. Reutilization of purine bases
enzyme is allopurinol?
A. Xanthine oxidase 10. A 65 year old man suffering from gout
B. Adenosine deaminase complains of kidney pain. Ultrasound
3
hange E hange E
XC di XC di
F- t F- t
PD

PD
or

or
!

!
W

W
O

O
N

N
Y

Y
U

U
B

B
to

to
ww

ww
om

om
k

k
lic

lic
C

C
.c examination revealed renal calculi. The most the formation of tophi, urate calculi in the

.c
w

w
tr re tr re
.

.
ac ac
k e r- s o ft w a k e r- s o ft w a

probable cause of calculi formation urinary tracts, as well as serious neuro-


is the strengthened concentration of the psychiatric disorders. The cause of this disease
following substance: is the reduced activity of the following enzyme:
A. Uric acid A.Hypoxanthine-guanine
B. Cholesterol phosphoribosyltransferase
C. Bilirubin B. Xanthine oxidase
D. Urea C. Dihydrofolate reductase
E. Cystine D. Thymidylate synthase
E. Karbamoyl phosphate synthetase
11. In the synthesis of purine nucleotides, some
amino acids, indigenous vitamins, phosphoric 16. A 1,7-year-old child with a developmental
ribozymes are involved. Name the most delay and manifestations of self-agression has
important vitamin needed for purine nucleotide the concentration of uric acid in blood at the rate
synthesis de novo. of 1,96 millimole/l. What metabolic disoder is
A. Folic acid this typical for?
B. Pantothenic acid A. Lesch-Nyhan syndrome
C. Nicotinic acid B. Podagra
D. Ryboflavin C. Acquired immunodeficiency syndrome
E. Pyridoxine D. Gierke’s disease
E. Cushing’s basophilism
12. The patient has a hypovitaminosis of folate,
which may lead to a violation of the synthesis: 17. A newborn child gains weight very slowly,
A. Purin and Tymidyl nucleotides his urine contains too much orotic acid that is
B. Purine nucleotides and cholesterol indicative of disturbed synthesis of pyrimidine
C. Tymidyl nucleotides and fatty acids nucleotides. What metabolite should be used in
D. Heme and creatine order to normalize metabolism?
E. Citrate and ketone bodies A. Uridine
B. Adenosine
13. Purine ring biosynthesis occurs in ribose-5- C. Guanosine
phosphate through gradual accumulation of D. Thymidine
nitrogen and carbon atoms and closing of the E. Histidine
rings. The source of ribose phosphate is the
process of: 18. In orоtaciduria the release of Orotic acid is
A. Pentose phosphate cycle many times higher than normal. Synthesis of
B. Glycolysis what substances will be disturbed in this
C. Glyconeogenesis pathology?
D. Gluconeogenesis A. Pyrimidine nucleotides.
E. Glycogenolysis B. Biogenic Amines.
C. Purine nucleotides.
14. A 8 year old boy suffering from the Lesh- D. Urea
Nichan disease.He has increased content of uric E. Uric acid
acid in his blood. Indicate which violation of
the process is the cause of this hereditary 19. Pterin derivatives (aminopterin and
disease? methotrexate) are the inhibitors of dihydrofolate
A. Lysis of purine nucleotides reductase, so that they inhibit the regeneration
B. Synthesis of purine nucleotides of tetrahydrofolic acid from dihydrofolate.
C. Synthesis of pyrimidine nucleotides These drugs inhibit the intermolecular tranfer of
D. Lysis of pyrimidine nucleotides monocarbon groups, thus suppressing the
E. Formation of deoxyribonucleotides synthesis of the following polymer:
A. DNA
15. Children with Lesch-Nyhan syndrome have B. Protein
a severe form of hyperuricemia accompanied by C. Homopolysaccharides

4
hange E hange E
XC di XC di
F- t F- t
PD

PD
or

or
!

!
W

W
O

O
N

N
Y

Y
U

U
B

B
to

to
ww

ww
om

om
k

k
lic

lic
C

C
.c D. Gangliosides D. Pyridoxalphosphate

.c
w

w
tr re tr re
.

.
ac ac
k e r- s o ft w a k e r- s o ft w a

E. Glycosaminoglycans E. Nicotinamidadenindynucleotide

20. Methotrexate (structural analogue of the 25. radiation and chemotherapy. The complex
folic acid which is competitive inhibitor of the of medicinal products included 5-fluoro-
dihydrofolatreductase) is prescribed for deoxyuridine - thymidylate synthetase inhibitor.
treatment of the malignant tumour. On which Synthesis of what substance is blocked by this
level does methotrexate hinder synthesis of the drug?
nucleic acids? A. DNA
A. Mononucleotide synthesis B. i-RNA
B. Replication C. p-RNA
C. Transcription D. t-RNA
D. Reparation E. -
E. Processing
26. According to the model of double DNA
21. Methotrexate (structural analogue of the helix that was suggested by Watson and Creek,
folic acid which is competitive inhibitor of the it was established that one of chains would not
dihydrofolatreductase) is prescribed for be lost during replication and the second chain
treatment of the malignant tumour. On which would be synthesized complementary to the first
level does methotrexate hinder synthesis of the one. What way of replication is it?
nucleic acids? A. Semiconservative
A. Mononucleotide synthesis B. Analogous
B. Replication C. Identical
C. Transcription D. Dispersed
D. Reparation E. Conservative
E. Processing
27. Among organic substances of a cell there is
22. An oncological patient was administered a polymer composed of dozens, hundreds, and
methotrexate. With the lapse of time the target thousands of monomers. This molecule is
cells of the tumour lost sensitivity to this capable of self-reproduction and can be an
preparation. We can observe changes in the information carrier. X-ray structure analysis
gene expression of the following enzyme: shows this molecule to consist of two
A. Dihydrofolate reductase complementary spiral threads. Name this
B. Thiminase compound:
C. Desaminase A. DNA
D. Folate oxidase B. RNA
E. Folate decarboxylase C. Cellulose
D. Carbohydrate
23. Leukoses are treated with antimetabolite E. Hormone
methotrexate. What vitamin is its antagonist?
A. Folic acid 28. Ability to divide is characteristic of
B. Cyanocobalamin procariotic and eukaryotic cells. Procariotic cell
C. Phyllochinone division is different from that of eukaryotic, but
D. Piridoxine there is one molecular process that is the basis
E. Rutin of both types of division. Name this process.
A. DNA replication
24. For the normal of the replication process B. Transcription
necessary Thimidyl nucleotides that are C. Reparation
synthesized by the enzyme Thymidylate D. Translation
synthase are , as coenzyme is used: E. Gene amplification
A. Methylenetetrahydrofolate
B. Carboxybiotin 29. Epithelium regeneration of mucous
C. Thiamindyfosphate membrane of oral cavity (cell reproduction) was

5
hange E hange E
XC di XC di
F- t F- t
PD

PD
or

or
!

!
W

W
O

O
N

N
Y

Y
U

U
B

B
to

to
ww

ww
om

om
k

k
lic

lic
C

C
.c accompanied by semiconservative DNA 34. You are studying functioning of a bacteria

.c
w

w
tr re tr re
.

.
ac ac
k e r- s o ft w a k e r- s o ft w a

replication (selfreproduction). Nucleotides of a operon. The operator gene has been released
new DNA chain are complementary to: from the repressor gene. Immediately after this
A. Maternal chain the following process will start in the cell:
B. Sense codons A. Transcription
C. DNA-polymerase enzyme B. Translation
D. Introns C. Replication
E. RNA-polymerase enzyme D. Processing
E. Repression
30. During cell division DNA replication occurs
after a signal is received from the cytoplasm, 35. It was proved that a molecule of immature
then a certain portion of the DNA helix unwinds mRNA (precursor mRNA) contained more
and splits into two individual strains. What triplets than amino acids found in the
enzyme facilitates this process? synthesized protein. The reason for that is that
A. Helicase translation is normally preceded by:
B. RNA polymerase A. Processing
C. Ligase B. Initiation
D. Restrictase C. Reparation
E. DNA polymerase D. Mutation
E. Replication
31. A group of researchers set an experiment
and obtained anucleatemutant cells. In the first 36. Nowadays about 50 minor bases have been
place they will have disturbed synthesis of the found in the t-RNA structure besides the main
following compounds: four nitrogenous bases. Choose the minor
A. Ribosomal RNA nitrogenous base:
B. Transfer RNA A. Dihydrouracil
C. Lipids B. Uracil
D. Monosaccharides C. Cysteine
E. Polysaccharides D. Adenine
E. Cytosine
32. Nucleolus organizers of human
chromosomes 13-15, 21, 22 include about 200 37. Inside a human cell the informational RNA
gene clusters that synthesize RNA. These containing both exons and introns was delivered
chromosomal regions contain the information to the granular endoplasmic reticulum to the
on the following type of RNA: ribosomes. What process does NOT take place?
A. rRNA A. Processing
B. tRNA B. Replication
C. mRNA C. Transcription
D. snRNA D. Translation
E. tRNA + rRNA E. Prolongation

33. Amino acids join to each other in ribosomes 38. Synthesis of i-RNA passes on the DNA matrix,
of granular endoplasmic reticulum. Knowing taking into account the principle of
the sequence of amino acids and applying complementarity. If the triplets in the DNA are the
genetic code, it is possible to determine the following - ATG-CGT, then the corresponding i-
RNA codons will be:
sequence of nucleoids in:
A. UAC-GCA
A. mRNA B. AUG-CGU
B. Introns C. ATG-CGT
C. Proteins D. UAG-CGU
D. Carbohydrates E. TAG-UGU
E. rRNA
39. General structure of eukaryotic genes is as
follows: exon-intron-exon. Such functional

6
hange E hange E
XC di XC di
F- t F- t
PD

PD
or

or
!

!
W

W
O

O
N

N
Y

Y
U

U
B

B
to

to
ww

ww
om

om
k

k
lic

lic
C

C
.c structure of a gene leads to certain specifics of E. Regulator

.c
w

w
tr re tr re
.

.
ac ac
k e r- s o ft w a k e r- s o ft w a

the transcription process. What sequence will


correspond with precursor mRNA (immature)? 44. A patient with pulmonary tuberculosis is
A. Exon-intron-exon prescribed the most effective antituberculous
B. Exon-exon-intron antibiotic. Name this drug:
C. Exon-exon A. Rifampicin
D. Intron-exon B. Tetracycline
E. Exon-intron C. Streptocide
D. Furasolidone
40. In the nucleus of eukaryote’s cells the first E. Bactrim (Co-trimoxazole)
pro-RNA molecule is synthesized that is
complementary to the exons and introns of the 45. Tuberculosis can be treated by means of
structural gene. But the ribosome receives such combined chemotherapy that includes
i-RNA, which is complementary to exons only. substances with different mechanisms of
This indicates that the nucleus takes place: action.What antituberculous medication inhibits
A. Processing transcription of RNA into DNA in
B. Transcription mycobacteria?
C. Reparation A. Rifampicin
D. Replication B. Isoniazid
E. Reverse transcription C. Streptomycin
D. Ethionamide
41. As a result of intoxication in the epithelial E. Para-aminosalicylic acid
cell of the mucous membrane of the oral cavity,
enzymes that provide splicing are not 46. RNA that contains AIDS virus penetrated
synthesized. What is the reason for the into a leukocyte and by means of reverse
termination of protein biosynthesis in this case? transcriptase forced a cell to synthetize a viral
A. The i-RNA is not synthesised DNA. This process is based upon:
B. ATP is not synthesised A. Reverse transcription
C. The p-RNA is not synthesised B. Operon repression
D. Amino acids are not activated C. Reverse translation
E. Transport of amino acids has been disturbed D. Operon depression
E. Convariant replication
42. During reproduction of some RNA-
containing viruses that cause tumors in animals, 47. T-lymphocytes are determined to be affected
genetic information can be transmitted in the with HIV. In this case viral enzyme reverse
opposite direction from the RNA to the DNA transcriptase (RNA-dependent DNA-
via a specific enzyme. The enzyme of reverse polymerase) catalyzes the synthesis of:
transcription is called: A. DNA based on the viral RNA matrix
A. Reverse transcriptase B. Viral RNA based on the DNA matrix
B. DNA polymerase C. Viral protein based on the viral RNA matrix
C. Ligase D. Viral DNA based on the DNA matrix
D. Primase E. Informational RNA based on the viral protein
E. Topoisomerase matrix

43. In the body of a person transcription occurs. 48. For the treatment of urogenital infections
The RNA polymerase, moving along the DNA use hinolones - inhibitors of the enzyme DNA
molecule, has reached a certain sequence of gyrase. What process is disrupted by hinolones
nucleotides. After this transcription stopped. in the first place?
This section of the DNA is called: A. Replication of DNA
A. Terminator B. Reparation of DNA
B. Promoter C. Amplification of the genes
C. Repressor D. Recombination of the genes
D. Operator E. Reverse transcription

7
hange E hange E
XC di XC di
F- t F- t
PD

PD
or

or
!

!
W

W
O

O
N

N
Y

Y
U

U
B

B
to

to
ww

ww
om

om
k

k
lic

lic
C

C
.c by different quantity of triplets ranging from

.c
w

w
tr re tr re
.

.
ac ac
k e r- s o ft w a k e r- s o ft w a

49. It was found out that some compounds, for one to six. Name this property of genetic code:
instance fungi toxins and some antibiotics can A. Degeneracy
inhibit activity of RNA-polymerase. What B. Universality
process will be disturbed in a cell in case of C. Disjointness
inhibition of this enzyme? D. Triplety
A. Transcription E. Specificity
B. Processing
C. Replication 54. Formation of ribosome subunits in a cell
D. Translation was disturbed in course of an experiment
E. Reparation (bymeans of . activated mutagenic factors). This
will have an effect on the following metabolic
50. At the stage of translation in the rough process:
endoplasmic reticulum, the ribosome moves A. Protein biosynthesis
along the mRNA. Amino acids are joined B. Carbohydrate biosynthesis
together by peptide bonds in a specific C. ATP synthesis
sequence, and thus polypeptide synthesis takes D. Photosynthesis
place. The sequence of amino acids in a E. Biological oxidation
polypeptide corresponds to the sequence of:
A. mRNA codons 55. Amino acids join to each other in ribosomes
B. tRNA nucleotides of granular endoplasmic reticulum. Knowing
C. tRNA anticodons the sequence of amino acids and applying
D. rRNA nucleotides genetic code, it is possible to determine the
E. rRNA anticodons sequence of nucleoids in:
A. mRNA
51. Students studying the features of the genetic B. Introns
code found that there are amino acids that are C. Proteins
answered by 6 codons, 5 amino acids - 4 D. Carbohydrates
different codons. Other amino acids are encoded E. rRNA
by three or two codons and only two amino
acids by one codon. What feature of the genetic 56. A tissue sample of benign tumor was studied
code did students discover? under the electron microscope. A lot of small
A. Degeneracy (Redundancy) (15-20 nm) spherical bodies, consisting of 2
B. Universality unequal subunits were
C. Collinearity detected. These are:
D. Unidirectional A. Ribosomes
E. Compose of three nucleotide B. Golgi complex
C. Smooth endoplasmic reticulum
52. The mutation of the structural gene did not D. Microtubules
lead to the substitution of amino acids in protein E. Mitochondria
molecules. This revealed the following property
of the genetic code: 57. A 36-year-old patient under went tooth
A. Degeneracy (Redundancy) extraction at a dental clinic. After two weeks the
B. Mutable stratified squamous epithelium regenerate dat
C. Collinearity the site of extraction.What organelles were
D. Insufficiency involved in there storation of the mucous
E. Universality membrane?
A. Ribosomes
53. It is known that information about amino B. Centrosomes
acid sequence in a protein molecule is stored as C. Postlysosomes
a sequence of four nucleotide types in a DNA D. Smooth EPR
molecule, and different amino acids are encoded E. Mitochondria

8
hange E hange E
XC di XC di
F- t F- t
PD

PD
or

or
!

!
W

W
O

O
N

N
Y

Y
U

U
B

B
to

to
ww

ww
om

om
k

k
lic

lic
C

C
.c 58. Genetic information is stored in DNA but

.c
w

w
tr re tr re
.

.
ac ac
k e r- s o ft w a k e r- s o ft w a

does not participate directly in protein synthesis 63. One of the protein synthesis stages is
with in DNA cells. What process ensures recognition. The first iRNA triplet starts with
transfer of genetic information into polypeptide UAU triplet. What complementary triplet is
chain? found in tRNA?
A. Translation A. AUA
B. FormationofrRNA B. AAA
C. FormationoftRNA C. GUG
D. FormationofiRNA D. UGU
E. Replication E. CUC

59. A patient has decreased concentration of 64. Cytological studies revealed a large number
magnesium ions that are required for ribosomes of different t-RNA molecules that deliver amino
connection to granular endoplasmic reticulum. acids to the ribosome. The number of different
This condition is known to disrupt the process types of t-RNA in a cell will be equal to the
of protein biosynthesis. Disruption occurs at the number of:
following stage: A. Triplets encoding amino acids
A. Translation B. Nucleotides
B. Transcription C. Amino acid
C. Replication D. Proteins synthesised in the cell
D. Amino acids activation E. Different types of RNA
E. Processing
65. Labelled amino acids alanine and
60. For the formation of a transport form of tryptophane were injected to a mouse in order to
amino acids for the synthesis of a protein, it is study localization of protein synthesis in its
necessary: cells. The labelled amino acids will be
A. Aminoacyl-tRNA synthetase accumulated near the following organellas:
B. GTP A. Ribosomes
C. m-RNA B. Smooth endoplasmic reticulum
D. Ribosome C. Cell centre
E. Revertase D. Lysosomes
E. Golgi apparatus
61. During the study of cells, a high content of
the aminoacyl-tRNA synthetase enzyme was 66. The patient was prescribed antibiotic
established in their cytoplasm. This enzyme chloramphenicol (levometsitin), which disrupts
provides the following process in a cell: protein synthesis in the body by inhibiting the
A. Activation of amino acids process:
B. Reparation A. Translation elongation
C. Elongation B. Formation of polyribosomes
D. Transcription C. Transcription
E. Replication D. Processing
E. Gene amplification
62. In the cell there is a process of translation.
When the ribosome reaches the codons of UAA, 67. Infectious diseases are treated with
UAG, or UGA, the polypeptide chain synthesis antibiotics (streptomycin, erythromycin,
is terminated. These codons are not recognized chloramphenicol). They inhibit the following
by any t-RNA in the biosynthesis of the stage of protein synthesis:
polypeptide and this is a signal: A. Translation
A. Termination B. Transcription
B. Post-translational modification C. Replication
C. Begin transcription D. Processing
D. Elongation E. Splicing
E. Initiation

9
hange E hange E
XC di XC di
F- t F- t
PD

PD
or

or
!

!
W

W
O

O
N

N
Y

Y
U

U
B

B
to

to
ww

ww
om

om
k

k
lic

lic
C

C
.c 68. A 28-year-old patient with bacterial E.coli. What portion of the lactose operon will

.c
w

w
tr re tr re
.

.
ac ac
k e r- s o ft w a k e r- s o ft w a

pneumonia was prescribed a course of treatment be unlocked from the repressor in these
with erythromycin. Its antibacterial properties conditions?
are known to be due to the ability of this A. Operator
substance to combine with the free 50S-subunit B. Promoter
of the ribosome. What substances synthesis does C. Structural gene
this antibiotic block in bacterial cells? D. Regulatory gene
A. Proteins E. Primer
B. RNA
C. DNA 73. It is known that the gene responsible for the
D. Fat development of the MN blood groups has two
E. Polysaccharides allelic states. If the gene M is considered as the
initial gene, the allelic gene N appeared due to:
69. Streptomycin and other aminoglycosides A. Mutations
prevent the joining of formylmethionyl tRNA B. Gene combinations
by bonding with the 30S ribosomal subunit. C. DNA repair
This effect leads to disruption of the following D. DNA replication
process: E. Crossing over
A. Translation initiation in procaryotes
B. Translation initiation in eucaryotes 74. As a result of treatment of viral RNA with
C. Transcription initiation in procaryotes nitrous acid, UCA triplet mutated to UGA
D. Transcription initiation in eucaryotes triplet. What kind of mutation occurred?
E. Replication initiation in procaryotes A. Transition
B. Nucleotide deletion
70. In a genetical laboratory in course of work C. Missense
with DNA molecules of white rats of Wistar’s D. Nucleotide insertion
line a nucleotide was substituted for another E. Inversion
one. At that only one amino acid was
substituded in the peptide. This 75. A mutation has occurred in a cell in the first
result is caused by the following mutation: exon of the structural gene. The number of
A. Transversion nucleotide pairs changed from 290 to 250.
B. Deletion Name this type of mutation:
C. Duplication A. Deletion
D. Displacement of reading frame B. Inversion
E. Translocation C. Duplication
D. Translocation
71. Hurtnup’s disease is caused by point E. Nullisomy
mutation of only one gene. This results in
abnormal absorption of tryptophane in the 76. When examining a 2-month-old child, the
intestine as well as its abnormal pediatrician noticed that the crying of the child
reabsorption in renal tubules. This causes resembles a cat's cry. Diagnosed microcephaly
synchronous disorders in digestive and urinary and heart disease. Using the cytogenetic
excretion systems. What genetic phenomenon is method, the child's cartiotype: 46, XX, 5p. This
observed in this case? disease is a consequence of this process:
A. Pleiotropy A. Deletion
B. Complementary interaction B. Duplication
C. Polymery C. Inversion
D. Codominance D. Translocation
E. Semidominance E. Pleiotropy

72. During the experiment, an increase in β- 77. Parents of a sick 5-year-old girl visited a
galactosidase activity was demonstrated after genetic consultation. Karyotype investigation
lactose was added to the culture medium with revealed 46 chromosomes. One chromosome of

10
hange E hange E
XC di XC di
F- t F- t
PD

PD
or

or
!

!
W

W
O

O
N

N
Y

Y
U

U
B

B
to

to
ww

ww
om

om
k

k
lic

lic
C

C
.c the 15th pair was abnormally long, having a part D. Translocation

.c
w

w
tr re tr re
.

.
ac ac
k e r- s o ft w a k e r- s o ft w a

of the chromosome belonging to the21st pair E. Replication


attached to it. What mutation occurred in this
girl? 82. An experiment proved that UV irradiated
A. Translocation skin cells of patients with xeroderma
B. Deletion pigmentosum restore the native structure of
C. Inversion DNA slower than the cells of healthy people
D. Deficiency due to the defect in repair enzyme. What
E. Duplication enzyme takes part in this process?
A. Endonuclease
78. In some regions of South Africa there is a B. RNA ligase
spread sickle-shaped cell anemia, in which C. Primase
erythrocytes have shape of a sickle as a result of D. DNA polymerase
substitution of glutamin by valine in the E. DNA gyrase
hemoglobin molecule. What is the cause of this
disease? 83. In the course of evolution there developed
A. Gene mutation molecular mechanisms for correction of
B. Disturbance of mechanisms of genetic damaged DNA molecules. This process is
information realization called:
C. Crossingover A. Reparation
D. Genomic mutations B. Transcription
E. Transduction C. Translation
D. Replication
79. Sickle-shaped e cell anemia in humans is E. Processing
accompanied by the appearance of abnormal
hemoglobin in the blood, a change in the form 84. In the human cells under the action of
of red blood cells and the development of ultraviolet radiation the DNA molecule has been
anemia. This disease is the result of: damaged. The system of repairing the damaged
A. Gene mutations area of the DNA molecule to the intact chain
B. Polythene through a specific enzyme was revised. What is
C. Chromosomal aberration the name of this phenomenon?
D. Polyplody A. Reparation
E. Mitochondrial mutation B. Duplication
C. Replication
80. A 15-year-old man is complaining of D. Initiation
general weakness, dizziness, rapid fatigability. E. Termination
During the examination, erythrocytes of a
modified form were detected, their number was 85. Cells of a person working in the Chornobyl
reduced. Preliminary diagnosis: sickle cell Exclusion Zone have under gone a mutation in
anemia. What type of mutation causes the DNA molecule. However, with time the
development of this pathological condition? damaged interval of DNA molecule has been
A. Point mutation restored to its initial structure with a specific
B. Reading frame shift mutation enzyme. In this case the following occurred:
C. Deletion A. Repair
D. Inversion B. Replication
E. Chromosomal aberration C. Transcription
D. Reverse transcription
81. Part of the DNA chain turned 180 degree as E. Translation
a result of gamma radiation. What type of
mutation took place in the DNA chain? 86. Under the influence of physical factors there
A. Inversion can develop defect sin a DNA molecule.
B. Deletion Ultraviolet irradiation, for instance, can cause
C. Doubling development of dimers. Dimers are two adjacent

11
hange E hange E
XC di XC di
F- t F- t
PD

PD
or

or
!

!
W

W
O

O
N

N
Y

Y
U

U
B

B
to

to
ww

ww
om

om
k

k
lic

lic
C

C
.c pyrimidine bases joined together. Name these 91. Fetal malformations may result from such

.c
w

w
tr re tr re
.

.
ac ac
k e r- s o ft w a k e r- s o ft w a

bases: maternal diseases as rubella, syphilis,


A. Thymine and cytosine toxoplasmosis, cytomegaly, herpes, chlamydia.
B. Adenine and thymine What is the form of variability relating to such
C. Guanine and cytosine malformations?
D. Adenine and guanine A. Modifiation
E. Guanine and thymine B. Mutational
C. Combinative
87. Patients suffering from xeroderma D. Genomic imprinting
pigmentosum have extremely photosensitive E. Epimutational
skin due to disrupted excision repair. Specify
the process that is affected in such patients: 92. As a result of iodine deficiency in
A. Repair of DNA molecule foodstuffs Transcarpathian people often have
B. Synthesis of iRNA endemic goiter. This disease is caused by the
C. Maturation of iRNA following type of variability:
D. Synthesis of protein primary structure A. Modification
E. Intron extraction and exon connection B. Mutational
Test items for licensing examination Krok 1 C. Combinative
«Stomatology». – 2016. - № 192. D. Ontogenetical
88. An employee of a chemical enterprise was E. Correlative
exposed to nitric acid and nitrite, which cause
cytosine deamination in the DNA molecule. 93. Lymphocyte is affected by HIV retrovirus
What enzyme initiates a chain of reparation (AIDS). In this case, the direction of
processes? information flow in the cell will be:
A. Uridine DNA glycosidase A. RNA → DNA → i-RNA → polypeptide
B. Citidine triphosphate synthetase B. DNA → i-RNA → polypeptide → DNA
C. Orotidyl monophosphate decarboxylase C. DNA → Polypeptide → i-RNA
D. DNA-dependent RNA polymerase D. i-RNA → polypeptide → DNA
E. Thymidylat synthase E. Polypeptide → RNA → DNA → i-RNA

89. Tetracycline taking in the first half of 94. A young family came for a genetic
pregnancy causes abnormalities of fetus organs counseling to identify the father of their child.
and systems, including tooth hypoplasia and The husband insists that the child does not
alteration of their colour. What type of resemble him at all and cannot possibly be his.
variability is the child’s disease related to? Polymerase chain reaction method
A. Modification for person identification is based on the
B. Combinative following:
C. Mutational A. Gene amplification
D. Hereditary B. Nucleotide deletion
E. Recombinan C. Genetic recombination
D. Missense mutation
90. The woman took antibiotics in the first half E. Transduction
of pregnancy. This led to the hypoplasia of the
teeth and the change in their color in the child. 95. A doctor was addressed by a 30-year old
The genotype has not changed. Set the type of man. There is a probability of the patient being
variability that underlies the disease: HIV-positive. To clarify the diagnosis the
A. Modification doctor proposed to perform polymerase chain
B. Combinative reaction. The basic process in this kind
C. Mutational of investigation is:
D. Correlative A. Gene amplification
E. Recombinative B. Transcription
C. Genetic recombination
D. Genomic mutation

12
hange E hange E
XC di XC di
F- t F- t
PD

PD
or

or
!

!
W

W
O

O
N

N
Y

Y
U

U
B

B
to

to
ww

ww
om

om
k

k
lic

lic
C

C
.c E. Chromosome mutation A. Phenocopy

.c
w

w
tr re tr re
.

.
ac ac
k e r- s o ft w a k e r- s o ft w a

B. Mutation
96. It is known that the gene responsible for C. Recombination
development of blood groups according to AB0 D. Heterosis
system has three allele variants. Existence of the E. Replication
IV blood group can be explained by the
following variability form: 98. The doctor discovered a kid's disease due to
A. Combinative a lack of vitamin D, but in his manifestation
B. Mutational similar to hereditary vitamin-resistant rickets
C. Phenotypic (curvature of tubular bones, deformity of joints
D. Genocopy of lower extremities, dental abscesses). How
E. Phenocopy called defects of development, which resemble
hereditary, but not inherited?
97. A mother had taken synthetic hormones A. Phenocopy
during pregnancy. Her daughter was born with B. Genocopy
hirsutismformally resembling of adrenal C. Monosomies
syndrome. Such D. Trisomy
manifestation of variability is called: E. Gene diseases

13
hange E hange E
XC di XC di
F- t F- t
PD

PD
or

or
!

!
W

W
O

O
N

N
Y

Y
U

U
B

B
to

to
ww

ww
om

om
k

k
lic

lic
C

C
.c

.c
Biochemistry of intercellular communications: hormones of protein-
w

w
tr re tr re
.

.
ac ac
k e r- s o ft w a k e r- s o ft w a

peptide nature
1. Degeneration of glycogen in liver is B. Triamcinolone
stimulated by glucagon. What secondary C. Norepinephrine
messenger (mediator) is thus formed in the cell? D. Prenisolone
A. c-AMP E. Hydrocortisone
B. c-GMP
C. CO 6. Condition of a patient with diabetes mellitus
D. NO sharply deteriorated after a regular injection of
E. Triacylglycerol insulin. The patient became anxious and broke
out in cold sweat; tremor of the extremities,
2. The ions of metals take part in the regulation general weakness, and dizziness appeared. What
of physiological functions. One of them was medicine can remove these symptoms?
named the “king of messengers”. Such a A. Adrenaline
bioelement mediator is: B. Tolbutamide
A. Ca ++ C. Caffeine
B. Na + D. Noradrenaline
C. K + E. Glibutid(Buformin)
D. Fe +++
E. Zn ++ 7. Dentists widely apply local anaesthesia
adding adrenalin to an anaesthetic solution.
3. Tissue inosytol triphosphates are generated as What is the purpose of this method?
a result of the phosphatidyl inositol diphosphate A. Local vasoconstriction
hydrolysis and act as secondary agents B. Local vasodilatation
(mediators) in the mechanism of hormone C. Lowering of arterial pressure
action. Their effect in cells is directed at: D. Local reduction of vascular resistance
A. Calcium ion liberation from cellular depot E. Microcirculation improvement
B. Adenylate cyclase activation
C. Protein kinase A activation 8. A 50-year-old man declined anaesthesia
D. Phosphodiesterase inhibition during dental manipulations. Due to severe pain
E. Protein kinase A inhibition he developed anuria caused by acute increase in
production of:
4. The formation of a secondary mediatoris A. Adrenaline
obligatory in membrane-intracellular B. Renin
mechanism of hormone action. Point out the C. Thymosin
substance that is unable to be a secondary D. Thyroxin
mediator: E. Glucagon
A. Glycerol
B. Diacylglycerol 9. A sick woman after parenteral administration
C. Inositol-3,4,5-triphosphate of the hormone had an increase in blood
D. CAMP pressure, as well as increased levels of glucose
E. Ca2+ and lipids in the blood. What hormone was
introduced?
5. A patient with a diagnosis of diabetes, in the A. Adrenaline
morning on an empty stomach received a B. Glucagon
prescribed dose of insulin with prolonged C. Insulin
action. He missed another meal and soon felt D. Progesterone
weakness, pain, dizziness, sweating, trembling E. Folliculin
of the body, convulsions, feeling of hunger,
hypoglycemia. The use of glucose did not 10. A patient suffering from pheochromocytoma
alleviate the condition. What hormonal drug is complains of thirst, dry mouth, hunger. Blood
necessary to enter to stop this condition? test for sugar revealed hyperglycemia. What
A. Adrenaline type of hyperglycemia is it?
14
hange E hange E
XC di XC di
F- t F- t
PD

PD
or

or
!

!
W

W
O

O
N

N
Y

Y
U

U
B

B
to

to
ww

ww
om

om
k

k
lic

lic
C

C
.c A. Adrenal

.c
w

w
tr re tr re
.

.
ac ac
k e r- s o ft w a k e r- s o ft w a

B. Hypercorticoid 15. Before the cells can utilize the glucoze, it is


C. Alimentary first transported from the extracellular space
D. Somatotropic through the plasmatic membrane inside theml.
E. Hypoinsulinemic This process is stimulated by the following
hormone:
11. Patient with diabetes mellitus experienced A. Insulin
loss of consciousness and convulsions after an B. Glucagon
injection of insulin. What might be the result of C. Thyroxin
biochemical blood analysis for concentration of D. Aldosterone
sugar? E. Adrenalin
A. 1,5 mmol/L
B. 8,0 mmol/L 16. After insulin injection, glucose levels fall
C. 10,0 mmol/L within a few seconds. This is due to the
D. 3,3 mmol/L activation of this process: A. Transport of
E. 5,5 mmol/L glucose into cells
B. Glycolysis
12. A patient with diabetes mellitus had an C. Synthesis of glycogen
insuline injection. It caused loss of D. Synthesis of lipids
consciousness and convulsions. What was the E. Pentose phosphate cycle
result of biochemic blood analysis on glucose
content? 17. Prior to glucose utilization in cells it is
A. 2,5 mmоle/l transported inside cells from extracellular space
B. 3,3 mmоle/l through plasmatic membrane. This process is
C. 8,0 mmоle/l stimulated by the following hormone:
D. 10 mmоle/l A. Insulin
E. 5,5 mmоle/l B. Glucagon
C. Thyroxin
13. A 16-year-old young man was taken to D. Aldosterone
hospital, the patient had insulin-dependent E. Adrenalin
diabetes mellitus. The patient's blood glucose
level was 1.8 mmol/l. The patient was given 18. A patient is in the state of hypoglycemic
insulin. Two hours later, the glucose level coma. What hormone can cause this condition if
decreased to 8.2 mmol/l, since insulin: overdosed?
A. Stimulates the conversion of glucose to A. Insulin
glycogen and TAG in the liver B. Progesterone
B. Stimulates glucose transport through plasma C. Cortisol
membranes in the brain and liver D. Somatotropin
C. It inhibits the synthesis of ketone bodies from E. Corticotropin
glucose
D. Stimulates the breakdown of glycogen in the 19. A patient during fasting developed
liver E. Stimulates the breakdown of glycogen ketoacidosis as a result of increased fatty acids
in the muscles decomposition. This decomposition can be
inhibited with:
14. A patient with insulin-dependent diabetes A. Insulin
had an insulin injection. Some time later he felt B. Glucagon
weakness, irritability, excessive sweating. What C. Adrenaline
is the main reason of these disorders? D. Thyroxin
A. Carbohydrate starvation of brain E. Cortisol
B. Intensified glycogenolysis
C. Intensified ketogenesis 20. Examination of a patient revealed
D. Intensified lypogenesis overgrowth of facial bones and soft tissues,
E. Reduced glyconeogenesis tongue enlargement, wide interdental spaces in

15
hange E hange E
XC di XC di
F- t F- t
PD

PD
or

or
!

!
W

W
O

O
N

N
Y

Y
U

U
B

B
to

to
ww

ww
om

om
k

k
lic

lic
C

C
.c the enlarged dental arch. What changes of the 25. A student who passes an exam has a plasma

.c
w

w
tr re tr re
.

.
ac ac
k e r- s o ft w a k e r- s o ft w a

hormonal secretion are the most likely? glucose content of 8 mmol/l. Increased secretion
A. Hypersecretion of the somatotropic hormone which of hormones contributes to the
B. Hyposecretion of the somatotropic hormone development of hyperglycemia?
C. Hypersecretion of insulin A. Glucagon
D. Hyposecretion of thyroxin B. Insulin
E. Hyposecretion of insulin C. Thyroxine
D. Triiodothyronine
21. A 49-year-old patient was found to have a E. Aldosterone
disproportionate enlargement of hands, feet,
nose, ears, superciliary arches and cheek bones. 26. A patient with tress and painful sensation
Blood test revealed hyperglycemia, impaired before a visit to the dentist is accompanied by
glucose tolerance. What is the most likely cause anuria (lack of urination). This phenomenon is
of this pathology development? due to an increase in:
A. Hypersecretion of growth hormone A. Secretion of vasopressin and adrenaline
B. Posterior pituitary hormone hypersecretion B. Activity of the parasympathetic nervous
C. Insulin hyposecretion system
D. Vasopressin hyposecretion C. Activity antinociceptive system
E. Glucocorticoid hypersecretion D. Secretion of vasopressin and a decrease in
adrenaline
22. A woman after labor lost 20 kg of body E. Secretion of adrenaline and a decrease in
weight, her hair and teeth fall out, she has vasopressin
muscle atrophy (hypophysial cachexia).
Synthesis of what hypophysis hormone is 27. As a result of a home injury, a patient
disturbed? suffered a significant blood loss, which led to a
A. Somatotropic fall in blood pressure. Rapid blood pressure
B. Corticotrophic recovery after the blood loss is provided by the
C. Thyreotropic following hormones:
D. Gonadotropic A. Adrenaline, vasopressin
E. Prolactin B. Cortisol
C. Sex hormones
23. Adult height 100 cm with a proportional D. Oxytocin
body structure and normal mental development. E. Aldosterone
It might be caused by deficiency secretion of the
following hormone: 28. A severe injury in a 36-year-old patient
A. Somatotropic resulted in a significant blood loss which was
B. Gonadotropic accompanied by a blood pressure drop. What
C. Adrenocorticotropic hormones provide rapid recovery of blood
D. Thyroid-stimulating pressure after the blood loss?
E. Prolactin A. Adrenalin, vasopressin
B. Cortisol
24. Examination of a patient revealed C. Sex hormones
enlargement of some body parts (jaw, nose, D. Oxytocin
ears, feet, hands), but body proportions were E. Aldosterone
conserved. It might be caused by intensified
secretion of the following hormone: 29. A man has the increased volume of
A. Somatotropin circulating blood and reduced osmotic pressure
B. Somatostatin of plasma considerable. He has decrease in
C. Tetraiodothyronine diuresis. The primary cause of such diuresis
D. Triiodothyronine disorder is the hypersecretion of the following
E. Cortisol hormone:
A. Vasopressin
B. Aldosterone

16
hange E hange E
XC di XC di
F- t F- t
PD

PD
or

or
!

!
W

W
O

O
N

N
Y

Y
U

U
B

B
to

to
ww

ww
om

om
k

k
lic

lic
C

C
.c C. Adrenalin C. Vasopressin

.c
w

w
tr re tr re
.

.
ac ac
k e r- s o ft w a k e r- s o ft w a

D. Renin D. Thyrocalcitonin
E. Natriuretic E. Glucagon

30. A 20 year old patient complains of excessive 35. A 25-year-old woman one month after
thirst and urinary excretion upto 10 L a day. The giving birth consulted a doctor about a decrease
level of glucose in blood is normal, there is no in milk production. What hormone deficiency
glucose in urine. What hormone deficit can led to this condition?
cause such changes? A. Prolactin
A. Vasopressin B. Somatostatin
B. Oxytocin C. Adrenocorticotropic hormone
C. Insulin D. Insulin
D. Triiodothyronine E. Glucagon
E. Cortisol
36. Products of some proteins hydrolysis and
31. A patient with pituitary tumor complains of modification are the biologically active
increased daily diuresis (polyuria). Glucose substances called hormones. Lipotropin,
concentration in blood plasma equals 4,8 corticotropin, melanotropin and endorphins are
mmol/l. What hormone can be the cause of this synthesized in the hypophysis of the following
if its secretion is disturbed? protein:
A. Vasopressin A. Proopiomelanocortin (POMC)
B. Aldosterone B. Neuroalbumin
C. Natriuretic hormone C. Neurostromin
D. Insulin D. Neuroglobulin
E. Angiotensin I E. Thyreoglobulin

32. Limiting water intake has led to dehydration 37. After a case of sepsis a 27-year-old woman
of the organism. What mechanism is activated developed ”bronzed” skin discoloration
under these conditions to preserve water in the characteristic of Addison’s disease.
body? Hyperpigmentation mechanismin this case is
A. Increased vasopressin secretion based on increased secretion of:
B. Increased somatostatin secretion A. Melanocyte-stimulating hormone
C. Reduced vasopressin secretion B. Somatotropin
D. Increased aldosterone secretion C. Gonadotropin
E. Reduced aldosterone secretion D. β-lipotropin
E. Thyroid-stimulating hormone
33. At ria of a test animal were super distended
with blood,which resulted in decreased 38. A 20-year-old woman came to the doctor
reabsorption of Na+ and water in renal with complaints of general weight loss, loss of
tubules.This can be explained by the effect of appetite, weakness, skin discoloration
the following factor on the kidneys: resembling bronze tan. In addtition to
A. Natriuretic hormone hyperpigmentation, examination in the hospital
B. Aldosterone revealed bilateral adrenal tuberculosis. What
C. Renin substance leads to skin hyperpigmentation,
D. Angiotensin when accumulated excessively?
E. Vasopressin A. Melanin
B. Bilirubin
34. A 32-year-old patient consulted a doctor C. Hemozoin
about the absence of lactation after parturition. D. Lipofuscin
Such disorder might be explained by the deficit E. Adrenochrome
of the following hormone:
A. Prolactin 39. A patient with gastric juice hypersecretion
B. Somatotropin has been recommended to exclude from the diet

17
hange E hange E
XC di XC di
F- t F- t
PD

PD
or

or
!

!
W

W
O

O
N

N
Y

Y
U

U
B

B
to

to
ww

ww
om

om
k

k
lic

lic
C

C
.c rich broths and vegetable infused water. A

.c
w

w
tr re tr re
.

.
ac ac
k e r- s o ft w a k e r- s o ft w a

doctor recommended it, because these food 44. A 40-year-old patient complains of intensive
products stimulate production of the following heartbeats, sweating, nausea, visual impairment,
hormone: arm tremor, hypertension. From his anamnesis:
A. Production of gastrin 2 years ago he was diagnosed with
B. Taste Receptors pheochromocytoma. Hyperproduction of what
C. Mechano-receptors of the oral cavity hormones causes the given pathology?
D. Mechanoreceptors of the stomach A. Catecholamines
E. Formation of secretin B. Aldosterone
C. Glucocorticoids
40. During an acute experiment some of diluted D. ACTH
solution of hydrochloric acid was injected into E. Thyroidal hormones
the duodenal cavity of an experimental animal.
This will result in hypersecretion of the 45. A special diet has led to a decrease in Ca2+
following hormone: ions in the blood. What hormone will increase
A. Secretin the secretion?
B. Gastrin A. Parathormone
C. Motilin B. Thyrocalcitonin
D. Neurotensin C. Vasopressin
E. Histamine D. Somatotropin
E. Thyroxine
41. A pregnant woman with a low labor activity
entered the maternity ward. Assign a hormonal 46. A patient has the sudden decrease of Ca2+
remedy to enhance labor activity: content in blood. What hormone secretion will
A. Oxytocin increase?
B. Progesterone A. Parathormone
C. Methandrostenolone B. Thyrocalcitonin
D. Hydrocortisone C. Aldosterone
E. ACTH D. Vasopressin
E. Somatotropin
42. A 26-year-old woman at 40 weeks pregnant
has been delivered to the maternity ward. 47. The calcium ions in the blood plasma of a
Objectively: the uterine cervix is opened, but patient is increased, reduced - in the bones. The
the contractions are absent. The doctor has excessive secretion of which hormone may
administered her a hormonal drug to stimulate cause such changes?
the labor. Name this drug: A. Parathormone
A. Oxytocin B. Thyroxine
B. Hydrocortisone C. Triiodothyronine
C. Estrone D. Thyreocalcitonin
D. Testosterone E. Aldosteron
E. ACTH
48. Kidneys of a man under examination show
43. To stimulate the laboractivity aparturient increased resorbtion of calcium ions and
woman was prescribed a drug a posterior decreased resorbtion of phosphate ions. What
pituitary hormone that does not affect the blood hormone causes this phenomenon?
pressure. As the pregnancy progresses, the A. Parathormone
sensitivity to this hormone increases.Name the B. Thyrocalcitonin
prescribed drug: C. Hormonal form D3
A. Oxytocin D. Aldosterone
B. Dinoprostone E. Vasopressin
C. Dinoprost
D. Pituitrin 49. Following thyroid surgery, a 47-year old
E. Ergotal female patient had fibrillary twitching of

18
hange E hange E
XC di XC di
F- t F- t
PD

PD
or

or
!

!
W

W
O

O
N

N
Y

Y
U

U
B

B
to

to
ww

ww
om

om
k

k
lic

lic
C

C
.c muscles in the arms, legs and face. These

.c
w

w
tr re tr re
.

.
ac ac
k e r- s o ft w a k e r- s o ft w a

disorders can be treated by the introduction of 54. A patient has hypocalcemia. What hormone
the following hormone: deficiency may be it’s cause?
A. Parathyroid hormone A. Parathormone
B. Triiodothyronine B. Thyrocalcitonin
C. Thyrotropin C. Aldosterone
D. Thyroxine D. Corticotropin
E. Thyroid-stimulating hormone E. Corticoliberin

50. During removal of the hyperplastic thyroid 55. A patient presents with osteoporosis;
gland of a 47-year-old woman, the parathyroid hypercalcemia and hypophosphatemia are
gland was damaged.One month after the surgery observed in the patient’s blood. What is the
the patient developed signs of cause of this condition?
hypoparathyroidism: frequent convulsions, A. Increased parathormone secretion
hyperreflexia, laryngospasm. What is the most B. Increased thyroxin secretion
likely cause of the patient’s condition? C. Inhibited parathormone secretion
A. Hypocalcemia D. Increased corticosteroid secretion
B. Hyponatremia E. Inhibited corticosteroid secretion
C. Hyperchlorhydria
D. Hypophosphatemia 56. A 5-month-old boy was hospitalized for
E. Hyperkalemia tonic convulsions. He has a lifetime history of
this disease. Examination revealed coarse hair,
51. A child has abnormal formation of tooth thinned and fragilenails, pale and dry skin. In
enamel and dentin as a result of low blood: calcium - 1,5 millimole/l, phosphor - 1,9
concentration of calcium ions in blood. Such millimole/l. These changes are associated with:
abnormalities might be caused by deficiency of A. Hypoparathyroidism
the following hormone: B. Hyperparathyroidism
A. Parathormone C. Hyperaldosteronism
B. Thyrocalcitonin D. Hypoaldosteronism
C. Thyroxin E. Hypothyroidism
D. Somatotropic hormone
E. Triiodothyronine 57. A 5-month-old boy was hospitalized for
tonic convulsions. Sick since birth. On
52. Owing to a pronounced decrease in plasma examination, the hair is hard, the nails are
calcium concentration, a 2-year-old child had thinned and brittle, the skin is pale and dry. In
tetanic reductions in the respiratory and the blood: calcium - 1.5 mmol / l, phosphorus -
pharyngeal muscles. Decreased secretion of 1.9 mmol / l. What are these changes related to?
what hormone can be the cause of this? A. Hypoparathyroidism
A. hormone B. Hyperparathyroidism
B. Calcitonin C. Hyperaldosteronism
C. Aldosterone D. Hypoaldosteronism
D. Somatotropin E. Hypothyroidism
E. Cortisol
58. Periodic renal colics attacks areobserved in
53. A 2-year-old child experienced convulsions a woman with primery hyperparathyroidizm.
because of lowering calciumions concentration Ultrasonic examination revealed small stones in
in the blood plasma. Function of what structure the kidneys. What is the most plausible reason
is decreased? of the stones’s formation?
A. Parathyroid glands A. Hypercalcemia
B. Hypophysis B. Hyperphosphatemia
C. Adrenal cortex C. Hypercholesterinemia
D. Pineal gland D. Hyperuricemia
E. Thymus E. Hyperkalemia

19
hange E hange E
XC di XC di
F- t F- t
PD

PD
or

or
!

!
W

W
O

O
N

N
Y

Y
U

U
B

B
to

to
ww

ww
om

om
k

k
lic

lic
C

C
.c B. Uric acid

.c
w

w
tr re tr re
.

.
ac ac
k e r- s o ft w a k e r- s o ft w a

59. A 56-year-old man presents with C. Cystine


parathyroid tumor. The following is observed: D. Bilirubin
muscle weakness, osteoporosis, bone E. Cholesterol
deformation, nephroliths consisting of oxalates
and phosphates. The patient’s condition is 64. Parodontitis is treated with calcium
caused by: preparations and a hormone that stimulates
A. Increased secretion of parathyroid hormone tooth mineralization and inhibits tissue
B. Decreased secretion of parathyroid hormone resorption. What hormone is it?
C. Increased secretion of calcitonin A. Calcitonin
D. Decreased secretion of calcitriol B. Parathormone
E. Increased secretion of thyroxin C. Adrenalin
D. Aldosterone
60. Due to trauma the patient’s parathyroid E. Thyroxine
glands have been removed, which resulted in
inertness, thirst, sharp increase of 65. A patient with hypoparathyreosis has
neuromuscular excitability. Metabolism of the multiple carious lesions of teeth. This pathology
following substance is disturbed: is caused by insufficiency of the following
A. Calcium hormone:
B. Manganese A. Calcitonin
C. Chlorine B. Thyroxin
D. Molybdenum C. Triiodothyronine
E. Zinc D. Thyroid-stimulating hormone
E. Somatotropin
61. There is a 9 year old boy in endocrinological
department, who has already had a few fractures 66. A 46-year-old patient suffering from the
of extremeties caused by fragility of bones. diffuse toxic goiter underwent resection of the
Malfunction of what endocrinous glands (gland) thyroid gland. After the surgery the patient
takes place? presents with appetite loss, dyspepsia, increased
A. Parathyroid glands neuromuscular excitement. The body weight
B. Thyroid gland remained unchanged. Body temperature is
C. Thymus normal. Which of the following has caused such
D. Adrenal glands a condition in this patient?
E. Epiphysis A. Reduced production of parathormone
B. Increased production of thyroxin
62. A patient with signs of osteoporosis and C. Increased production of calcitonin
urolithiasis has been admitted to the D. Increased production of thyroliberin
endocrinology department. Blood test revealed E. Reduced production of thyroxin
hypercalcemia and hypophosphatemia. These
changes are associated with abnormal synthesis 67. During surgery on the thyroid gland due to
of the following hormone: the disease of the Basedow's disease, the
A. Parathyroid hormone parathyroid glands were mistakenly removed.
B. Calcitonin There were convulsions, tetany. Exchange of
C. Cortisol which item was broken?
D. Aldosterone A. Calcium
E. Calcitriol B. Magnesium
C. Potassium
63. Cardinal symptoms of primary D. Iron
hyperparathyroidism are osteoporosis andrenal E. Sodium
lesion along with development of urolithiasis.
What substance makes up the basis of these 68. Soldiers injured in the midst of a battle may
calculi in this disease? not feel pain before it is completed. What
A. Calcium phosphate

20
hange E hange E
XC di XC di
F- t F- t
PD

PD
or

or
!

!
W

W
O

O
N

N
Y

Y
U

U
B

B
to

to
ww

ww
om

om
k

k
lic

lic
C

C
.c hormones of the opiate antinociceptive system C. Vasopresin

.c
w

w
tr re tr re
.

.
ac ac
k e r- s o ft w a k e r- s o ft w a

reduce the sensation of pain? D. Aldosterone


A. Endorphins E. Oxytocin
B. Serotonin

21
hange E hange E
XC di XC di
F- t F- t
PD

PD
or

or
!

!
W

W
O

O
N

N
Y

Y
U

U
B

B
to

to
ww

ww
om

om
k

k
lic

lic
C

C
.c

.c
Biochemistry of intercellular communications: hormones of steroid and
w

w
tr re tr re
.

.
ac ac
k e r- s o ft w a k e r- s o ft w a

thyroid nature
1. A girl is diagnosed with adrenogenital 6. Parents of a 10 y.o. boy consulted a doctor
syndrome (pseudohermaphroditism). This about extension of hair-covering, growth of
pathology was caused by hypersecretion of the beard and moustache, low voice. Intensified
following adrenal hormone: secretion of which hormone must be assumed?
A. Androgen A. Of testosterone
B. Estrogen B. Of somatotropin
C. Aldosterone C. Of oestrogen
D. Cortisol D. Of progesterone
E. Adrenalin E. Of cortisol

2. The secretion of which hypophysial 7. A female patient presents with endocrine


hormones will be inhibited after taking the oral dysfunction of follicular cells of the ovarian
contraceptives containing sexhormones? follicles resulting from an inflammation. The
A. Gonadotropic hormone synthesis of the following hormone will be
B. Vasopressin inhibited:
C. Thyrotrophic hormone A. Estrogen
D. Somatotropic hormone B. Progesterone
E. Ocytocin C. Lutropin
D. Follicle stimulating hormone
3. The woman was threatened with premature E. Follistatine
termination of pregnancy. This is most likely
due to insufficient secretion of such a hormone: 8. Intake of oral contraceptives containing sex
A. Progesterone hormones inhibits secretion of the hypophysiae
B. Estradiol hormones. Secretion of which of the indicated
C. Oxytocin hormones is inhibited while taking oral
D. Testosterone contraceptives with sexhormones?
E. Aldosterone A. Follicle-stimulating
B. Vasopressin
4. Testosterone and it’s analogs increase the C. Thyrotropic
mass of skeletal muscles that allows to use them D. Somatotropic
for treatment of dystrophy. Dueto interaction of E. Oxytocin
the hormone with what cell substrate is this
action caused? 9. Corticosteroid hormones regulate the
A. Nuclear receptors adaptation processes of the body as a whole to
B. Membrane receptors environmental changes and ensure the
C. Ribosomes maintenance of internal homeostasis. What
D. Chromatin hormone activates the hypothalamo-pituitary-
E. Proteins-activators of transcription adrenal axis?
A. Corticoliberin
5. A 30-year-old female exhibits signs of B. Somatoliberin
virilism (growth of body hair, balding temples, C. Somatostatin
menstrual disorders). This condition can be D. Corticostatin
caused by the over production of the following E. Thyroliberin
hormone:
A. Testosterone 10. A man prescribed by a doctor for a long
B. Oestriol time took the preparation of a group of
C. Relaxin glucocorticoid hormones. Which of the
D. Oxytocin following hormones will be depressed due to
E. Prolactin this?
A. Corticotropic
B. Somatotropic
22
hange E hange E
XC di XC di
F- t F- t
PD

PD
or

or
!

!
W

W
O

O
N

N
Y

Y
U

U
B

B
to

to
ww

ww
om

om
k

k
lic

lic
C

C
.c C. Tyrotropic C. Muscle glycogenolysis

.c
w

w
tr re tr re
.

.
ac ac
k e r- s o ft w a k e r- s o ft w a

D. Sexual D. Intestinal glucose absorption


E. Mineralocorticoid E. Synthesis of glycogen

11. The release of adrenal hormones is regulated 16. A patient with Itsenko-Cushing syndrome
by ACTH of the adenohypophysis. What has persistent hyperglycemia and glycosuria,
hormones are secreted by the adrenal glands hypertension, osteoporosis, obesity. What
under the action of the latter? hormon’s synthesis and secretion are intensified
A. Glucocorticoids in this case?
B. Androgenes A. Cortisol
C. Catecholamines B. Adrenaline
D. Mineralocorticoid C. Glucagon
E. Prostaglandins D. Thyroxin
E. Aldosterone
12. A patient has been taking glucocorticoids
for a long time. Drug withdrowal caused acute 17. On examination of the patient, the doctor
attack of his disease, blood pressure reduction, suspected Itsenko-Cushing's syndrome.
weakness. What are these occurences connected Determination of what substance in the patient's
with? blood will confirm the doctor's presumption
A. Adrenal glands insufficiency A. Cortisol
B. Drug habituation B. Tocopherol
C. Sensibilisation C. Retinol
D. Hyperproduction of corticotroph hormone D. Adrenaline
E. Cumulation E. Cholesterol

13. Chronic overdosage of glucocorticoids leads 18. A patient with a diagnosis of Itsenko-
to the development of hyperglycemia. What Cushing's disease (hyperproduction of adrenal
process of carbohydrate metabolism is hormones) has an increased concentration of
responsible for this effect? glucose, ketone bodies, sodium in the blood .
A. Gluconeogenesis What is the biochemical mechanism leading to
B. Glycogenolysis the occurrence of hyperglycemia?
C. Aerobic glycolisis A. Gluconeogenesis
D. Pentose-phosphate cycle B. Glycogenesis
E. Glycogenesis C. Glycogenolysis
D. Glycolysis
14. A 40-year-old woman with Cushing’s E. Aerobic glycolysis
disease presents with steroid diabetes. On
biochemical examination she has hyperglycemia 19. The patient was found to have obesity,
and hypochloremia. What process activates in hirsutism, a “moon-shaped” face, scarring of a
the first place in such patients? purple color on the skin of the thighs. Blood
A. Gluconeogenesis pressure is 180/110 mm Hg, blood glucose is
B. Glycogenolysis 17.2 mmol / l. What change in the production of
C. Glucose reabsorption adrenal hormones is possible such a picture?
D. Glucose transportation into a cell A. Hyperproduction of glucocorticoids
E. Glycolysis B. Hypoproducts of glucocorticoids
C. Hyperproduction of mineralocorticoids
15. A 28-year-old patient with Itsenko-Cushing D. Hypoproducts of mineralocorticoids
syndrome Hyperglycemia, glycosuria were E. Hypoproduction of adrenaline
detected hyperglycemia, glycosuria. The main
mechanism of hyperglycemia in this patient is 20. A 44 year old woman complains of general
stimulation: weakness, heart pain, significant increase of
A. Gluconeogenesis body weight. Objectively: moon face, hirsutism,
B. Liver glycogenolysis AP is 165/100 mm Hg, height - 164 cm, weight

23
hange E hange E
XC di XC di
F- t F- t
PD

PD
or

or
!

!
W

W
O

O
N

N
Y

Y
U

U
B

B
to

to
ww

ww
om

om
k

k
lic

lic
C

C
.c - 103 kg; the fat is mostly accumulated on her D. Diabetes mellitus

.c
w

w
tr re tr re
.

.
ac ac
k e r- s o ft w a k e r- s o ft w a

neck, thoracic girdle, belly. What is the main E. Essential hypertension


pathogenetic mechanism of obesity?
A. Increased production of glucocorticoids 25. A 38-year-old female patient complains of
B. Reduced production of thyroid hormones general weakness, cardiac pain, increased
C. Increased insulin production appetite, no menstruation. Objectively: the
D. Reduced glucagon production height is 166 cm, weight 108 kg, the patient has
E. Increased mineralocorticoid production moon-shaped face, subcutaneous fat is
deposited mainly in the upper body, torso and
21. A 16-year-old patient suffering from hips. There are also blood-red streaks. Ps-
Itsenko-Cushing's disease was counseled for 62/min, AP-160/105 mm Hg. Which of the
over weight. During the survey, it turned out following diseases is the described pattern of
that the energy value of the food consumed is obesity most typical for?
1700-1900 kcal / day. What is the leading cause A. Cushing pituitary basophilism
of obesity in this case? B. Alimentary obesity
A. Excess glucocorticoid C. Myxedema
B. Lack of insulin D. Insulinoma
C. Excess insulin E. Babinski-Frohlich syndrome
D. Lack of glucocorticoids
E. Hypodynamia 26. Examination of a patient with high blood
pressure revealed secondary arterial
22. A 29-year-old female patient has moon - hypertension. It was found that the cause of this
shaped face, upper body obesity, stretch marks condition of the patient is a hormonally active
on the abdomen, hirsutism; urine shows an tumor of the adrenal cortex. Hyperproduction of
increased rate of 17-oxy ketosteroids. What what hormone is the cause of secondary arterial
disease are these presentations typical for? hypertension in a patient?
A. Itsenko-Cushing syndrome A. Cortisol
B. Pheochromocytoma B. Adrenaline
C. Conn’s syndrome C. Thyroxine
D. Primary aldosteronism D. Insulin
E. Secondary aldosteronism E. Glucagon

23. The patient with complaints of permanent 27. Acceptance of corticosteroid analogues
thirst applied to the doctor. Hyperglycemia, causes the breakdown of muscle proteins to free
polyuria and increased concentration of 17- amino acids. In which process will amino acids
ketosteroids in the urine were revealed. What be involved in such conditions?
disease is the most likely? A. Gluconeogenesis in the liver
A. Steroid diabetes B. Muscle glycolysis
B. Insulin-dependent diabetes mellitus C. Synthesis of higher fatty acids
C. Myxoedema D. Glycogenolysis
D. Type I glycogenosis E. Decarboxylation
E. Addison’s disease
28. A 19-year-old male was found to have an
24. Examination of a 42 year old patient elevated level of potassium in the secondary
revealed a tumour of adenohypophysis. urine. These changes might have been caused
Objectively: the patient’s weight is 117 kg, he by the increase in the following hormone level:
has moon-like hyperemic face, redblue striae of A. Aldosterone
skin distension on his belly. Osteoporosis and B. Oxytocin
muscle dystrophy are present. AP is 210/140 C. Adrenaline
mm Hg. What is the most probable diagnosis? D. Glucagon
A. Cushing’s disease E. Testosterone
B. Cushing’s syndrome
C. Conn’s disease

24
hange E hange E
XC di XC di
F- t F- t
PD

PD
or

or
!

!
W

W
O

O
N

N
Y

Y
U

U
B

B
to

to
ww

ww
om

om
k

k
lic

lic
C

C
.c 29. A concentrated solution of sodium chloride 34. A humans are reduced diuresis,

.c
w

w
tr re tr re
.

.
ac ac
k e r- s o ft w a k e r- s o ft w a

was intravenously injected to ananimal. This hypernatremia and hypokalemia. What hormone
caused decreased reabsorption of sodium ions in hypersecretion can cause such changes?
the renal tubules. It is the result of the following A. Aldosterone
changes of hormonal secretion: B. Vasopressin
A. Aldosterone reduction C. Atrial Natriuretic Factor
B. Aldosterone increase D. Adrenaline
C. Vasopressin reduction E. Parathormone
D. Vasopressin increase
E. Reduction of atrial natriuretic factor 35. Atria of an experimental animal were
superdistended by blood that resulted in
30. A patient with adenoma of glomeral zone of decreased reabsorption of Na+ and water in
adrenal cortex (Conn’s disease) has arterial renal tubules. This can be explained by the
hypertension, convulsions, polyuria. What is the influence of the following factor upon kidneys:
main link in pathogenesis of these disorders? A. Natriuretic hormone
A. Aldosterone hypersecretion B. Aldosterone
B. Aldosterone hyposecretion C. Renin
C. Catecholamine hypersecretion D. Angiotensin
D. Glucocorticoid hypersecretion E. Vasopressin
E. Glucocorticoid hyposecretion
36. A patient has insufficient blood supply to
31. After a traffic accident a man presents with the kidneys, which has caused the development
severe blood loss, consciousness disturbance, of pressor effect due to the constriction of
low blood pressure, as well as compensatory arterial resistance vessels. This is the result of
activation of the reninangiotensin system, which the vessels being greately affected by the
results in: following substance:
A. Hyperproduction of aldosterone A. Angiotensin II
B. Increased blood coagulation B. Angiotensinogen
C. Intensification of erythropoiesis C. Renin
D. Hyperproduction of vasopressin D. Catecholamines
E. Intensification of heart contractions E. Norepinephrine

32. A patient has hyperkaliemia and 37. Examination of a patient with high blood
hyponatremia. Reduced secretion of what pressure showed him secondary arterial
hormone may cause such changes? hypertension. The cause of this condition is a
A. Aldosterone renin-producing tumor of the kidney. What is
B. Vasopressin the main link in the pathogenesis of secondary
C. Cortosol arterial hypertension in a patient?
D. Parathormone A. Angiotensin 2, aldosterone hyperproduction
E. Natriuretic hormone B. Cortisol hyperproduction
C. Insulin hyperproduction
33. People adapted to high external D. Insufficient vasopressin production
temperatures have such pecularity: profuse E. Insufficient catecholamines production
sweating isn’t accompanied by loss of large
volumes of sodium chloride.This is caused by 38. A patient with cirrhosis of the liver has
the effect of the following hormone upon the sustained arterial hypotension (BP is 90/50 mm
perspiratory glands: Hg). What is the reason for lowering blood
A. Aldosterone pressure in this pathology of the liver?
B. Vasopressin A. Reduced angiotensin synthesis
C. Cortisol B. Increased synthesis of Na-uretic hormone
D. Tgyroxin C. Excessive inactivation of vasopressin
E. Natriuretic D. Increasing the reflex influence from the
receptor zone of the aortic arch

25
hange E hange E
XC di XC di
F- t F- t
PD

PD
or

or
!

!
W

W
O

O
N

N
Y

Y
U

U
B

B
to

to
ww

ww
om

om
k

k
lic

lic
C

C
.c E. Activation of the kallikrein-kinin system

.c
w

w
tr re tr re
.

.
ac ac
k e r- s o ft w a k e r- s o ft w a

44. Indirect calorimetry allowed to establish that


39. A patient with kidney disease has high blood a 30-year-old male patient had a 30% decrease
pressure, espesially the diastolic one. in basal metabolic rate. This might be caused by
Hypersecretion of what biologically active the reduced concentration of the following
substance causes blood pressure rise? hormones in blood plasma:
A. Renin A. Triiodothyronine, tetraiodothyronine
B. Adrenaline B. Thyrocalcitonin, parathormone
C. Noradrenaline C. Glucocorticoids
D. Vasopressin D. Catecholamines
E. Catecholamines E. Somatoliberin, somatostatin

40. Under the experimental conditions, a rabbit 45. A 37-year-old patient has has lost 5 kg in
was tied up the renal artery, it was resulted in a weight over the past three months, he complains
significant increase in blood pressure after 2 of hand tremor, excessive sweating,
weeks. As a result of an increase in the secretion exophthalmos, tachycardia. These changes
of a biologically active substance, this might have been caused by the increased
happened? secretion of the following hormone:
A. Renin A. Thyroxine
B. Adrenaline B. Cortisol
C. Vasopressin C. Insulin
D. Norepinephrine D. Glucagon
E. Natriuretic hormone E. Thyrocalcitonin

41. A patient who had been continuously taking 46. A 38-year-old woman complains of
drugs blocking the production of angiotensin II excessive sweating, palpitations, fever in the
developed bradycardia and arrhythmia. A likely evening hours. The main exchange is increased
cause of these disorders is: by 60%. The doctor decided the diagnosis -
A. Hyperkalemia thyrotoxicosis. What properties of thyroxin lead
B. Hypokalemia to increased heat production?
C. Hypernatremia A. Separates oxidative phosphorylation
D. Hypocalcemia B. Increases conjugation of oxidation and
E. Hypercalcemia phosphorylation
C. Reduces β-oxidation of fatty acids
42. A patient with kidney disease has high blood D. Reduces amino acid deamination
pressure, espesially the diastolic one. E. Promotes acetyl-coA accumulation
Hypersecretion of what biologically active
substance causes blood pressure rise? 47. A 40-year-old woman suffering from diffuse
A. Renin toxic goiter presents with constant increase of
B. Adrenaline her body temperature. What mechanism results
C. Noradrenaline in such clinical presentation?
D. Vasopressin A. Separation of oxidation and phosphorization
E. Catecholamines in cell mitochondria
B. Increased breakdown of glycogen inhepatic
43. A 40-year-old woman on examination cells
presents with intensified basal metabolic rate. C. Increased catabolism of protein incells
What hormone present in excess leads to such D. Increased excitability of nerve cells
condition? E. Increased cell sensitivity to catecholamines
A. Triiodothyronine
B. Thyrocalcitonin 48. A patient is followed up in an
C. Glucagon endocrinological dispensary on account of
D. Aldosterone hyperthyreosis. Weight loss, tachycardia, finger
E. Somatostatin tremor are accompanied by hypoxia symptoms -

26
hange E hange E
XC di XC di
F- t F- t
PD

PD
or

or
!

!
W

W
O

O
N

N
Y

Y
U

U
B

B
to

to
ww

ww
om

om
k

k
lic

lic
C

C
.c headache, fatigue, eye flicker. What mechanism 53. A doctor has established significant growth

.c
w

w
tr re tr re
.

.
ac ac
k e r- s o ft w a k e r- s o ft w a

of thyroid hormones action underlies the retardation, disproportional body build, and
development of hypoxia? mental deficiency of a child. What is the most
A. Disjunction, oxydation and phosphorilation likely cause of this pathology?
B. Inhibition of respiratory ferment synthesis A. Hypothyroidism
C. Competitive inhibition of respiratory B. Insufficient nutrition
ferments C. Hyperthyroidism
D. Intensification of respiratory ferment D. Genetic defects
synthesis E. Hypopituitarism
E. Specific binding of active centres of
respiratory ferments 54. A 12-year-old child is of short stature, has
disproportionate body structure and mental
49. A 56 y.o. patient has been suffering from retardation. These characteristics might be
thyreotoxicosis for a long time.What type of caused by the hyposecretion of the following
hypoxia can be developed? hormone:
A. Tissue A. Thyroxine
B. Hemic B. Insulin
C. Circulatory C. Cortisol
D. Respiratory D. Somatotropin
E. Mixed E. Glucagon

50. The patient mistakenly took an excess dose 55. Inhabitants of territories with cold climate
of thyroxine. What changes in secretion of have high content of an adaptive
thyroliberin and thyrotropin will it lead to? thermoregulatory hormone. What hormone is
A. Secretion of hormones will decrease meant?
B. Secretion of hormones will increase. A. Thyroxin
C. Secretion of hormones will not change B. Insulin
D. Secretion of thyroliberin will increase, C. Glucagon
thyrotropin will decrease D. Somatotropin
E. Secretion of thyrotropin will increase, E. Cortisol
thyroliberin will decrease
56. The clinical examination of the patient
51. A 19-year-old female suffers from revealed an increase in the thyroid gland
tachycardia in rest condition, weight loss, (goiter), an increase in basal metabolism, loss of
excessive sweating, exophtalmos and body weight, an imbalance of heat, an increase
irritability. What hormone would you expect to in appetite, an increase in excitability and
find elevated in her serum? irritability, exophthalmos and tachycardia were
A. Thyroxine detected. What endocrine disruption leads to
B. Cortisol these symptoms?
C. Mineralocorticoids A. Hyperfunction of the thyroid gland
D. ACTH B. Hypofunction of the parathyroid glands
E. Insulin C. Pituitary Hyperfunction
D. Epiphysis hypofunction
52. A child presents with symptoms of psychic E. Hypofunction of the thyroid gland
and physical retardation (cretinism). It is usually
associated with the following hormone 57. It is known that steroid anti-inflammatory
deficiency: drugs inhibit the activity of phospholipase A2,
A. Thyroxin which is necessary for the synthesis of
B. Somatotropic prostaglandins. What substance is the precursor
C. Calcitonin of these inflammatory mediators?
D. Insulin A. Arachidonic acid
E. Testosterone B. Cholesterol
C. Tyrosine

27
hange E hange E
XC di XC di
F- t F- t
PD

PD
or

or
!

!
W

W
O

O
N

N
Y

Y
U

U
B

B
to

to
ww

ww
om

om
k

k
lic

lic
C

C
.c D. Proopiomelanocortin 59. Utilization of arachidonic acid via

.c
w

w
tr re tr re
.

.
ac ac
k e r- s o ft w a k e r- s o ft w a

E. Palmitic acid cyclooxigenase pathway results in formation of


some bioactive substances. Name them:
58. A patient is followed up in the clinic on A. Prostaglandins
account of for pneumonia complicated by B. Thyroxine
pleurisy. He was given prednisolon as part of a C. Biogenic amins
combination therapy. The anti-inflammatory D. Somatomedins
effect of this synthetic glucocorticoid is E. Insulin-like growth factors
associated with blocking the release of
arachidonic acid by inhibiting: 60. Experimental studies revealed steroid
A. Phospholipase A2 hormones to have an effect on proteosynthesis.
B. Cyclo-oxygenase They influence synthesis of the following
C. Phospholipase C substances:
D. Lipoxygenase A. Specific messenger RNA
E. Peroxidase B. Adenosine triphosphate
C. Specific transfer RNA
D. Guanosine triphosphate
E. Specific ribosomal RNA

28
hange E hange E
XC di XC di
F- t F- t
PD

PD
or

or
!

!
W

W
O

O
N

N
Y

Y
U

U
B

B
to

to
ww

ww
om

om
k

k
lic

lic
C

C
.c

.c
Biochemistry and pathobiochemistry of blood
w

w
tr re tr re
.

.
ac ac
k e r- s o ft w a k e r- s o ft w a

1. A 7-year-old girl has signsof anemia. E. -


Laboratory examination revealed pyruvate
kinase deficiency in erythrocytes. What process 6. Sulfanilamides are applied as antimicrobal
disturbance plays the main role in agents in clinical practice.Sulfanilamide
anemiadevelopment? treatment, however, canresult in hemolytic
A. Anaerobic glycolysis anemia developmentin patients that suffer from
B. Oxidative phosphorylation geneticdefect of the following enzyme
C. Tissue respiration ofpentose phosphate metabolism inerythrocytes:
D. Peroxide decomposition A. Glucose-6-phosphate dehydrogenase
E. Aminoacids desamination B. Hexokinase
C. Transketolase
2. Human red blood cells contain no D. Transaldolase
mitochondria. What is the main pathway E. Pyruvate kinase
forATP production in these cells?
A. Anaerobic glycolysis 7. Biochemical analysis of an
B. Aerobic glycolysis infant’serythrocytes revealed evident
C. Oxidative phosphorylation glutathioneperoxidase deficiency and low
D. Creatine kinase reaction concentration of reduced glutathione. What
E. Cyclase reaction pathological condition can develop in this
infant?
3. Erythrocytes of the patient withhemolytic A. Hemolytic anemia
anemia present with significantdecrease of B. Pernicious anemia
pyruvate kinase activity. Whatmethabolic C. Megaloblastic anemia
process is disturbed in thiscase? D. Sicklemia
A. Glycolysis E. Iron-deficiency anemia
B. Glycogenolysis
C. Gluconeogenesis 8. A 20 year old patient complains ofgeneral
D. Pentose-phosphate pathway of weakness, dizziness, quick fatigability. Blood
glucoseoxidation analysis results: Hb-80g/l. Microscopical
E. Glycogen synthesis examination results:erythrocytes are of modified
form. Thiscondition might be caused by:
4. A 3 year old child with fever wasgiven A. Sickle-cell anemia
aspirin. It resulted in intensified erythrocyte B. Hepatocellular jaundice
haemolysis. Hemolyticanemia might have been C. Acute intermittent porphyria
caused bycongenital insufficiency of the D. Obturative jaundice
followingenzyme: E. Addison’s disease
A. Glucose 6-phosphate dehydrogenase
B. Glucose 6-phosphatase 9. With a number of hemoglobinopathies, amino
C. Glycogen phosphorylase acid substitutions occur in the α- and β-chains of
D. Glycerol phosphate dehydrogenase hemoglobin. Which of them is characteristic of
E. γ-glutamiltransferase Hb S (sickle cell anemia)?
A. Glutamate-valine
5. There is an increased tendency of B. Aspartate-lysine
erythrocytes to hemolysis in patients with a C. Alanin-serine
hereditary defect of glucose-6-phosphate D. Methionine-histidine
dehydrogenase. What metabolic process is E. Glycine-serine
impaired under these conditions?
A. Pentose phosphate pathway of glucose 10. Substitution of the glutamic acid onvaline
oxidation was revealed while examining initial molecular
B. Gluconeogenesis structure. For what inheritedpathology is this
C. Aerobic glucose oxidation symptom typical?
D. Synthesis of glycogen A. Sickle-cell anemia
29
hange E hange E
XC di XC di
F- t F- t
PD

PD
or

or
!

!
W

W
O

O
N

N
Y

Y
U

U
B

B
to

to
ww

ww
om

om
k

k
lic

lic
C

C
.c B. Thalassemia vitamin deficiency leads to such changes train

.c
w

w
tr re e
ar
.

.
ac
k e r- s o ft w a cke
r- s o ft w
C. Minkowsky-Shauffard disease blood?
D. Favism A. B12
E. Hemoglobinosis B. C
C. P
11. A patient suffers from mutation of a gene D. B6
that corresponds with hemoglobin synthesis. E. PP
This condition led to development of sicklecell
disease. Name the pathologicalhemoglobin 16. A 43-year-old patient with chronic atrophic
characteristic of this disease: gastritis and megaloblastic hyperchromic
A. HbS anemia excreted methylmalonic acid in the
B. HbA urine. What vitamin deficiency is caused by the
C. HbF occurrence of the specified symptom complex?
D. HbA1 A. В12
E. Bart-Hb B. В2
C. В3
12. Along with normal hemoglobin typesthere D. В5
can be pathological ones in the organism of an E. В6
adult. Name one of them:
A. HbS 17. A pregnant woman turned to the
B. HbF obstetrician-gynecologist, who was diagnosed
C. HbA1 with megaloblastic anemia. Which of the
D. HbA2 following drugs should be prescribed?
E. HbO2 A. Cyanocobalamin
B. Pentoxyl
13. After an extended treatment with C. Methyluracil
sulfanamides a patient has developedmacrocytic D. Glaucin
anemia. Production of activeforms of the E. Streptokinase
following vitamin is disruptedin such a
condition: 18. The patient was diagnosed with
A. Folic acid megaloblastic anemia. Specify a vitamin
B. Thiamine deficiency which can lead to the development of
C. Riboflavin this disease.
D. Pyridoxine A. Cyanocobalamin.
E. Cyanocobalamin B. Rutin.
C. Nicotinamide.
14. A 50-year-old patient has beenexamined by D. Thiamine.
a dentist and found tohave crimson smooth E. Cholecalciferol
tongue. Bloodanalysis revealed a decrease in
RBC leveland hemoglobin concentration, 19. In a 65-year-old patient with prolonged
symptoms of megaloblastichematopoiesis, complaints characteristic of chronic gastritis,
degenerative changes inWBCs. What blood megalocytes were found in peripheral blood,
disorder was found inthis patient? and megaloblastic erythropoiesis was found in
A. B12-folic-acid-deficiency anemia the bone marrow. What is the most likely
B. Iron deficiency anemia diagnosis?
C. Myeloid leukemia A. B12-folic deficiency anemia
D. Aplastic anemia B. Aplastic anemia
E. Hemolytic anemi C. Hypoplastic anemia
D. Hemolytic anemia
15. After removal in the patient 2/3 of the E. Iron deficiency anemia
stomach in the blood decreased the amount of
hemoglobin, the number of erythrocytes, A patient who had subtotal gastrectomy 5 years
increased the size of these blood counts. What ago developed B12 folic acid deficiency

30
hange E hange E
XC di XC di
F- t F- t
PD

PD
or

or
!

!
W

W
O

O
N

N
Y

Y
U

U
B

B
to

to
ww

ww
om

om
k

k
lic

lic
C

C
.c anemia. What is the leading mechanism in the hemolysis of red blood cells and

.c
w

w
tr re tr re
.

.
ac ac
k e r- s o ft w a k e r- s o ft w a

development of such anemia? hemoglobinuria were detected. The action of


A. The absence of an internal factor Castle snake venom due to the presence of the enzyme
B. The absence of an external factor Castle in it:
C. Impaired absorption of vitamin B12 in the A. Phospholipase A2
small intestine B. Phospholipase A1
D. Folic acid deficiency C. Phospholipase C
E. Transcobalamin deficiency. D. Phospholipase D
E. Sphingomyelinase
20. Examination of a patient, suffering from
atrophic gastritis, revealedmegaloblastic 25. For people who permanently reside in highlands,
anemia. The anemia is likely to be caused by the adaptation to "oxygen starvation" is carried out by
deficiency of the following substance: facilitating the release of hemoglobin oxygen due to:
A. Gastromucoproteid A. Increased formation of 2,3-diphosphoglycerate in
B. Vitamin B6 erythrocytes
B. Reduced formation of 2,3-diphosphoglycerate in
C. Vitamin B1 erythrocytes
D. Iron C. The increase in CO2 partial pressure
E. Erythropoietins D. Increasing blood pH
E. Decreased blood temperature
21. Surgical removal of a part of
stomachresulted in disturbed absorption of 26. After the accident in the chemical industry,
vitamin B12, it is excreted with feces. the environment was polluted with nitro
Thepatient was diagnosed with anemia. compounds. The people living in this area, there
Whatfactor is necessary for absorption of was a sharp weakness, headache, shortness of
thisvitamin? breath, dizziness. What is the cause of hypoxia?
A. Gastromucoprotein A. Methemoglobin formation
B. Gastrin B. Inhibition of dehydrogenase
C. Hydrochloric acid C. Formation of carboxyhemoglobin
D. Pepsin D. Reduced function of flavin enzymes
E. Folic acid E. Inactivation of cytochrome oxidase

22. Examination of a 52-year-oldfemale patient 27. A 7-year-old child presents with marked
has revealed a decreasein the amount of red signs of hemolytic anemia. Biochemical
blood cells andan increase in free hemoglobin analysis of erythrocytes determined low
inthe blood plasma (hemoglobinemia).What concentration of NADPH and reduced
type ofanemia is being observed in the patient? glutathione. What enzyme is deficient in this
A. Acquired hemolytic case leading to the biochemical changes and
B. Hereditary hemolytic their clinical manifestations?
C. Acute hemorrhagic A. Glucose-6-phosphate dehydrogenase
D. Chronic hemorrhagic B. Hexokinase
E. Anemia due to diminishederythropoiesis C. Fructokinase
D. Pyruvate kinase
23. A patient with hemolytic anemia showed E. Lactate dehydrogenase
pyruvate kinase deficiency in erythrocytes. In these
conditions, the cause of hemolysis of red blood cells
28. A 38 year old patient takes aspirin and
are:
sulfanilamides. After their intake intensified
A. Reduced Na+, K+ -ATPases
B. Na+ deficiency in red blood cells erythrocyte haemolysis is observed which is
C. Excess K+ in red blood cells caused by deficiency of glucose 6-phosphate
D. Genetic defects of glycophorin A dehydrogenase. This pathologyis caused by
E. Spectrin deficiency failure of the following coenzyme:
A. NADP – H
24. During laboratory examination of the blood B. FAD - H2
of a person who was bitten by a snake, C. Pyridoxal phosphate

31
hange E hange E
XC di XC di
F- t F- t
PD

PD
or

or
!

!
W

W
O

O
N

N
Y

Y
U

U
B

B
to

to
ww

ww
om

om
k

k
lic

lic
C

C
.c D. FMN - H2 B. Cytochrome oxidase

.c
w

w
tr re tr re
.

.
ac ac
k e r- s o ft w a k e r- s o ft w a

E. Ubiquinone C. Succinate dehydrogenase


D. Ketoglutarate dehydrogenase
29. A 22 year old woman has been taking E. Aconitase
sulfanilamides for a long time that led to
symptoms of hemolytic anaemia caused by 34. When periodontitis develops lipid
hereditary disturbance of synthesis of glucose 6- peroxidation in periodontal tissues, the content
phosphate dehydrogenase. This enzyme of of malondialdehyde, hydrogen peroxide
pentose-phosphate cycleis responsible for increases in the oral cavity. Which of the
generation of: following enzymes provide antioxidant
A. NADP - H2 protection?
B. NAD A. Superoxide dismutase, catalase
C. FAD B. Amylase, trypsin
D. FMN C. Maltase, chymotrypsin
E. ATP D. Lactase, lysozyme
E. Sucrase, prothrombin
30. It is known that pentose-phosphate pathway
actively functions in the erythrocytes. What is 35. Periodontitis induces the development of
the main function of this metabolic pathway in lipid peroxidation in the periodontal tissues, as
the erythrocytes? well as an increase in malondialdehyde and
A. Counteraction to lipid peroxidation hydrogen peroxide concentration in the oral
B. Activation of microsomal oxidation cavity. Which of the following enzymes
C. Neutralization of xenobiotics provides antioxidant protection?
D. Oxidation of glucose into lactate A. Catalase
E. Increase of lipid peroxidation B. Amylase
C. Maltase
31. In course of metabolic process active forms D. Lactase
of oxygen including superoxide anion radical E. Invertase
are formed in the human body. By means of
what enzyme is this anion inactivated? 36. Those organisms which in the process of
A. Superoxide dismutase evolution failed to develop protection from
B. Catalase H2O2 can exist only in anaerobic conditions.
C. Peroxidase Which of the following enzymes can break
D. Glutathioneperoxidase hydrogen peroxide down?
E. Glutathionereductase A. Peroxidase and catalase
B. Oxygenase and hydroxylase
32. Reduced activity of antioxidant enzymes C. Cytochrome oxidase, cytochrome B5
enhances peroxidation of cell membrane lipids. D. Oxygenase and catalase
The reduction of glutathione peroxidase activity E. Flavin-dependent oxidase
is caused by the following microelement
deficiency: 37. Patient with abscess of the cut wound
A. Selenium applied to the traumatological department. In
B. Molybdenum order to clean the wound from the pus doctor
C. Cobalt washed it with 3% hydrogen peroxide. Foam
D. Manganese was absent. What caused the absence of the
E. Copper drug activity?
A. Inherited insufficiency of catalase
33. In pathological processes accompanied by B. Low concentration H2O2
hypoxia, an incomplete reduction of the oxygen C. Inherited insufficiency of erythrocyte
molecule in the respiratory chain and phosphatdehydrogenase
accumulation of hydrogen peroxide occurs. D. Shallow wound
Specify the enzyme that ensures its destruction: E. Pus in the wound
A. Catalase

32
hange E hange E
XC di XC di
F- t F- t
PD

PD
or

or
!

!
W

W
O

O
N

N
Y

Y
U

U
B

B
to

to
ww

ww
om

om
k

k
lic

lic
C

C
.c 38. A worker has decreased buffer capacity of B. Ketone bodies

.c
w

w
tr re tr re
.

.
ac ac
k e r- s o ft w a k e r- s o ft w a

blood due to exhausting muscular work. The C. Acetyl CoA


influx of what acid substance in the blood can D. Glucose 6-phosphate
cause this symptom? E. Oxaloacetate
A. Lactate
B. Pyruvate 43. A 32-year-old female patient suffers from
C. 1,3-bisphosphoglycerate gingivitis accompanied by gum hypoxia. What
D. α-ketoglutarate metabolite of carbohydrate metabolism is
E. 3-phosphoglycerate produced inthe periodontium tissues more
actively in this case?
39. A child during the first 3 months after birth A. Lactate
developed a severe form of hypoxia, which was B. Ribose 5-phosphate
manifested by asphyxia and cyanosis of skin. C. Glycogen
The reason for this is a violation of the D. Glucose 6-phosphate
replacement of fetal hemoglobin on: E. NADPH-H
A. Hemoglobin A
B. Hemoglobin S 44. A 29-year-old patient was delivered to a
C. Glycated hemoglobin hospital because of intoxication with carbon
D. Methemoglobin monoxide. Objectively: the patient presents with
E. Hemoglobin M symptoms of severe hypoxia- evident dyspnea,
cyanosis, tachycardia. What compound is
40. Diseases of the respiratory system and produced as a result of intoxication with carbon
circulatory disorders impair the transport of monooxide?
oxygen, thus leading to hypoxia. Under these A. Carboxyhemoglobin
conditions the energy metabolism is carried out B. Methemoglobin
byanaerobic glycolysis. As a result, the C. Carbhemoglobin
following substance is generated and D. Sulfhemoglobin
accumulated in blood: E. Oxyhemoglobin
A. Lactic acid
B. Pyruvic acid 45. A man lost consciousness in a car with
C. Glutamic acid running engine where he had been waiting for a
D. Citric acid friend for a long time. What hemoglobin
E. Fumaric acid compaund can be found in the blood of the
patient?
41. A patient is followed up in an A. Carboxyhemoglobin
endocrinological dispensary on account of B. Deoxyhemoglobin
hyperthyreosis. Weight loss, tachycardia, finger C. Carbhemoglobin
tremor are accompanied by hypoxia symptoms - D. Methemoglobin
headache, fatigue, eye flicker. What mechanism E. Oxyhemoglobin
of thyroid hormones action underlies the
development of hypoxia? 46. A patient with respiratory failure has blood
A. Disjunction oxydation and phosphorilation pH of 7,35. pCO2 test revealed hypercapnia.
B. Inhibition of respiratory fermentsynthesis Urine pH test revealed an increase in the urine
C. Competitive inhibition of respiratoryferments acidity. What form of acid-base imbalance is the
D. Intensification of respiratory case?
fermentsynthesis A. Compensated respiratory acidosis
E. Specific binding of active centres B. Compensated metabolic acidosis
ofrespiratory ferments C. Decompensated metabolic acidosis
D. Compensated respiratory alkalosis
42. After a sprint an untrained person develops E. Decompensated respiratory alkalosis
muscle hypoxia. This leads to the accumulation
of the following metabolite in muscles: 47. An infant has apparent diarrhea resulting
A. Lactate from improper feeding. One of the main

33
hange E hange E
XC di XC di
F- t F- t
PD

PD
or

or
!

!
W

W
O

O
N

N
Y

Y
U

U
B

B
to

to
ww

ww
om

om
k

k
lic

lic
C

C
.c diarrhea effects is plentiful excretion of sodium 52. Prophylactic examination of a patient

.c
w

w
tr re tr re
.

.
ac ac
k e r- s o ft w a k e r- s o ft w a

bicarbonate. What form of acid-base balance revealed hyperglycemia, ketonuria, polyuria,


disorder is the case? glycosuria. What form of acid-base balance
A. Metabolic acidosis disorder is the case?
B. Metabolic alkalosis A. Metabolic acidosis
C. Respiratory acidosis B. Gaseous acidosis
D. Respiratory alkalosis C. Nongaseous acidosis
E. No disorders of acid-base balance will be D. Gaseous alkalosis
observed E. Metabolic alkalosis

48. A 30-year-old man with diabetes mellitus 53. A patient with diabetes developed a diabetic
type I was hospitalised. The patient is comatose. coma due to the acid-base imbalance. Specify
Laboratory tests revealed hyperglycemia and the kind of this imbalance:
ketonemia. What metabolic disorder can be A. Metabolic acidosis
detected in this patient? B. Metabolic alkalosis
A. Metabolic acidosis C. Respiratory acidosis
B. Metabolic alkalosis D. Gaseous alkalosis
C. Respiratory acidosis E. Non-gaseous alkalosis
D. Respiratory alkalosis
E. Normal acid-base balance 54. Ketoacidosis that develops due to
accumulation of ketone bodies in blood serum is
49. A patient suffers from disrupted patency of a primary complication of diabetes mellitus.
the airways at the level of small and medium- What acid-base disbalance develops during this
sized bronchial tubes. What changes of acid- condition?
base balance can occur in the patient? A. Metabolic acidosis
A. Respiratory acidosis B. Metabolic alkalosis
B. Respiratory alkalosis C. Respiratory acidosis
C. Metabolic acidosis D. Respiratory alkalosis
D. Metabolic alkalosis E. –
E. Acid-base balance remains unchanged
55. Due to recurring vomiting a patient has lost
50. Diabetes mellitus causes ketosis as a result significant amount of gastric juice, which led to
of activated oxidation of fatty acids. What development of acidbase dysbalance. What type
disorders of acid-base equilibrium may be of acid-base dysbalance has developed?
caused by excessive accumulation of ketone A. Nongaseous alkalosis
bodies in blood? B. Gaseous acidosis
A. Metabolic acidosis C. Nongaseous acidosis
B. Metabolic alcalosis D. Gaseous alkalosis
C. Any changes woun’t happen E. Metabolic acidosis
D. Respiratory acidosis
E. Respiratory alcalosis 56. A mountaineer who rose to a height of
5200m developed gas alkalosis. What is the
51. Ketosis develops in the patients with cause of its development?
diabetes mellitus, as the result of activation of A. Hyperventilation
fatty acids oxidation processes. What acidbase B. Hypoventilation of the lungs
imbalance can result from accumulation of C. Hyperoxemia
excessive ketone bodies in the blood? D. Hypoxemia
A. Metabolic acidosis E. Reduced ambient temperature
B. Metabolic alkalosis
C. No imbalance occurs 57. In a laboratory study of the respiratory
D. Respiratory acidosis function of the blood, it has been established
E. Respiratory alkalosis that there is a deterioration of CO2 transport.

34
hange E hange E
XC di XC di
F- t F- t
PD

PD
or

or
!

!
W

W
O

O
N

N
Y

Y
U

U
B

B
to

to
ww

ww
om

om
k

k
lic

lic
C

C
.c What enzyme deficiency in red blood cells can examination revealed anaemia and presence traof

.c
w

w
tr re e
ar
.

.
ac
k e r- s o ft w a cke
r- s o ft w
this be due to? the paraprotein in the zone of gamma-globulins.
A. Carboanhydrase To confirm the myeloma diagnosis it is
B. 2,3-diphosphoglycerate necessary to determine the following index in
C. Adenylate cyclase the patient’s urine:
D. Protein kinase A. Bence Jones protein
E. Phosphorylase B. Bilirubin
C. Haemoglobin
58. In the process of hemoglobin catabolismiron D. Ceruloplasmin
is released and then as a part of special transport E. Antitrypsin
protein is returned to the bonemarrow, to be
used again for hemoglobinsynthesis. Name this 63. A 16-year-old girl, who has been starving
transport protein: herself for a long time to lose weight, developed
A. Transferrin an edema. This phenomenon is mainly caused
B. Transcobalamin by:
C. Haptoglobin A. Hypoproteinemia due to protein synthesis
D. Ceruloplasmin disturbance
E. Albumin B. Hypoglycemia due to glycogen synthesis
disturbance
59. Hemoglobin catabolism results in release of C. Venous congestion and increased venous
iron which is transported to the bone marrow by pressure
a certain transfer protein and used again for the D. Deceleration of glomerular filtration rate
synthesis of hemoglobin. Specify this transfer E. Decreased production of vasopressin in the
protein: hypothalamus
A. Transferrin (siderophilin)
B. Transcobalamin 64. A 36-year-old female patient who has been
C. Haptoglobin limiting the number of food stuffs in her diet for
D. Ceruloplasmin 3 months presents with a decrease in body
E. Albumin weight, deterioration of physical and mental
health, face edemata. These changes may be
60. In the liver of a patient suffering from iron caused by the deficiency of the following
deficiency anemia, a violation of the synthesis nutrients:
of iron-containing protein, which is a source of A. Proteins
iron for heme synthesis, was found. What is the B. Vitamins
name of this protein? C. Fats
A. Ferritin D. Carbohydrates
B. Transferrin E. Micronutrients
C. Hemosiderin
D. Ceruloplasmin 65. Upon toxic damage of hepatic cells resulting
E. Hemoglobin in disruption of liver function the patient
developed edemas. What changes of blood
61. In men of 40 years as a result of enhanced plasma are the main cause of edema
hemolysis of erythrocytes, the iron content in development?
the blood plasma increased. What protein A. Decrease of albumin content
provides its deposition in the tissues? B. Increase of globulin content
A. Ferritin C. Decrease of fibrinogen content
B. Haptoglobin D. Increase of albumin content
C. Transferrin E. Decrease of globulin content
D. Transcortin
E. Albumin 66. In case of toxic damage to hepatocytes with
a violation of their protein synthesis function,
62. A patient complains about dyspnea the patient's content of albumin in the blood
provoked by the physical activity. Clinical plasma and oncotic pressure of the plasma

35
hange E hange E
XC di XC di
F- t F- t
PD

PD
or

or
!

!
W

W
O

O
N

N
Y

Y
U

U
B

B
to

to
ww

ww
om

om
k

k
lic

lic
A. γ-globulins
C

C
.c sharply decreased. What will be the result of

.c
w

w
tr re tr re
.

.
ac ac
k e r- s o ft w a k e r- s o ft w a

these changes? B. Albumins


A. The appearance of edema C. α1-globulins
B. Reduced diuresis D. α2-globulins
C. Reduced ESR E. β-globulins
D. Increased circulating blood volume
E. Increased blood viscosity 72. When the inflammatory process is activated,
some autoimmune and infectious diseases in the
67. Toxic affection of liver results in blood plasma sharply increase the level of
dysfunction of protein synthesis. It is usually proteins of the acute phase. Which of the
accompanied by the following kind of following proteins can form a gel when the
dysproteinemia: whey cools?
A. Absolute hypoproteinemia A. Cryoglobulin
B. Relative hypoproteinemia B. Haptoglobin
C. Absolute hyperproteinemia C. Ceruloplasmin
D. Relative hyperproteinemia D. C-reactive protein
E. Paraproteinemia E. α2 macroglobin

68. The concentration of albumins in human 73. A 49-year-old male patient with acute
blood sample is lower than normal. This leads to pancreatitis was likely to develop pancreatic
edema of tissues. What blood function is necrosis, while active pancreatic proteases were
damaged? absorbed into the blood stream and tissue
A. Maintaining the oncotic blood pressure proteins broke up. What protective factors of the
B. Maintaining the Ph level body can inhibit these processes?
C. Maintaining the body temperature A. α2-macroglobulin, α1-antitrypsin
D. Maintaining the blood sedimentation system B. Immunoglobulin
E. All answers are correct C. Cryoglobulin, interferon
D. Ceruloplasmin, transferrin
69. A 4 y.o. child with signs of durative E. Hemoplexin, haptoglobin
proteinic starvation was admitted to the hospital.
The signs were as follows: growth inhibition, 74. Wilson’s disease is a disorder of copper
anemia, edemata, mental deficiency. Choose a transport which leads to the accumulation of this
cause of edemata development: metal in brain and liver cells. It is associated
A. Reduced synthesis of albumins with a disturbance in the synthesis of the
B. Reduced synthesis of globulins following protein:
C. Reduced synthesis of hemoglobin A. Ceruloplasmin
D. Reduced synthesis of lipoproteins B. Metallothionein
E. Reduced synthesis of glycoproteins C. Transcobalamin
D. Haptoglobin
70. The prolonged action of a number of E. Siderophilin
antibiotics and sulfonamides is due to the fact
that they circulate in the blood for a long time in 75. Examination of a 27-year-old patient
combination with: revealed pathological changes in liver and brain.
A. Albumin Blood plasma analysis revealed an abrupt
B. Transferrin decrease in the copper concentration, urine
C. Hemoglobin analysis revealed an increased copper
D. Haptoglobin concentration. The patient was diagnosed with
E. Hemopexin Wilson’s degeneration. To confirm the
diagnosis it is necessary to study the activity of
71. Electrophoretic study of a bloodserum the following enzyme in blood serum:
sample, taken from the patient with pneumonia, A. Ceruloplasmin
revealed an increase in one of the protein B. Carbonic anhydrase
fractions. Specify this fraction: C. Xanthine oxidase

36
hange E hange E
XC di XC di
F- t F- t
PD

PD
or

or
!

!
W

W
O

O
N

N
Y

Y
U

U
B

B
to

to
ww

ww
om

om
k

k
lic

lic
C

C
.c D. Leucine aminopeptidase B. Prothrombin

.c
w

w
tr re tr re
.

.
ac ac
k e r- s o ft w a k e r- s o ft w a

E. Alcohol dehydrogenase C. Fibrinogen


D. G immunoglobulin
76. Biochemical analysis of the blood serum of E. A immunoglobulin
a patient with hepatolenticular degeneration
(Wilson-Konovalov disease) revealed a 81. A 60-year-old man complains of joint pain.
decrease in the content of ceruloplasmin. In this An increase in the concentration of C-reactive
patient, the concentration of such ions in the protein and hydroxyproline was detected in the
serum will be increased: patient's serum. What disease are these
A. Copper symptoms characteristic of?
B. Calcium A. Rheumatism
C. Phosphorus B. Gout
D. Potassium C. Hepatitis
E. Sodium D. Jaundice
E. Diabetes
77. A patient suffering from hepatocerebral
degeneration has low concentration of 82. A 6-month-old child experienced frequent
ceruloplasmin in blood serum. What element and intense subcutaneous hemorrhages. The
accumulation will be observed in liver, appointment of a synthetic analogue of vitamin
cerebrum and kidneys of the patient? K (vikasol) gave a positive effect. In the γ-
A. Cuprum carboxylation of glutamic acid, what protein of
B. Calcium the blood coagulation system is this vitamin
C. Sodium taking part in?
D. Potassium A. Prothrombin
E. Ferrum B. Fibrinogen
C. Hageman factor
78. A 33-year-old woman suffers from D. Antihemophilic globulin A
hepatocerebral dystrophy (Wilson's disease). In E. Rosenthal factor
the blood - low content of ceruloplasmin. In the
urine - sharply elevated amino acids. What 83. After implantation of a cardiac valve a
process has increased these changes? young man constantly takes indirect
A. Complexation of amino acids with copper anticoagulants. His state was complicated by
B. Urea synthesis hemorrhage. What substance content has
C. Transamination of amino acids decreased in blood?
D. Disintegration of tissue proteins A. Prothrombin
E. Gluconeogenesis B. Haptoglobin
C. Heparin
79. A 38 year old patient suffers from D. Creatin
rheumatism in its active phase. What laboratory E. Ceruloplasmin
characteristic of blood serumis of diagnostic
importance in case of this pathology? 84. The patient complains of frequent bleeding
A. C-reactive protein from the gums. A deficiency of coagulation
B. Uric acid factor II (prothrombin) was found in the blood.
C. Urea What phase of blood coagulation is impaired in
D. Creatinine a person, above all?
E. Transferrin A. Thrombin formation
B. The formation of prothrombinase
80. Blood plasma of a healthy man contains C. Formation of fibrin
several dozens of proteins. During an illness D. Fibrinolysis
new proteins can originate, namely the protein E. Clot retraction
of "acute phase". Select such protein from the
listed below:
A. C-reactive protein

37
hange E hange E
XC di XC di
F- t F- t
PD

PD
or

or
!

!
W

W
O

O
N

N
Y

Y
U

U
B

B
to

to
ww

ww
om

om
k

k
lic

lic
C

C
.c 85. Inflammatory processes cause synthesis of 90. The student used canned donor blood trato

.c
w

w
tr re e
ar
.

.
ac
k e r- s o ft w a cke
r- s o ft w
protein of acute phase in an organism. What determine the time it was collected. However,
substances stumulate their synthesis? he could not get any positive result. The reason
A. Interleukin-1 for this is the lack of blood:
B. Immunoglobulins A. Ionized calcium
C. Interferons B. Hageman factor
D. Biogenic amins C. Thromboplastin
E. Angiotensin D. Fibrinogen
E. Vitamin K.
86. Pyrogenal administered to a rabbit, in the
course of an experiment, resulted in increase of 91. A 16 year old boy after an illness has
its body temperature. What substance of those diminished function of protein synthesis in liver
named below acts as a secondary pyrogen that is as a result of vitamin K deficiency. It will cause
a part of fever inducing mechanism? disturbance of:
A. Interleukin 1 A. Blood coagulation
B. Pseudomonas polysaccharide (Piromen) B. Erythrocyte sedimentation rate
C. Histamine C. Anticoagulant generation
D. Bradykinin D. Erythropoietin secretion
E. Immunoglobulin E. Osmotic blood pressure

87. A patient who suffers from pneumonia has 92. The patient has hemorrhages, the
high body temperature. What biologically active concentration of prothrombin is reduced in the
substance plays the leading part in origin of this blood. What vitamin deficiency led to a
phenomenon? violation of the synthesis of this clotting factor?
A. Interleukin-I A. K
B. Histamine B. A
C. Bradykinin C. D
D. Serotonin D. C
E. Leukotrienes E. E

88. After transfusion of 200 ml of blood apatient 93. A 37-year-old patient, with long-term use of
presented with body temperature rise up to antibiotics, has increased bleeding with minor
37,9oC. Which of the following substances is injuries. In the blood - a decrease in activity II,
the most likely cause of temperature rise? VII, X blood clotting factors; lengthening of
A. Interleukin-1 blood clotting time. What vitamin deficiency
B. Interleukin-2 caused these changes?
C. Tumour necrosis factor A. Vitamin K
D. Interleukin-3 B. Vitamin A
E. Interleukin-4 C. Vitamin C
D. Vitamin D
89. For the development of febrile conditions, E. Vitamin E
an increase in the level of the “acute phase”
proteins of ceruloplasmin, fibrinogen, and C- 94. Plasmic factors of blood coagulationare
reactive protein is characteristic. Specify the exposed to post-translational modification with
possible mechanism of this phenomenon: the participation of vitamin K. It is necessary as
A. Stimulating effect of IL-1 on hepatocytes a cofactor in the enzyme system of γ-
B. The destructive effect of temperature on the carboxylation of protein factors of blood
cells of the body coagulation due to the increased affinity of their
C. Proliferative effect of IL-2 on T-lymphocytes molecules with calcium ions. What amino acid
D. Degranulation of tissue basophils is carboxylated in these proteins?
E. - A. Glutamic
B. Valine
C. Serine

38
hange E hange E
XC di XC di
F- t F- t
PD

PD
or

or
!

!
W

W
O

O
N

N
Y

Y
U

U
B

B
to

to
ww

ww
om

om
k

k
lic

lic
C

C
.c D. Phenylalanine likely to have a disorder of the following

.c
w

w
tr re tr re
.

.
ac ac
k e r- s o ft w a k e r- s o ft w a

E. Arginine biochemical process:


A. Production of gammacarboxyglutamate
95. As a result of posttranslative modifications B. Conversion of homocysteine to methionine
some proteins taking part in blood coagulation, C. Conversion of methylmalonyl CoA to
particularly prothrombin, become capable of succinyl CoA
calcium binding. The following vitamin takes D. Degradation of glutathione
part in this process: E. Hydroxylation of proline
A. K
B. C 100. Activation of a number of hemostatic
C. A factors occurs through their joining with
D. B1 calcium ions. What structural component allows
E. B2 for adjoining of calcium ions?
A. Gamma-carboxyglutamic acid
96. To prevent postoperative bleeding a 6 y.o. B. Gamma-aminobutyric acid
child was administered vicasol that is a C. Gamma-oxybutyric acid
synthetic analogue of vitamin K. Name post- D. Hydroxyproline
translational changes of blood coagulation E. Monoamine-dicarboxylic acids
factors that will be activated by vicasol:
A. Carboxylation of glutamin acid 101. A 46-year-old female patient hasa
B. Phosphorylation of serine radicals continuous history of progressive muscular
C. Partial proteolysis (Duchenne’s) dystrophy. Which blood enzyme
D. Polymerization changes will be of diagnostic value in this case?
E. Glycosylation A. Creatine phosphokinase
B. Lactate dehydrogenase
97. A few days before an operation apatient C. Pyruvate dehydrogenase
should be administered vitamin K or its D. Glutamate dehydrogenase
synthetic analogue Vicasol. Vitamin K takes E. Adenylate cyclase
part in the following posttranslational
modification of the II, VII, IX, X blood clotting 102. A 15-year-old boy has been diagnosedwith
factors: acute viral hepatitis. What blood values hould
A. Carboxylation be determined to confirm acute affection of
B. Decarboxylation hepatic cells?
C. Deamination A. Aminotransferase activity (AST, ALT)
D. Transamination B. Unconjugated and conjugated bilirubin
E. Glycosylation content
C. Erythrocytes sedimentation rate (ESR)
98. A patient, who has been suffering for a long D. Cholesterol content
time from intestine disbacteriosis, has increased E. Protein fraction content
hemorrhaging caused by disruption of
posttranslational modification of blood- 103. 12 hours after an accute attack of
coagulation factors II, VII, IХ, and Х in the retrosternal pain a patient presented a jump of
liver. What vitamin deficiency is the cause of aspartate aminotransferase activity in blood
this condition? serum. What pathology is this deviation typical
A. К for?
B. 12 A. Myocardium infarction
C. 9 B. Viral hepatitis
D. С C. Collagenosis
E. Р D. Diabetes mellitus
E. Diabetes insipidus
99. A newborn baby has numerous
hemorrhages. Blood coagulation tests reveal 104. A 43-year-old man after eating fatty foods
increased prothrombin time. The child is most and alcohol complains of severe abdominal

39
hange E hange E
XC di XC di
F- t F- t
PD

PD
or

or
!

!
W

W
O

O
N

N
Y

Y
U

U
B

B
to

to
ww

ww
om

om
k

k
lic

lic
C

C
.c pain. The serum content of trypsin is 850 mmol D. Lungs

.c
w

w
tr re tr re
.

.
ac ac
k e r- s o ft w a k e r- s o ft w a

/ (h • l) (normal 60-240 mmol / (h • l)). What is E. Spleen


the most characteristic pathology of the
digestive system? 109. In the serum of the patient increased
A. Acute pancreatitis activity of hyaluronidase. Determining what
B. Dynamic intestinal obstruction biochemical index of serum will allow to
C. Mechanical intestinal obstruction confirm the assumption of connective tissue
D. Gastric ulcer pathology?
E. Hepatitis A. Sialic acids
B. Bilirubin
105. For biochemical diagnostics of myocardial C. Uric acid
infarction it is necessary to measure activity of a D. Glucose
number of enzymes and their isoenzymes. What E. Galactose
enzymatic test is considered to be the best to
prove or disprove the diagnosis of infarction in 110. Marked increase of activity of МВ forms
the early period after the chest pain is detected? of CPK (creatinephosphokinase) and LDH-1
A. Creatine kinase isoenzyme CK-MB was revealed by examination of the patient’s
B. Creatine kinase isoenzyme CK-MM blood. What is the most probable pathology?
C. LDH1 lactate dehydrogenaseisoenzyme A. Miocardial infarction
D. LDH2 lactate dehydrogenaseisoenzyme B. Hepatitis
E. Aspartate aminotransferasecytoplasmic C. Rheumatism
isoenzyme D. Pancreatitis
E. Cholecystitis
106. An increase in the activity of LDH4,5,
ALT, carbamoylornitene transferase was 111. An increase in the activity of LDH 1, LDH
detected in the patient's blood. In which body 2, AsAT, and creatine kinase was found in the
can you predict the development of the patient's blood. In which organ of the patient is
pathological process? the most likely development of the pathological
A. Liver (possible hepatitis) process?
B. Heart muscle (possible myocardial A. Heart
infarction) B. Pancreas
C. Skeletal muscle C. Liver
D. Kidneys D. Kidneys
E. Connective tissue E. Skeletal muscle

107. Albumin concentration in the patient’s 112. There is increased activity of AST, LDH1,
blood is 2.8 g / l, increased concentration of LDH2, and CPK in the patient’s blood.
lactate dehydrogenase 5 (LDH 5). What kind of Pathological process most likely occurs in the:
organ disease does this indicate? A. Heart
A. Liver B. Skeletal muscles
B. Kidney C. Kidneys
C. Heart D. Liver
D. Lung E. Adrenal glands
E. Spleen
113. The activity of the isoenzymes LDG1 and
108. Blood test of the patient revealed albumine LDG2 appeared in the patient's blood plasma.
content of 20 g/l and increased activity of On the pathology of a body that indicates?
lactate dehydrogenase isoenzyme 5 (LDH5). A. Myocardium
These results indicate disorder of the following B. Liver
organ: C. Kidney
A. Liver D. Brain
B. Kidneys E. Skeletal muscle
C. Heart

40
hange E hange E
XC di XC di
F- t F- t
PD

PD
or

or
!

!
W

W
O

O
N

N
Y

Y
U

U
B

B
to

to
ww

ww
om

om
k

k
lic

lic
C

C
.c 114. A patient presents high activity of LDH1,2, B. Skeletal muscle dystrophy

.c
w

w
tr re tr re
.

.
ac ac
k e r- s o ft w a k e r- s o ft w a

aspartate aminotransferase, creatine C. Diabetes mellitus


phosphokinase. In what organ (organs) is the D. Viral hepatitis
development of apathological process the most E. Acute pancreatitis
probable?
A. In the heart muscle (initial stage of 119. A 60-year-old man consulted a doctor
myocardium infarction) about an onset of chest pain. In blood serum
B. In skeletal muscles (dystrophy, atrophy) analysis showed a significant increase in the
C. In kidneys and adrenals activity of the following enzymes: creatine
D. In connective tissue kinase and its MB-isoform, aspartate
E. In liver and kidneys aminotransferase. These changes indicate the
development of the pathological process in the
115. A 49-year-old driver complains about following tissues:
unbearable constricting pain behind the A. Cardiac muscle
breastbone irradiating to the neck. The pain B. Lungs
arose 2 hours ago. Objectively: the patient’s C. Skeletal muscles
condition is grave, he is pale, heart tones are D. Liver
decreased. Laboratory studies revealed high E. Smooth muscles
activity of creatinekinase and LDH1. What
disease are these symptoms typical for? 120. Hereditary hyperlipoproteinemia type I is
A. Acute myocardial infarction caused by lipoprotein lipase deficiency.
B. Acute pancreatitis Increasing the level of some transport forms of
C. Stenocardia lipids in plasma even on an empty stomach is
D. Cholelithiasis characteristic?
E. Diabetes mellitus A. Chylomicrons
B. Low density lipoproteins
116. 6 hours after the myocardial infarction a C. Very low density lipoproteins
patient was found to have elevated level of D. High density lipoproteins
lactate dehydrogenase in blood. What E. Modified lipoproteins
isoenzyme should be expected in this case?
A. LDH1 121. Cholesterol content in blood serum of a 12-
B. LDH2 year-old boy is 25 mmol/l. Anamnesis states
C. LDH3 hereditary familial hypercholesterolemia caused
D. LDH4 by synthesis disruption of receptor-related
E. LDH5 proteins for:
A. Low-density lipoproteins
117. A 47-year-old male patient was diagnosed B. High-density lipoproteins
with myocardial infarction in the intensive care C. Chylomicrons
unit. Which of the lactate dehydrogenase (LDH) D. Very low-density lipoproteins
fractions will prevail in the blood serum during E. Middle-density lipoproteins
the first two days? М-а-2016-176
A. LDH 1
B. LDH 2 122. Examination of the patient revealed an
C. LDG 3 increase in the content of low-density
D. LDH 4 lipoproteins in the blood serum. What disease
E. LDG 5 can be assumed in this patient?
A. Atherosclerosis
118. The high level of Lactate Dehydrogenase B. Kidney damage
(LDH) isozymes concentration showed the C. Acute pancreatitis
increase of LDH-1 and LDH-2 in a patient’s D. Gastritis
blood plasma. Point out the most probable E. Inflammation of the lungs
diagnosis:
A. Myocardial infarction

41
hange E hange E
XC di XC di
F- t F- t
PD

PD
or

or
!

!
W

W
O

O
N

N
Y

Y
U

U
B

B
to

to
ww

ww
om

om
k

k
lic

lic
C

C
.c 123. A 58-year-old patient suffers from the

.c
w

w
tr re tr re
.

.
ac ac
k e r- s o ft w a k e r- s o ft w a

cerebral atherosclerosis. Examination revealed 128. During examination of a teenager with


hyperlipoidemia. What class of lipoproteins will xanthomatosis the family history of
most probably show increase in concentration in hypercholesterolemia is revealed. What
this patient’s blood serum? transportable lipids are increased in
A. Low-density lipoproteins concentration in case of such a disease?
B. High-density lipoproteins A. Low-density lipoproteins
C. Fatty acid complexes with albumins B. Chylomicrons
D. Chylomicrons C. Very low-density lipoproteins
E. Cholesterol D. High-density lipoproteins
E. Intermediate-density lipoproteins
124. Rabbits lived on food with addition of
cholesterol. Five months later the 129. The level of which plasma protein allows
atherosclerotic aorta changes were revealed. retrospectively (for the previous 4-8 weeks
Name the main cause of atherogenesis in this before the examination) to assess the level of
case: glycemia, if the patient has diabetes mellitus,
A. Exogenous hypercholesterolemia which is accompanied by fasting hyperglycemia
B. Overeating more than 7.2 mmol / l?
C. Hypodynamia A. Glycosylated hemoglobin.
D. Endogenous hypercholesterolemia B. Albumin.
E. – C. C-reactive protein.
D. Ceruloplasmin.
125. Examination of an ill child’s blood E. Fibrinogen.
revealed inherited hyperlipoproteinemia.
Genetic defect of what enzyme synthesis causes 130. A patient is diagnosed with pancreatic
this phenomenon? diabetes with associated hyperglycemia.
A. Lipoprotein lipase Glycemia rate can be assessed retrospectively
B. Glycosidase (4-8 weeks prior to examination) by measuring
C. Proteinase concentration of the following blood plasma
D. Hemsynthetase protein:
E. Phenylalanine hydroxylase A. Glycated hemoglobin
B. Albumin
126. In the study of the patient's blood plasma 4 C. Fibrinogen
hours after ingestion of fatty foods, it was found D. C-reactive protein
that it is cloudy. The most likely cause of this E. Ceruloplasmin
condition is an increase in plasma
concentration: 131. A patient is ill with diabetes mellitus that is
A. Chylomicrons accompanied by hyperglycemia of over 7,2
B. HDL millimole/l on an empty stomach. The level of
C. LDL what blood plasma protein allows to estimate
D. Cholesterol the glycemia rate retrospectively (4-8 weeks
E. Phospholipids before examination)?
A. Glycated hemoglobin
127. Blood serum of the patient has milky B. Albumin
appearance. Biochemical analysis revealed high C. Fibrinogen
content of triacylglycerols and chylomicrons. D. C-reactive protein
This condition is caused by hereditary defect of E. Ceruloplasmin
the following enzyme:
A. Lipoprotein lipase 132. A 42-year-old woman, who has been
B. Phospholipase keeping to a vegetarian diet for a long period of
C. Pancreatic lipase time, consulted a doctor. Examination revealed
D. Adipose tissue hormone-sensitive lipase negative nitrogen balance in the patient. What
E. Phosphodiesterase

42
hange E hange E
XC di XC di
F- t F- t
PD

PD
or

or
!

!
W

W
O

O
N

N
Y

Y
U

U
B

B
to

to
ww

ww
om

om
k

k
lic

lic
C

C
.c factor is the most likely cause of such a D. Nitrogen balance unchanged

.c
w

w
tr re tr re
.

.
ac ac
k e r- s o ft w a k e r- s o ft w a

condition? E. Ketonemia
A. Insufficient amount of proteins in the diet
B. Insufficient amount of dietary fiber 138. A patient with primary nephrotic syndrome
C. Excessive amount of fats in the diet has the following content of whole protein: 40
D. Insufficient amount of fats in the diet g/l. What factor caused hypoproteinemia?
E. Decreased rate of metabolic processses A. Proteinuria
B. Transition of protein from vessels to tissues
133. Examination of a 45-year-old man whohad C. Reduced protein synthesis in liver
kept to a vegeterian diet for a long time revealed D. Increased proteolysis
negative nitrogen balance.Which peculiarity of E. Disturbance of intestinal protein absorption
his diet is the causeof this phenomenon?
A. Lack of proteins 139. Examination of a patient with chronicrenal
B. Lack of fats insufficiency revealed an increase in residual
C. Excess of water nitrogen concentration in blood up to 35
D. Excess of carbohydrates millimole/l, more than half of which is urea.
E. Lack of vitamins What type of hyperazotemia is it?
A. Retentional
134. A 14 year old child was found to have a B. Hepatic
positive nitrogen balance. Which of the C. Productional
following could be the reason for this? D. Residual
A. Body growth E. Combined
B. Fasting
C. Reduced protein in food 140. On the basis of laboratory analysis, the
D. Significant physical loads patient confirmed the diagnosis of gout. To
E. Emotional stress establish the diagnosis was carried out
determination of the content:
135. A month after a serious operation a 38- A. Urinary acid in the blood and urine
year-old patient has recovered and has now B. Creatinine in urine
positive nitrogen balance. Urine of this patient C. Residual nitrogen in the blood
may be found to have low concentration of the D. Urea in the blood and urine
following nitrogencontaining substance: E. Ammonia in the urine
A. Urea
B. Lactate 141. A young man of 18 years old was
C. Stercobilinogen diagnosed with muscular dystrophy. Is the
D. Galactose increase in the serum content of the substance
E. 17-ketosteroids most likely in this pathology?
A. Creatine
136. A chiled was diagnosed with acuterenal B. Myoglobin
failure. What biochemic saliva indices can C. Myosin
confirm this diagnosis? D. Lactate
A. Increased level of rest nitrogen E. Alanin
B. Increase of immunoglobuline A
C. Reduction of alkaline phosphatase 142. A 46-year-old patient complains of dry
D. Increase of alpha amylase mouth, thirst, frequent urination, and general
E. Decreased level of phosphate weakness. In the blood: hyperglycemia,
hyperketonemia. In the urine: glucose, ketone
137. A patient who goes out of a state of bodies. On ECG: diffuse changes in the
prolonged fasting has determined nitrogen myocardium. What is the most likely diagnosis?
metabolism. What result can you expect? A. Diabetes
A. Reducing nitrogen excretion B. Alimentary hyperglycemia
B. Increased nitrogen excretion C. Acute pancreatitis
C. Nitrogen equilibrium D. Diabetes mellitus

43
hange E hange E
XC di XC di
F- t F- t
PD

PD
or

or
!

!
W

W
O

O
N

N
Y

Y
U

U
B

B
to

to
ww

ww
om

om
k

k
lic

lic
C

C
.c E. Coronary heart disease 147. A 48 year old patient complained about

.c
w

w
tr re tr re
.

.
ac ac
k e r- s o ft w a k e r- s o ft w a

intense pain, slight swelling and reddening of


143. The patient after suffering parotitis lost skin over the joints, temperature rise up to
weight, constantly feeling thirsty, drinks a lot of 38oC. Bloodanalysis revealed high
water, notes frequent urination, increased concentration ofurates. This condition might be
appetite, pruritus, weakness, furunculosis. In the caused by disturbed metabolism of:
blood: glucose - 16 mmol / l, ketone bodies 100 A. Purines
mkmol / l. What disease has the patient B. Collagen
developed? C. Cholesterol
A. Insulin-dependent diabetes mellitus D. Pyrimidines
B. Insulin independent diabetes E. Carbohydrates
C. Steroid diabetes
D. Diabetes mellitus 148. A patient has increased content of uric acid
E. Diabetes mellitus malnutrition in his blood that is clinically presented by pain
syndrome as a result of urate deposition in the
144. A patient was delivered to the hospital by joints. What process does this acid result from?
an emergency team. Objectively: grave A. Lysis of purine nucleotides
condition, unconscious, adynamy. Cutaneous B. Lysis of pyrimidine nucleotides
surfaces are dry, eyes are sunken, face is C. Heme catabolism
cyanotic. There is tachycardia and smell of D. Proteolysis
acetone from the mouth. Analysis results: blood E. Reutilization of purine bases
glucose -20,1 micromole/l (standard is 3,3-5,5
micromole/l), urine glucose - 3,5% (standardis - 149. A 1,7-year-old child with a developmental
0). What is the most probable diagnosis? delay and manifestations of self-agression has
A. Hyperglycemic coma the concentration of uric acid in blood at the rate
B. Hypoglycemic coma of 1,96 millimole/l. What metabolic disoder is
C. Acute heart failure this typical for?
D. Acute alcoholic intoxication A. Lesch-Nyhan syndrome
E. Anaphylactic shock B. Podagra
C. Acquired immunodeficiency syndrome
145. A 38-year-old patient was admitted to the D. Gierke’s disease
intensive care unit in an unconscious state. E. Cushing’s basophilism
Reflexes are absent. Blood sugar - 2.1 mmol / l.
In history - diabetes mellitus from 18 years. 150. M-r S presents all signs of the hepatic
What kind of coma does the patient have? coma: loss of consciousness, absence of
A. Hypoglycemic reflexes, cramps, convulsion, disorder of heart
B. Ketoacidotic activity, recurrent (periodical) respiration. What
C. Lacticidemic are cerebrotoxical substances which accumulate
D. Hyperosmolar in blood under hepar insufficiency?
E. Hyperglycemic A. Ammonia
B. IL-1
146. A patient with a diagnosis of Itsenko- C. Autoantibody
Cushing's disease (hyperproduction of adrenal D. Necrosogenic substances
hormones) in the blood has an increased E. Ketonic body
concentration of glucose, ketone bodies,
sodium. What is the biochemical mechanism 151. After a serious viral infection a 3-year-old
leading to the occurrence of hyperglycemia? child has repeated vomiting, loss of
A. Gluconeogenesis consciousness, convulsions. Examination
B. Glycogenesis revealed hyperammoniemia. What may have
C. Glycogenolysis caused changes of biochemical blood indices of
D. Glycolysis this child?
E. Aerobic glycolysis A. Disorder of ammonia neutralization in
ornithinic cycle

44
hange E hange E
XC di XC di
F- t F- t
PD

PD
or

or
!

!
W

W
O

O
N

N
Y

Y
U

U
B

B
to

to
ww

ww
om

om
k

k
lic

lic
C

C
.c B. Activated processes of aminoacids E. Pancreas

.c
w

w
tr re tr re
.

.
ac ac
k e r- s o ft w a k e r- s o ft w a

decarboxylation
C. Disorder of biogenic amines neutralization 156. Laboratory examination of a child revealed
D. Increased purtefaction of proteins in increased concentration of leucine, valine,
intestines isoleucine and their ketoderivatives in blood and
E. Inhibited activity of transamination enzymes urine. Urine smelt of maple syrup. This disease
is characterized by the deficit of the following
152. A 2-year-old child presents with mental enzyme:
development retardation, intolerance of A. Dehydrogenase of branched amino acids
proteins, severe hyperammonemia against the B. Aminotransferase
background of low blood urea content. This C. Glucose-6-phosphatase
condition is caused by the congenital deficiency D. Phosphofructokinase
of the following mitochondrial enzyme: E. Phosphofructomutase
A. Carbamoyl phosphate synthetase
B. Citrate synthase 157. A sick child presents with high content of
C. Succinate dehydrogenase phenyl pyruvate in urine (normally it is
D. Malate dehydrogenase practically absent). Blood phenylalanine level is
E. Monoamine oxidase 350 mg/L (norm - 15 mg/L). What disease are
these symptoms characteristic of?
153. A newborn presents with weak suckling, A. Phenylketonuria
frequent vomiting, and hypotonia. Blood and B. Albinism
urine citrulline are very high. What metabolic C. Tyrosinosis
process is disturbed? D. Alkaptonuria
A. Ornithine cycle E. Gout
B. Tricarboxylic acid cycle
C. Glycolysis 158. A hospital admitted a 9 y.o. boy with
D. Gluconeogenesis mental and physical retardation. Biochemical
E. Cori cycle blood analysis revealed high content of
phenylalanine. Such condition may be caused
154. Nitrogen is being excreted from thebody by blocking of the following enzyme:
mainly as urea. When activity of acertain A. Phenylalanine-4-monooxigenase
enzyme in the liver is low, it resultsin inhibition B. Oxidase of homogentisic acid
of urea synthesis and nitrogenaccumulation in C. Glutamine transaminase
blood and tissues. Namethis enzyme: D. Aspartate aminotransferase
A. Carbamoyl phosphate synthetase E. Glutamate decarboxylase
B. Aspartate aminotransferase
C. Urease 159. Blood of the patients with diabetes mellitus
D. Amylase shows increased content of free fatty acids.
E. Pepsin Name the most likely cause of this:
A. Increased activity of adipose triglyceride
155. The patient has a reduced content of lipase
indican in the blood serum, and also his daily B. Accumulation of palmitoyl-CoA in cytosol
excretion with urine is reduced. Impairment of C. Activation of ketone bodies utilization
which organ is the cause of this? D. Activation of apoА1, apoА2, and apoА4
A. Liver apolipoprotein synthesis
B. Kidney E. Decreased activity of plasma
C. Heart phosphatidylcholine-cholesterolacyltransferase
D. Lungs

45
hange E hange E
XC di XC di
F- t F- t
PD

PD
or

or
!

!
W

W
O

O
N

N
Y

Y
U

U
B

B
to

to
ww

ww
om

om
k

k
lic

lic
C

C
.c

.c
Biochemistry of immune system
w

w
tr re tr re
.

.
ac ac
k e r- s o ft w a k e r- s o ft w a

1. Differentiation of B-lymphocytes into plasma 5. Bone marrow has been transplanted to the
cells leads to synthesis of immunoglobulins that liquidator of Chernobyl atomic power station
ensure specific immune response of the body. accident which was irradiated. The reaction “a
Differentiation of B-lymphocytes takes place in transplant against a host” development was
the following organ of immune system: diagnosed at the patient after operation. Which
A. Tonsils antigens are the reason of this reaction?
B. Red bone marrow A. Antigens of HLA system in the cells of
C. Liver liquidator’s organism
D. Thymus B. Antigens of Rh system in the erythrocytes of
E. Thyroid gland liquidator
C. Antigens HBs, HBc, HBe
2. Among lymphocytes, there are populations of D. Antigens of ABO system in the erythrocytes
cells with membrane receptors for IgM, they are of liquidator
activated by the action of specific antigens, E. Antigens of HLA system in the cells of
mitotically multiply, and differentiate into donor’s spinal cord
plasma cells that produce antibodies
(immunoglobulins). What are these cells called? 6. A male patient has been diagnosed with acute
A. B-lymphocytes poststreptococcal glomerulonephritis. It is most
B. Memory T lymphocytes likely that the lesion of the basement membrane
C. T-killer lymphocytes of renal corpuscles was caused by the following
D. T-suppressor lymphocytes allergic reaction:
E. - A. Immune complex
B. Anaphylactic
3. A 2-year-old boy often became ill with C. Cytotoxic
respiratory diseases, stomatitis, pustular skin D. Delayed
lesions. Even small damage to the gums and E. Stimulating
mucous membranes is complicated by
prolonged inflammation. It is established that 7. A 5-year-old child is diagnosed with Bruton
the blood of the child is practically absent syndrome (X-linked agammaglobulinemia) that
immunoglobulins of all classes. Reduction manifests itself in severe clinical course of
the functional activity of a cell population is the bacterial infections and absence of B
basis of the syndrome described? lymphocytes and plasma cells. What changes of
A. B-lymphocytes immunoglobulin content can be observed in
B. T lymphocytes blood serum of the child with
C. Neutrophils immunodeficiency?
D. Macrophages A. Decreased IgA, IgM
E. NK lymphocytes B. Increased IgA, IgM
C. Decreased IgD, IgE
4. A 13-year-old boy presents with eczematous D. Increased IgD, IgE
rashes on his shins and torso. Anamnesis states E. No changes
cases of otitis, pneumonia, and furuncles in the
patient. Blood test: platelets - 70 • 109/l, low 8. Examination of a child who frequently suffers
activity of T helper and T suppressor cells, low from infectious diseases revealed that IgG
IgM, with normal IgA and IgG. What concentration in blood serum was 10 times less
immunodeficient disease does this boy have? than normal, IgA and IgM concentration was
A.Wiskott-Aldrich syndrome also significantly reduced. Analysis showed also
B. Louis-Bar syndrome (Ataxiatelangiectasia) lack of B-lymphocytes and plasmocytes. What
C. Severe combined immunodeficiency (Swiss disease are these symptoms typical for?
type) A. Bruton’s disease
D. DiGeorge syndrome B. Swiss-type agammaglobulinemia
E. Chediak-Higashi syndrome C. Dysimmunoglobulinemia
D. Louis-Bar syndrome
46
hange E hange E
XC di XC di
F- t F- t
PD

PD
or

or
!

!
W

W
O

O
N

N
Y

Y
U

U
B

B
to

to
ww

ww
om

om
k

k
lic

lic
C

C
.c E. Di George syndrome B. Combined Immunodeficiency Syndrome

.c
w

w
tr re tr re
.

.
ac ac
k e r- s o ft w a k e r- s o ft w a

C. Secondary immunodeficiency syndrome


9. Parents of 5-year-old child report him o have D. Acute lymphocytic leukemia
frequent colds that develop into pneumonias, E. Syndrome of chronic intoxication
presence of purulent rashes on the skin.
Laboratory tests have revealed the following: 13. During the examination of the patient a
absence of immunoglobulins of any type, and insufficient quantity of immunoglobulins was
naked cells are absent from the lymph nodes detected. What kind of immune cells do they
punctate. What kind of immune disorder is it? produce?
A. X-linked hypogammaglobulinemia A. Plasma cells
(Bruton type agammaglobulinemia) B. T-helper
B. Autosomal recessive agammaglobulinaemia C. T-killers
(Swiss type) D. T-suppressors
C. Hypoplastic anemia E. Plasmoblasty
D. Agranulocytosis
E. Louis-Barr syndrome 14. Following exposure to radiation a lot of
mutant cells appeared in a patient. Some time
10. A child with suspected tuberculosis was later most of them were detected and destroyed
given Mantoux test. After 24 hours the site of by the following cells of the immune system:
the allergen injection got swollen, hyperemic A. T-lymphocytes-killers
and painful.What are the main components that B. Plasmoblasts
determine such response of the body? C. T-lymphocytes-supressors
A. Mononuclear cells, T-lymphocytes and D. B-lymphocyte
lymphokines E. Stem cells
B. Granulocytes, T-lymphocytes and IgG
C. Plasma cells, T-lymphocytes and 15. Numerous plasma cells were found in the
lymphokines blood girl 16 years old, suffering from an
D. B-lymphocytes, IgM autoimmune inflammation of the thyroid gland.
E. Macrophages, B-lymphocytes and monocytes With the proliferation and differentiation of
what blood cells cause increase of plasmocyte?
11. A 27-year-old woman has dropped penicillin A B-lymphocytes
containing eye drops. In a few minutes there B T-helper
appeared feeling of itching, burning of the skin, C Mast cells
lips and eyelids D T-killer
edema, whistling cough, decrease of BP. What E T-suppressor
imunoglobulins take part in the development of
this allergic reaction? 16. The patient is diagnosed with ARVI. Class
A. IgE and IgG M immunoglobulins have been found in the
B. IgM and IgG serum. What is the period of the infection
C. IgA and IgM process in this case?
D. IgM and IgD A. Acute
E. IgG and IgD B. Prodromally
C. Incubation
12. A 3 year old child with multiple impaired D. Reconvalescence
development of the bones of the facial skull E. Micro carrier
died. The cause of death is sepsis, which
developed on the background of 17. A 34-year-old patient, after suffering an
bronchopneumonia. Blood content intestinal infection caused by Salmonella,
immunoglobulins within the physiological symptoms of the disease began to fade. What
norm. At the autopsy revealed the absence of class of immunoglobulins will be detected in the
the thymus. Name the main cause of the child’s patient's blood during the recovery period?
illness: A. Ig G
A. Syndrome of cellular immunity deficiency B. Ig A

47
hange E hange E
XC di XC di
F- t F- t
PD

PD
or

or
!

!
W

W
O

O
N

N
Y

Y
U

U
B

B
to

to
ww

ww
om

om
k

k
lic

lic
C

C
.c C. Ig D hypotension. When this reaction is developed

.c
w

w
tr re tr re
.

.
ac ac
k e r- s o ft w a k e r- s o ft w a

D. Ig E and the allergen achieves tissue basophils, it


E. Ig M reacts with:
A. IgE
18. Preventive vaccination against polyomyelitis B. IgА
is made with inactivated vaccine introduced C. IgD
parenterally. What immunoglobulins create the D. IgМ
postvaccinal immunity in the case? E. T-lymphocytes
A. IgM, IgG
B. IgG, secretory IgA 23. A 34-year-old patient, after suffering an
C. IgM, secretory IgA intestinal infection caused by Salmonella,
D. Serum IgA, IgM symptoms of the disease began to fade. What
E. Ig E, IgM class of immunoglobulins will be detected in the
patient's blood during the recovery period?
19. A 37-year-old man was injected with A. Ig G
novocaine solution in the treatment of acute B. Ig A
pulpitis. A few minutes later the patient C. Ig D
developed an anaphylactic shock.What D. Ig E
immunoglobulin does the antigen mainly E. Ig M
interact with in the body during a given allergic
reaction? 24. One of the functions of saliva is protective,
A. IgE which is realized by the formation of local
B. IgM immunity of the mucous membrane due to the
C. IgA secretion of the parotid glands of such a protein:
D. IgD A. Secretory Immunoglobulin A
E. IgG B. Collagen
C. Elastin
20. After the introduction of lidocaine, a 25- D. Fibrinogen
year-old patient developed shortness of breath, E. Albumin
bronchospasm, and his blood pressure dropped
sharply, which required the use of immediate 25. Various cells of the oral mucous membrane
relief from a dentist doctor. What mechanism is and antimicrobial substances synthesized by
the basis of such phenomena? these cells play an important part in the local
A. Allergic reactions involving IgE immunity of the oral cavity. Specify the key
B. Allergic cytotoxic reactions factors for the local immunity:
C. The phenomenon of idiosyncrasy A. Secretory IgA
D. Hypersensitivity due to T-lymphocytes B. B-lymphocytes
E. Effects caused by T-killers C. IgG
D. Macrophages
21. Skin samples of a patient with bronchial E. Eosinophils
asthma revealed allergen sensitization of poplar
fuzz. What factor of immune system plays the 26. In our country, routine preventive
main part in vaccinations against poliomyelitis ivolve using
development of this immunopathological state? live vaccine that is administered orally. What
A. IgE immunoglobulins are responsible for the
B. IgD development of local post-vaccination immunity
C. IgM in this case?
D. Sensitized Т-lymphocytes A. Secretory IgA
E. – B. IgM
C. IgG
22. A teenger had his tooth extracted under D. Serum IgA
novocain anaesthesia. 10 minutes later he E. IgE
presented with skin pallor, dyspnea,

48
hange E hange E
XC di XC di
F- t F- t
PD

PD
or

or
!

!
W

W
O

O
N

N
Y

Y
U

U
B

B
to

to
ww

ww
om

om
k

k
lic

lic
B. DNA → i-RNA → polypeptide → DNA
C

C
.c 27. Throughout a year a 37-year-old woman

.c
w

w
tr re tr re
.

.
ac ac
k e r- s o ft w a k e r- s o ft w a

periodically got infectious diseases of bacterial C. DNA → polypeptide → i-RNA


origin, their course was extremely lingering, D. i-RNA → polypeptide → DNA
remissions were short. Examination revealed E. Polypeptide → RNA → DNA → i-RNA
low level of major classes of immunoglobulins.
The direct cause of this phenomenon may be the 32. A patient with acquired immunodeficiency
following cell dysfunction: syndrome (СНІД) the immunological reactivity,
A. Plasmocytes manifested by the development of chronic
B. Phagocytes inflammatory processes, infectious diseases,
C. Neutrophils tumor growth, is significantly reduced. What
D. Macrophages types of blood cells damages HIV-infections,
E. Lymphocytes which results in reduced immune protection?
A. T4 helper
28. A patient has been hospitalized with B. Natural Kileri (NK)
provisional diagnosis of virus B hepatitis. C. T-suppressors
Serological reaction based on complementation D. T8-Effects
of antigen with antibody chemically bound to E. B-lymphocytes
peroxidase or alkaline phosphatase has been
used for disease diagnostics. What is the name 33. A 20-year-old patient, an AIDS diagnosis
of the applied serological reaction? was established. What cell populations are most
A. Immune-enzyme analysis sensitive to human immunodeficiency virus?
B. Radioimmunoassay technique A. T-helpers
C. Immunofluorescence test B. Hepatocytes
D. Bordet-Gengou test C. Endotheliocytes
E. Antigen-binding assay D. Epithelial cells
E. B lymphocytes
29. A doctor examined a patient with recurrent
aphthous stomatitis with concominant 34. A 1-year-old child often suffers from viral
candidosis and decided to eliminate a possibility and bacterial infections that are difficult to treat.
of HIV-infection. What examination can help to During the investigation the immunological
clear the situation up and make a provisional status revealed the absence of lymphocytes in
diagnosis? the blood providing cellular immunity. What
A. Immune-enzyme analysis immunodeficiency and what system is detected
B. Gel precipitation reaction in this child?
C. Reaction of hemagglutination inhibition A. In a T cell primary response
D. Reaction of hemagglutination B. In a T-cell secondary response
E. Phase-contrasr microscopy C. In a B cell primary response
D. In a macrophage system primary response
30. T-lymphocytes are determined to be affected E. In a microphages primary response
with HIV. In this case viral enzyme reverse
transcriptase (RNA-dependent DNA- 35. Blood serum of a newborn contains
polymerase) catalyzes the synthesis of: antibodies to measles virus. What kind of
A. DNA based on the viral RNA matrix immunity is this indicative of?
B. Viral RNA based on the DNA matrix A. Natural passive
C. Viral protein based on the viral RNA matrix B. Natural active
D. Viral DNA based on the DNA matrix C. Artificial passive
E. Informational RNA based on the viral protein D. Artificial active
matrix E. Heredoimmunity

31. Lymphocytes are affected by HIV retrovirus 36. A child was born with cleft palate.
(AIDS). In this case, the direction of Examination revealed aorta defects and reduced
information flow in the cell will be: number of T-lymphocytes in blood. What
A. RNA → DNA → i-RNA → Polypeptide immunodeficient syndrome is it?

49
hange E hange E
XC di XC di
F- t F- t
PD

PD
or

or
!

!
W

W
O

O
N

N
Y

Y
U

U
B

B
to

to
ww

ww
om

om
k

k
lic

lic
C

C
.c A. Di George A. Interleukin-1

.c
w

w
tr re tr re
.

.
ac ac
k e r- s o ft w a k e r- s o ft w a

B.Wiskott-Aldrich B. Immunoglobulins
C. Chediak-Higashi C. Interferons
D. Louis-Bar D. Biogenic amins
E. Swiss-type E. Angiotensin

37. A female patient underwent liver 42. Donor skin transplantation was performed to
transplantation. 1,5 month after it her condition a patient with extensive burns. On the 8-th day
became worse because of reaction of transplant the graft became swollen and changed colour;
rejection. What factor of on the 11-th day graft rejection started.What
immune system plays the leading part in this cells take part in this process?
reaction? A. T-lymphocytes
A. T-killers B. Erythrocytes
B. Interleukin-1 C. Basophils
C. Natural killers D. Eosinophils
D. B-lymphocytes E. B-lymphocytes
E. T-helpers
43. The patient contacted the dermatologist with
38. A patient consulted an immunologist about complaints about eczematous lesions of the skin
diarrhea, weight loss within several months, that appeared after contact with the detergent
low-grade fever, enlarged lymph nodes. The "Lotus". The use of rubber gloves prevents this.
doctor suspected HIV infection. What The pathological reaction of the skin is caused
immunocompetent cells must be studied in the by activation:
first place? A. T-lymphocytes
A. Helper T-lymphocytes B. B-lymphocytes
B. Suppressor T-lymphocytes C. Monocytes
C. B-lymphocytes D. Neutrophils
D. Monocytes E. Basohiles
E. Plasma cells
44. The development of febrile states are
39. After transfusion of 200 ml of blood a characterized by an increase in the level “the
patient presented with body temperature rise up acute phase” proteins. There are ceruloplasmin,
to 37,9°C. Which of the following substances is fibrinogen, C-reactive protein. Enter the
the most likely cause of temperature rise? Possible mechanism of this phenomenon:
A. Interleukin-1 A. Stimulating effect of IL-1 on hepatocytes
B. Interleukin-2 B. The destructive effect of temperature on the
C. Tumour necrosis factor cells of the body
D. Interleukin-3 C. The proliferative effect of IL-2 on
E. Interleukin-4 T lymphocytes
D. Degranulation of tissue basophils
40. A patient with skin mycosis has disorder of E. -
cellular immunity. The most typical
characteristic of it is reduction of the following 45. A pregnant woman with the previous
index: diagnosis of toxoplasmosis has been
A. T-lymphocytes hospitalized. What serological reaction should
B. Immunoglobulin G be used to determine toxoplasmosis in test
C. Immunoglobulin E specimens?
D. B-lymphocytes A. Complement fixation test.
E. Plasmocytes B. Neutralization
C. Haemadsorption
41. Inflammatory processes cause synthesis of D. Agglutination
protein of acute phase in an organism. What E. Inhibition of hemagglutination
substances simulate their synthesis?

50
hange E hange E
XC di XC di
F- t F- t
PD

PD
or

or
!

!
W

W
O

O
N

N
Y

Y
U

U
B

B
to

to
ww

ww
om

om
k

k
lic

lic
C

C
.c

.c
Biochemistry of liver
w

w
tr re tr re
.

.
ac ac
k e r- s o ft w a k e r- s o ft w a

1. Disorderthe splitting of the lipids in the small achilic syndrome due to obturation of the biliary
intestine is due to a disorder of lipase activity. tract. Which components of food will be
Which of the following factors activates the mastered the most?
lipase? A. Fats
A. Bile acids B.Carbohydrates
B. Hydrochloric acid C. Proteins
C. Enterokinase D. Nucleic acids
D. Pepsin E. Electrolytes
E. Solts of Na+
7. The drug "Geptral", which is used in liver
2. Examination of a patient revealed that dental diseases contains S-adenosylmethionine. This
hypoplasia was caused by hypovitaminosis of active amino acid is involved in the synthesis
vitamins A and D. These vitamins were of:
administered perorally but they didn’t have any A. Phospholipids
medicinal effect. What is the probable cause of B. Fatty acids
disturbed vitamin assimilation? C. Triacylglycerols
A. Bile acid deficiency D. Cholesterol
B. Phospholipase A2 deficiency E. Heme
C. Cholesterolesterase deficiency
D. Colipase deficiency 8. The patient appointed a lipotropic drug a
E. Pancreatic lipase deficiency donor of methyl groups, to prevent a fatty liver
distrophia. This is sensible:
3. A coprological study found that the feces are A. S-Adenosylmethionine
discolored, there are found drops of neutral fat. B. Cholesterol
The most likely cause of this is a violation: C. Bilirubin
A. Flow of bile into the intestine D. Valine
B. pH of gastric juice E. Glucose
C. Secretions of pancreatic juice
D. Secretion of intestinal juice 9. Examination of cell culture gotfrom a patient
E. Absorption processes in the intestine with lysosomal pathology revealed
accumulation of great quantity of lipids in the
4. After eating fatty foods, the patient has lysosomes. What of the following diseases is
nausea and heartburn, steatorrhea occurs. The this disturbance typical for?
reason for such a state can be: A. Tay-Sachs disease
A. Lack of bile B. Gout
B. Increased lipase secretion C. Phenylketonuria
C. Violation of trypsin synthesis D.Wilson disease
D. Lack of amylase E. Galactosemia
E. Impaired phospholipase synthesis
10. Steatosis occurs as a result of the
5. Due to the blockage of the common bile duct accumulation of triacylglycerols in hepatocytes.
(which was radiographically confirmed), the One of the mechanisms of development of this
biliary flow to the duodenum was stopped. We disease is reduction of utilization neutral fat
should expect the impairment of: LDL. What lipotropic substances prevent the
A. Fat emulsification development of steatosis? A. Methionine, BC,
B. Protein absorption B12
C. Carbohydrate hydrolysis B. Arginine, B2, B3
D. Secretion of hydrochloric acid C. Alanine, B1, PP
E. Salivation inhibition D. Valine, B3, B2
E. Isoleucine, B1, B2
6. A 65-year-old patient suffers from
cholelithiasis. Recently, there were signs of
51
hange E hange E
XC di XC di
F- t F- t
PD

PD
or

or
!

!
W

W
O

O
N

N
Y

Y
U

U
B

B
to

to
ww

ww
om

om
k

k
lic

lic
C

C
.c 11. In an experimental animal, receiving non- E. Leucine

.c
w

w
tr re tr re
.

.
ac ac
k e r- s o ft w a k e r- s o ft w a

protein diet, fatty infiltration of the liver has


developed due to the deficiency of the 16. The gluconeogenesis is activated in the liver
methylating agents. The formation of which after intensive physical trainings. What substance is
metabolite is disturbed in the experimental utilized in gluconeogenesis first of all in this case:
animal? A. Lactate
A. Choline B. Pyruvate
C. Glucose
B. DOPA D. Glutamate
C. Cholesterol E. Alanine
D. Acetoacetate
E. Linolic acid 17. In patients with glycogenosis, that is von
Gierke’s disease, the conversion of glucose-6-
12. A dry cleaner’s worker has been found to phosphate in to glucose is inhibited, which is
have hepatic steatosis. This pathology can be accompanied by the improper breakdown of
caused by the disruption of synthesis of the glycogen in the liver. The cause of this
following substance: condition is the following enzyme deficiency:
A. Phosphatidylcholine A. Glucose-6-phosphatase
B. Tristearin B. Glycogen phosphorylase
C. Urea C. Glucose-6-phosphate dehydrogenase
D. Phosphatidic acid D. Phosphofructokinase
E. Cholic acid E. Phosphoglucomutase
13. In the 2-year-old boy, an increase in the size 18. A child has a history of hepatomegaly,
of the liver and spleen, cataract is observed. The hypoglycemia, seizures, especially on an empty
concentration of sugar is elevated in the blood, stomach and in stressful situations. The child is
but the test of glucose tolerance is normal. An diagnosed with Gierke disease. This disease is
hereditary violation of the metabolism of which caused by the genetic defect of the following
substance is the cause of this condition? enzyme:
A. Galactose A. Glucose-6-phosphatase
B. Fructose B. Amyloid-1,6-glycosidase
C. Glucose C. Phosphoglucomutase
D. Maltose D. Glycogen phosphorylase
E. Saccharose E. Glucokinase
14. During starvation normal rate ofglucose is
maintained by means ofactivation of 19. A patient with chronic hypoglycemia had
gluconeogenesis. Whatsubstance can be used as adrenaline introduction. After introduction
a substrate forthis process? blood test hasn’t changed essentially. Doctor
A. Alanine assumed liver pathology. What liver function
B. Ammonia may have been changed?
C. Adenine A. Function of glycogen depositing
D. Urea B. Function of cholesterin production
E. Guanine C. Ketogenic function
D. Glycolytic function
15. In a patient undergoing a course of medical E. Excretory function
starvation, the normal level of glucose in the
blood is maintained mainly due to 20. After the introduction of adrenaline in a
gluconeogenesis. From what amino acids at the patient with persistent hypoglycemia, blood test
same time in the human liver most actively has not changed significantly. In such
synthesized glucose? circumstances, there is a possibility of
A. Alanine disturbances in the liver. What function of the
B. Lysine liver is changed?
C. Valine A. Glycogendeposing.
D. Glutamic acid B. Glycolytic.
52
hange E hange E
XC di XC di
F- t F- t
PD

PD
or

or
!

!
W

W
O

O
N

N
Y

Y
U

U
B

B
to

to
ww

ww
om

om
k

k
lic

lic
C

C
.c C. Excretory. B. Increase of globulin conten

.c
w

w
tr re tr re
.

.
ac ac
k e r- s o ft w a k e r- s o ft w a

D. Ketogenic. C. Decrease of fibrinogen conten


E. Cholesterol-forming. D. Increase of albumin conten
E. Decrease of globulin conten
21. In the human diet there are a large number
of carbohydrates. The number of which 26. A 25-year-old patient has been diagnosed
structures will increase in cytoplasm of with chronic hepatitis. The patient complains of
hepatocytes? 10 kg weight loss within 2 months. Objectively:
A. Glycogen granules the patient has dry peeling skin, pale with
B. Drops of fat yellow shade, petechial haemorrhages,
C. The lysosomes stomatorrhagia. Petechial haemorrhages and
D. Free ribosomes stomatorrhagia are caused by the disturbance of
E. Inclusion of lipofuscine the following hepatic function:
A. Protein synthesizing
22. The main part of nitrogen is taking out of B. Chromogenic
organism as element of urea. Reducing the C. Glycogen synthesizing
activity of what liver enzyme leads to inhibition D. Detoxication
of the synthesis of urea and increasing the E. Depositing
accumulation of ammonia in blood and tissues?
A. Carbamoyl phosphate synthase 27. Hepatitis has led to the development of
B. Aspartate aminotransferase hepatic failure. Mechanism of edemata
C. Urease formation is activated by the impairment of the
D. Amilaza following liver function:
E. Pepsin A. Protein-synthetic
B. Barrier
23. In the biosynthesis of urea in the liver, the C. Chologenetic
formation of ornithine and urea is stimulated. D. Antitoxic
What amino acid is the intermediate product of E. Glycogen-synthetic
this synthesis?
A. Аrginine 28. A patient being treated for viral hepatitis
B. Leicinc. type B got symptoms of hepatic insufficiency.
C. Сitrate. What blood changes indicative of protein
D. Valin. metabolism disorder will be observed in this
E. Tryptophan. case?
A. Absolute hypoalbuminemia
24. In a boy of 4 years after suffering from B. Absolute hyperalbuminemia
severe viral hepatitis, vomiting, episodes of C. Absolute hyperfibrinogenemia
nephropathy, seizures are observed. In the blood D. Proteinic blood composition is unchanged
there is hyperammonia. Violation of which of E. Absolute hyperglobulinemia
the biochemical process in the liver caused such
a condition of the patient? 29. In obstructive jaundice and bile ducts,
A. Disposal of ammonia. prothrombin insufficiency is often observed.
B. Decarboxylation of amino acids. With a deficiency in the organism of which
C. Disposal of Biogenic Amines. vitamin it is connected?
D. Synthesis of alphabets. A. K
E. Gluconeogenesis. B. B6
C. A
25. Upon toxic damage of hepatic cells resulting D. C
in disruption of liver function the patient E. E
developed edemas. What changes of blood
plasma are the main cause of edema 30. A patient has been admitted to the
development? contagious isolation ward with signs of jaundice
A. Decrease of albumin content caused by hepatitis virus.Which of the

53
hange E hange E
XC di XC di
F- t F- t
PD

PD
or

or
!

!
W

W
O

O
N

N
Y

Y
U

U
B

B
to

to
ww

ww
om

om
k

k
lic

lic
C

C
.c symptoms given below is strictly specific for syndrome. Which type of jaundice is developed

.c
w

w
tr re tr re
.

.
ac ac
k e r- s o ft w a k e r- s o ft w a

hepatocellular jaundice? in this patient?


A. Increase of ALT, AST level A. Gallstone
B. Hyperbilirubinemia B. Hemolytic jaundice
C. Bilirubinuria C. Hepatitis
D. Cholemia D. Chronic gastritis
E. Urobilinuria E. Chronic colitis

31. A 15-year-old boy has been diagnosed with 36. The patient turned to doctor with
acute viral hepatitis. What blood value should disturbances in the right side of the subarea.
be determined to confirm acute affection of After examination of the patient, the doctor was
hepatic cells? found yellow sclera. Laboratory diagnostic
A. Aminotransferase activity (AST, ALT) revealed increased ALT, negative stercobilin in
B. Unconjugated and conjugated bilirubin feces. Which type of disease is developed these
content symptoms?
C. Erythrocytes sedimentation rate (ESR) A. Hepatitis
D. Cholesterol content B. Haemolytic jaundice
E. Protein fraction content C. Chronic gastroduodenitis
D. Chronic colitis
32. Blood analysis of a patient with jaundice E. Chronic gastritis
reveals conjugated bilirubinemia, increased
concentration of bile acids. There is no 37. The 20-years-old man is diagnosed with
stercobilinogen in urine. What type of jaundice heredity deficiency of UDP-
is it? glucuronosyltransferase. Change in which blood
A. Obstructive jaundice parameter can prove this diagnosis?
B. Hepatocellular jaundice A. Indirect (unconjugated) bilirubin
C. Parenchymatous jaundice B. Direct (conjugated) bilirubin
D. Hemolytic jaundice C. Urobilin
E. Cythemolytic jaundice D. Stercobilinogen
E. Animal indican
33. Encephalopathy has developed in a child
with hemolytic disease of the newborn. What 38. The preterm newborn has a jaundice. Which
substance had increased in the child’s blood, enzyme is probably deficient in this baby?
resulting in damage to the CNS? A. UDP-glucosyltransferase
A. Unconjugated bilirubin B. alkaline phosphatase
B. Bilirubin-albumin complex C. acidic phosphatase
C. Bilirubin glucuronide D. catalase
D. Verdohemoglobin E. NAD+-dehydrogenase
E. Bileacids
39. The born in time newborn has yellow skin
34. Barbiturates prescribed for jaundice and mucus color. The most probable reason of
treatment because it induces UDP- this condition is temporary insufissiency of the
glucuronosyltransferase synthesis. Which following enzyme:
substance formation provide therapeutic effect? A. UDP-glucosyltransferase
A. Direct (conjugated) bilirubin B. Glucuronosyltransferase
B. Indirect (unconjugated) bilirubin C. Heme synthase
C. Biliverdin D. Heme oxygenase
D. Protoporphyrin E. Biliverdin reductase
E. Heme
40. The newborn has a physiological jaundice.
35. The patient came to doctor with compliance The level of blood free bilirubin considerably
about yellow sclera and skin. No higher than normal. What enzyme deficiency
encephalopathy, cholestasis or acholic can cause this

54
hange E hange E
XC di XC di
F- t F- t
PD

PD
or

or
!

!
W

W
O

O
N

N
Y

Y
U

U
B

B
to

to
ww

ww
om

om
k

k
lic

lic
C

C
.c A. UDP-glucosyltransferase bilirubin is normal. Which type of jaundice this

.c
w

w
tr re tr re
.

.
ac ac
k e r- s o ft w a k e r- s o ft w a

B. Transaminase patient is most probably develop?


C. Xanthine oxidase A. Hemolytic
D. Adenosine deaminase B. Parenchymatous (hepatic)
E. Heme oxygenase C. Mechanical
D. Newborn jaundice
41. Transfusion of Rh-incompatible blood E. Gilbert's syndrome
resulted in hemolytic jaundice development in
the patient. What laboratory blood value 46. The patient develops yellowish of skin and
confirms this type of jaundice? mucus membranes. Analysis of blood plasma
A. Accumulation of unconjugated bilirubin revealed increased total bilirubin, feces –
B. Reduction of unconjugated bilirubin increased stercobilin, urine – increased urobilin.
C. Accumulation of urobilinogen What type of jaundice this patient has?
D. Reduction of stercobilin A. Hemolytic
E. Reduction of conjugated bilirubin B. Gilbert's syndrome
C. Parenchymatous
42. The patient has a following symptoms: D. Obstructive
yellow skin, dark urine, yellow-dark feces. E. Cholestatic
Which substance you will predict to be
increased in blood serum? 47. After blood transfusion patient has
A. Free bilirubin yellowish of skin and mucus membrane,
B. Conjugated bilirubin increased total and indirect bilirubin, increased
C. Mesobilirubin urine urobilin and feces stercobilin. What kind
D. Verdoglobin of jaundice can be the reason?
E. Biliverdin A. Hemolytic jaundice.
B. Newborn jaundice.
43. A 46 year old woman suffering from C. Obstructive jaundice.
chololithiasis developed jaundice. Her urine D. Parenchymatous jaundice.
became dark-yellow and feces became E. Genetic jaundice.
colourless. Blood serum will have the highest
concentration of the following substance: 48. The patient who suffers from jaundice
A. Conjugated bilirubin because of increased bilirubin as part of its
B. Unconjugated bilirubin indirect fraction. Urine and feces have intensive
C. Biliverdin color. What is the most probable explanation to
D. Mesobilirubin these disruptions?
E. Urobilinogen A. Increased erythrocyte hemolysis
B. Disrupted liver urobilinogen transformation.
44. Enzymatic jaundices are accompanied by C. Disruption of direct bilirubin formation.
abnormal activity of UDPglucuronyl D. Liver parenchyma damage.
transferase. What compound is accumulated in E. Disruption of bile flow from liver
blood serum in case of these pathologies?
A. Unconjugated bilirubin 49. The 28-years-old woman came to infectious
B. Conjugated bilrubin department with yellow skin, sclera, mucus
C. Dehydrobilirubin membrane. Laboratory diagnosis revealed
D. Hydrobilirubin increased blood direct bilirubin; urobilinogen
E. Choleglobin and bilirubin in urine. Which of following
condition can lead to those symptoms?
45. The 20-years-old patient has a jaundice and A. Parenchymatous jaundice
the following laboratory parameter: increase B. Hemolytic jaundice
general blood bilirubin, predominantly by C. Kidney infarction
indirect (free) fraction; urine and feces has a D. Renal tuberculosis
high stercobilin; blood direct (conjugated) E. Mechanical jaundice

55
hange E hange E
XC di XC di
F- t F- t
PD

PD
or

or
!

!
W

W
O

O
N

N
Y

Y
U

U
B

B
to

to
ww

ww
om

om
k

k
lic

lic
C

C
.c 50. The 28-years old man came to doctor to 54. The patient has yellow skin, skin rash,

.c
w

w
tr re tr re
.

.
ac ac
k e r- s o ft w a k e r- s o ft w a

complaints about acute disturbances in the right general weakness. In the urine: there is no
side of the subarea, nausea, vomiting. urobilin. Which type of pathology this patient is
Objectively: jaundice of the skin and sclera; the most probably develop?
temperature of the body is elevated, the urine is A. Mechanical jaundice
dark color. Feces is hypochloric. The patient B. Parenchymal jaundice
have hyperbilirubinemia (bilateral and indirect C. Acute liver insufficiency
biliary), bilirubinuria, urobilinuria, D. Haemolytic jaundice
gipoproteinemiya, reduction of blood E. Chronic liver insufficiency
transfusion. Which of following condition can
lead to those symptoms? 55. The 43-years-old patient during 10 years has
A. Hemolitic jaundice a repetitive compliance about acute bellyache,
B. Subacute jaundice cramps, vision problems. His relatives have
C. Suprachondrial hemolytic jaundice Patient urine have red color. He was diagnosed
D. Acute cholecystitis with acute intermittent porphyria. The reason of
E. Acute pancreatitis he his disease is not being a vampire but
disrupted biosynthesis of:
51. The 48-years-old woman was delivered to A. Heme
hospital with compliance of general weakness, B. Bile acids
sleep disturbance. Objectively: skin and sclera C. Insulin
are yellow. Blood analysis has increased total D. Collagen
bilirubin with direct bilirubin predominance. E. Prostaglandin
Feces is acholic. Urine has dark color, because
of bile pigments. Which type of jaundice this 56. The patients with erythropoietic porphyria
patient has? (Gunther disease) have ultraviolet red-
A. Mechanical fluorescent tooth, sun-sensitive skin, red-
B. Hemolytic colored urine. Which enzyme deficiency cause
C. Parenchymatous this disease?
D. Gilbert's syndrome A. Uroporphyrinogen III synthase
E. Crigler–Najjar syndrome B. δ- aminolevulinic acid synthase
52. The patient was delivered to hospital with C. Uroporphyrinogen I synthase
complaints about general weakness and sleep D. Uroporphyrinogen decarboxylase
disturbance. Skin has yellow color. There is E. Ferrochelatase
increased blood direct bilirubin and bile acids.
Feces is acholic. Which type of disease can 57. The patient who suffers from anemia has
provoke those changes? increased protoporphyrin IX. Which mineral
A. Mechanical jaundice deficiency can lead to such pathology?
B. Hemolytic jaundice A. Iron
C. Pre-hepatic jaundice B. Potassium
D. Gilbert's syndrome C. Magnesium
E. Chronic cholecestitis D. Sodium
E. Phosphorus
53. The patient with jaundice has increased
direct bilirubin and bile acids in blood; no 58. The patient has increased photosensitivity,
stercobilinogen was revealed in urine. What after prolong sun exposure urine starts to get
type of jaundice can be the reason of these dark-red color. Which type of disease has this
symptoms? characteristic?
A. Mechanical A. Porphyria
B. Hepatic B. Alkaptonuria
C. Parenchymatous C. Albinism
D. Hemolytic D. Hemolytic jaundice
E. Post-hepatic E. Pellagra

56
hange E hange E
XC di XC di
F- t F- t
PD

PD
or

or
!

!
W

W
O

O
N

N
Y

Y
U

U
B

B
to

to
ww

ww
om

om
k

k
lic

lic
C

C
.c 59. The 33 years old patient worries 10 years.

.c
w

w
tr re tr re
.

.
ac ac
k e r- s o ft w a k e r- s o ft w a

Periodically, he addresses the doctor with 63. Detoxification of xenobiotics (drugs,


complaints of acute abdominal pain, epoxides, the arena oxides, aldehydes, nitro
convulsions, visual impairment. His relatives derivatives, etc.) and endogenous metabolites
are witnessing similar symptoms. Urine red. (estradiol, prostaglandins, leukotrienes) takes
Hospitalized with diagnoses - acute diarrhea place in the liver by conjugation with:
porphyria. The cause of the disease may be a A. Glutathione
violation of the biosynthesis of such a B. Aspartic acid
substance: C. Glycine
A. Gem D. S-Adenozylmethionin
B. Insulin E. Fosfoadenozyn
C.Chicken acids 64. For a patient suffering from chronic
D. Prostaglandins hepatitis, a load of sodium benzoate was carried
E. Collagen out to evaluate the degreasing function of the
liver. For the discharge of what substances with
60. A 43-year-old woman complains of general urine judge the neutralizing function of the
weakness, weight loss, apathy, and drowsiness. stomach?
Chronic lead intoxication confirmed laboratory - A. Hippuric acid
revealed hypochromic anemia. In the blood, an B. Phenylacetic acid
increased level of Zn-protoporphyrin and a C. Citric acid
reduced level of alpha-aminolevulinic acid, D. Valerian acid
indicating a violation of the synthesis: E. Oxalic acid
A. Heme
B. DNA 65. Patient has cirrhosis of the liver. The study
C. RNA of which of the listed substances that are
D. Protein excreted in the urine can characterize the state
E. Mevalonic acid of the antitoxic function of the liver?
A. Hippuric acid
61. A mother consulted a doctor about her 5- B. Ammonium salts
year-old child who develops erythemas, C. Creatinine
vesicular rash and skin itch under the influence D. Uric acid
of sun. Laboratory studies revealed decreased E. Amino acids
iron concentration in the blood serum, increased
uroporphyrinogen I excretion with the urine. 66. A woman of a chemical enterprise due to
What is the most likely inherited pathology in violation of the rules of safe work has suffered a
this child? toxic action of nitric acid and nitrites, which
A. Erythropoietic porphyria cause deamination of cytosine in the DNA.
B. Methemoglobinemia What enzyme initiates a chain of reparative
C. Hepatic porphyria processes?
D. Coproporphyria A. Uridine-DNA-glucosidase
E. Intermittent porphyria B. Cytidine triphosphate synthetase
C. Orthotidyl monophosphate-decarboxylase
62. A patient suffering from chronic hepatitis D. DNA-Dependent-RNA Polymerase
complains of increasing the sensitivity to E. Timidilatsintaza
barbiturates, which were earlier tolerated
without symptoms of intoxication. With the 67. In the 70's, scientists found that the cause of
disturbance of which of the liquor functions, severe jaundice of newborns is a disturbance of
this is connected at the highest possible level? the binding of bilirubin in hepatocytes. What
A. Detoxification substance is used to form a conjugate?
B. Hemopoietic A. Glucuronic acid
C. Hemodynamic B. Uric acid
D. Formation of bile C. Sulfuric acid
E. Phagocytic D. Lactic acid

57
hange E hange E
XC di XC di
F- t F- t
PD

PD
or

or
!

!
W

W
O

O
N

N
Y

Y
U

U
B

B
to

to
ww

ww
om

om
k

k
lic

lic
C

C
.c E. Pyruvic acid

.c
w

w
tr re tr re
.

.
ac ac
k e r- s o ft w a k e r- s o ft w a

71. In the liver, detoxification of natural


68. A 50-year-old patient with food poisoning metabolites and xenobiotics is impaired. Name
was prescribed a dropper with a 10% glucose cytochrome, the activity of which can be
solution. It not only provides the energy needs reduced:
of the body, but also performs a detoxification A. Cytochrome P-450
function due to the formation of a metabolite B. Cytochrome oxidase
that participates in the conjugation reaction: C. Hemoglobin
A. Gluuronation D. Cytochrome b
B. Sulfonation E. Cytochrome c1
C. Methylation
D. Glycosylation 72. Study of conversion of a food colouring
E. Hydroxylation agent revealed that neutralization of this
xenobiotic takes place only in one phase -
69. To determine the antitoxic function of the microsomal oxydation. Name a component of
liver, the patient is assigned sodium benzonate, this phase:
which in the liver turns into hippuric acid. What A. Cytochrome Р-450
compound is used for this? B. Cytochrome B
A. Glycine C. Cytochrome C
B. Methionine D. Cytochrome A
C. Cysteine E. Cytochrome oxidase
D. FAFS
E. UDF-glucuronic acid 73. The universal biological system of oxidation
of nonpolar compounds (drugs, toxic
70. In a patient with chronic alcoholic liver compounds), steroid hormones, cholesterol is
disease, processes of xenobiotics and microsomal oxidation. What is the name of the
endogenous toxic compounds biotransformation cytochrome, which is part of the oxygenase
are disturbed. Reducing the activity of which chain of the microsome.
chromoprotein can be the cause of this? A. Cytochrome P 450.
A. Cytochrome P-450 B. Cytochrome a3.
B. Hemoglobin C. Cytochrome in.
C. Cytochrome Oxidase D. Cytochrome c.
D. Cytochrome b E. Cytochrome a.
E. Cytochrome c1

58
hange E hange E
XC di XC di
F- t F- t
PD

PD
or

or
!

!
W

W
O

O
N

N
Y

Y
U

U
B

B
to

to
ww

ww
om

om
k

k
lic

lic
C

C
.c

.c
Pathologic urine components
w

w
tr re tr re
.

.
ac ac
k e r- s o ft w a k e r- s o ft w a

1. A 13 year-old boy is complaining about A. Cystinuria.


general weakness, dizziness, fatigue. Mental B. Alpathururia.
retardation is detected. The examination C. Cystitis
revealed high concentrations of valine, D. Phenylketonuria.
isoleucine, and leucine in blood and urine. The E. Hartnup's disease.
urine has a specific smell. What is the most
possible diagnosis? 6. A 36-year-old female patient has a history of
A. Maple syrup urine disease collagen disease. Urine analysis is likely to
B. Addison’s disease reveal an increased concentration of the
C. Tyrosinosis following metabolite:
D. Histidinemia A. Oxyproline
E. Grave’s disease B. Indican
C. Creatinine
2. Excess of phenylpyruvate and phenylacetate D. Urea
was detected in the urine sample of a 6 day-old E. Urobilinogen
baby. Which amino acid metabolism is
disturbed? 7. A patient with systemic scleroderma has an
A. Phenylalanine intensified collagen destruction. Collagen
B. Tryptophan destruction will be reflected by intensified
C. Methionine urinary excretion of the
D. Histidine following amino acid:
E. Arginine A. Oxyproline
B. Alanine
3. In a sick child, a raised level of C. Tryptophane
phenylpyruvate was detected in the urine (in the D. Serine
norm practically absent). The amount of E. Phenylalanine
phenylalanine in the blood is 350 mg/l (about 15
mg/l). Which disease is characterized by the 8. A child has physical and mental retardation,
above symptoms? serious abnormalities in connective tissue of
A. Fenilketonuria. internal organs; urine contains keratan sulfates.
B. Albin. This is caused by metabolic disorder of the
C. Alpathonuria. following substance:
D. Gout A. Glycosaminoglycan
E. Tyrosinosis. B. Collagen
C. Elastin
4. A child with mental and physical D. Fibronectin
development retardation was delivered to the E. Hyaluronic acid
hospital. Phenylpyruvate was found in the urine.
Disturbance of which process is the reason for 9. A traumatology unit received a patient with
this pathology? crushed muscular tissue. What biochemical
A. Amino acid metabolism indicator of urine will be raised in this case?
B. Lipid metabolism A. Creatinine
C. Carbohydrate metabolism B. Total lipids
D. Water-salt balance regulation C. Glucose
E. Calcium and phosphate homeostasis D. Mineral salts
E. Uric acid
5. A 12-year-old boy in the urine revealed high
levels of all amino acids in the aliphatic row. In 10. A biochemical urine analysis has been
this case, the highest excretion of cysteine and performed for a patient with progressive
cysteine was noted. In addition, ultrasound of muscular dystrophy. In the given case muscle
the kidneys showed the presence of stones in disease can be confirmed by the high content of
them. Choose a possible pathology. the following substance in urine:
59
hange E hange E
XC di XC di
F- t F- t
PD

PD
or

or
!

!
W

W
O

O
N

N
Y

Y
U

U
B

B
to

to
ww

ww
om

om
k

k
lic

lic
C

C
.c A. Creatine B. Phenylalanine

.c
w

w
tr re tr re
.

.
ac ac
k e r- s o ft w a k e r- s o ft w a

B. Porphyrin C. Alanine
C. Urea D. Methionine
D. Hippuric acid E. Asparagine
E. Creatinine
16. The 28-years old man came to doctor to
11. A biochemical urine analysis has been complaints about acute disturbances in the right
performed for a patient with progressive side of the subarea, nausea, vomiting.
muscular dystrophy. In the given case muscle Objectively: jaundice of the skin and sclera; the
disease can be confirmed by the high content of temperature of the body is elevated, the urine is
the following substance in urine: dark color. Feces is hypochloric. The patient
A. Creatine have hyperbilirubinemia (bilateral and indirect
B. Porphyrin biliary), bilirubinuria, urobilinuria,
C. Urea gipoproteinemiya, reduction of blood
D. Hippuric acid transfusion. Which of following condition can
E. Creatinine lead to those symptoms?
A. Hemolitic jaundice
12. A baby has coloured sclera and mucouses B. Subacute jaundice
and her urine is darkening when exposed to air. C. Suprachondrial hemolytic jaundice
Homogentisic acid was found in urine and blood D. Acute cholecystitis
samples. What is the reason for this condition? E. Acute pancreatitis
A Alcaptonuria
B. Albinism 17. A patient has an increased pyruvate
C. Galactosemia concentration in blood. A large amount of it is
D. Cystinuria excreted with the urine. What vitamin is lacking
E. Histidinemia in this patient?
A. B1
13. A 5-year old child has darkened urine but no B. E
bile enzymes were found in it. The child is C. B3
diagnosed with alcaptonuria. Which enzyme D. B6
deficiency is the reason for it? E. B2
A. Homogentisic acid oxydase
B. Phenylalanine hydroxylase 18. A patient, who has been subsisting
C. Tyrosinase exclusively on polished rice, has developed
D. Oxyphenylpyruvate oxydase polyneuritis due to thiamine deficiency. What
E. Phenylpyruvate decarboxylase substance is an indicator of such avitaminosis,
when it is excreted with urine?
14. A lab test revealed increased concentration A. Pyruvic acid
of leucine, valine, isoleucine and ketone B. Malate
derivatives in the blood and urine. The urine has C. Methylmalonic acid
a specific smell of maple syrup. Which enzyme D. Uric acid
deficiency is typical for this condition? E. Phenyl pyruvate
A. Amino acid dehydrogenase
B. Aminotranspherase 19. A patient 43 years old with chronic atrophic
C. Glucose-6-phosphatase gastritis and hyperchromic megaloblastic
D. Phosphofructokinase anemia increased methylmalonic acid excretion
E. Phosphofructomutase in the urine. The lack of which vitamin caused
the occurrence of this symptom complex?
15. Excretion of homogentisic acid with the A. В12
urine is the reason for the condition called B. В2
alcaptonuria. Disturbance of which amino acid C. В3
metabolism causes this condition? D. В5
A. Tyrosine E. В6

60
hange E hange E
XC di XC di
F- t F- t
PD

PD
or

or
!

!
W

W
O

O
N

N
Y

Y
U

U
B

B
to

to
ww

ww
om

om
k

k
lic

lic
C

C
.c components are revealed in course of laboratory

.c
w

w
tr re tr re
.

.
ac ac
k e r- s o ft w a k e r- s o ft w a

20. A hereditary disease - homocystinuria - is examination of the patient’s urine?


caused by disturbed transformation of A. Glucose, ketone bodies
homocysteine in to methionine. Accumulated B. Protein, aminoacids
homocysteine forms its dimer (homocystine) C. Protein, creatine
that can be found in urine. What vitamin D. Bilirubin, urobilin
preparation can decrease homocysteine E. Blood
production?
A. Vitamin B12 25. A patient at the early stage of diabetes
B. Vitamin C mellitus was found to have polyuria.What is its
C. Vitamin B1 cause?
D. Vitamin B2 A. Hyperglycemia
E. Vitamin PP B. Ketonemia
C. Hypocholesterolemia
21. In a newborn child there is a decrease in D. Hypercholesterolemia
intensity of sucking, frequent vomiting, E. Hyperkaliemia
hypotension. In urine and blood, the
concentration of citrulline is significantly 26. At the reception to the therapist came a man
increased. What metabolic process is affected? of 37 years of age with complaints of periodic
A. Ornithine cycle intense pain in the joints of the thumb and their
В. Glycolysis swelling. In the urine: slightly acidic reaction
C. Core's cycle and pink color. With the presence of which
D. СТК substances can these changes be associated?
E. Gluconeogenesis A. Salt of uric acid
B. Chlorides
22. In the urine of the newborn, determined by C. Amoniev's salt
citrulline and high levels of ammonia. Indicate D. Phosphate calcium
which substance is most likely to be inflicted on E. Magnesium sulphate
a baby?
A. Urea. 27. On the basis of laboratory analysis, the
B. Bilirubin. patient was diagnosed with gout. Based on
C. Creatine. laboratory analysis, the patient confirmed the
D. Creatinine. diagnosis of gout. What was the analysis for the
E. Uric acid. diagnosis?
A. Determination of uric acid in the blood and
23. According to the results of glucose tolerance urine
test a patient has no disorder of carbohydrate B. Determination of urinary creatinine
tolerance. Despite that glucose is detected in the C. Determination of residual nitrogen in the
patients’s urine (5 mmol/l). The patient has been blood
diagnosed with renal diabetes. What renal D. The determination of urea in the blood and
changes cause glucosuria in this case? urine
A. Decreased activity of glucose reabsorption E. Determination of urine ammonia
enzymes
B. Increased activity of glucose reabsorption 28. A patient has been operated due to acute
enzymes abdomen. His urine is brown, concentration of
C. Exceeded glucose reabsorption threshold indican is higher than 93 mmol/day. What does
D. Increased glucose secretion it indicate?
E. Increased glucose filtration A. High intensity of protein decomposition
processes in the intestine
24. A 42 year old woman diagnosed with B. Decreased activity of the urea cycle enzymes
diabetes mellitus was admitted the C. Acceleration of aromatic amino acids
endocrinological department with complaints of desamination
thirst, excessive appetite. What pathological D. Disturbance of the kidney absorption

61
hange E hange E
XC di XC di
F- t F- t
PD

PD
or

or
!

!
W

W
O

O
N

N
Y

Y
U

U
B

B
to

to
ww

ww
om

om
k

k
lic

lic
C

C
.c E. Decrease of ammonia detoxification

.c
w

w
tr re tr re
.

.
ac ac
k e r- s o ft w a k e r- s o ft w a

34. Analysis of a dentist’s urine obtained at the


29. A 43 year-old woman has been operated due end of his working day revealed protein
to acute abdomen. Her urine is brown and the concentration at the rate of 0,7 g/l. His morning
concentration of indican in the blood has urine hadn’t such changes. What is this
increased rapidly. What does it indicate? phenomenon called?
A.High intensity of protein decomposition A. Functional proteinuria
B. Acceleration of amino acids desamination B. Organic proteinuria
C. Supression of glomerular filtration C. Nonselective proteinuria
D. Decreased intensity of the urea cycle D. Extrarenal proteinuria
E. Inhibition of gluconeogenesis E. Hematuria

30. A 60 year-old man suffers from chronic 35. Chronic glomerulonephritis was diagnosed
intestinal obstruction. The processes of protein in a 34-year-old patient 3 years ago. Edema has
decomposition in the large intestine are developed within the last 6 monthes. What
intensified. What is the evidence of these caused the edema?
processes? A. Proteinuria
A. Indicanuria B. Hyperproduction of vasopressin
B. Bilirubinuria C. Liver disfunction of protein formation
C. Hyperuricosuria D. Hyperosmolarity of plasma
D. Creatinuria E. Hyperaldosteronism
E. Glucosuria
36. Glucose concentration in a patient’s blood is
31. A newborn child gains weight very slowly, 15 millimole/l (reabsorption threshold is 10
his urine contains too much orotic acid that is millimole/l). What effect can be expected?
indicative of disturbed synthesis of pyrimidine A. Glucosuria
nucleotides. What metabolite should be used in B. Diuresis reduction
order to normalize metabolism? C. Reduced glucose reabsorption
A. Uridine D. Reduced vasopressin secretion
B. Adenosine E. Reduced aldosterone secretion
C. Guanosine
D. Thymidine 37. A patient with rheumatoid arthritis has been
E. Histidine given hydrocortisone for a long time. He has
developed hyperglycemia, polyuria, glycosuria,
32. In orоtaciduria the release of Orotic acid is thirst. These complications of treatment result
many times higher than normal. Synthesis of from the activation of the following process:
what substances will be disturbed in this A. Gluconeogenesis
pathology? B. Glycogenolysis
A. Pyrimidine nucleotides. C. Glycogenesis
B. Biogenic Amines. D. Glycolysis
C. Purine nucleotides. E. Lipolysis
D. Urea
E. Uric acid 38. A 28-year-old patient with Itsenko-Cushing
syndrome Hyperglycemia, glycosuria were
33. The patient has increased photosensitivity, detected hyperglycemia, glycosuria. The main
after prolong sun exposure urine starts to get mechanism of hyperglycemia in this patient is
dark-red color. Which type of disease has this stimulation:
characteristic? A. Gluconeogenesis
A. Porphyria B. Liver glycogenolysis
B. Alkaptonuria C. Muscle glycogenolysis
C. Albinism D. Intestinal glucose absorption
D. Hemolytic jaundice E. Synthesis of glycogen
E. Pellagra

62
hange E hange E
XC di XC di
F- t F- t
PD

PD
or

or
!

!
W

W
O

O
N

N
Y

Y
U

U
B

B
to

to
ww

ww
om

om
k

k
lic

lic
C

C
.c 39. Fructosuria is known to be connected with E. Sphingomyelinase

.c
w

w
tr re tr re
.

.
ac ac
k e r- s o ft w a k e r- s o ft w a

inherited deficiency of fructose-1-phosphate


aldolase.What product of fructose metabolism 44. A patient presents with high content of
will accumulate in the organism resulting in vasopressin (antidiuretic hormone) in the blood.
toxicaction? What changes in the patient’s diuresis will
A. Fructose-1-phosphate occur?
B. Glucose-1-phosphate A. Oliguria
C. Glucose-6-phosphate B. Polyuria
D. Fructose-1,6-biphosphate C. Anuria
E. Fructose-6-phosphate D. Glycosuria
E. Natriuria
40. Prophylactic examination of a patient
revealed hyperglycemia, ketonuria, polyuria, 45. A patient with tress and painful sensation
glycosuria. What form of acidbase balance before a visit to the dentist is accompanied by
disorder is the case? anuria (lack of urination). This phenomenon is
A. Metabolic acidosis due to an increase in:
B. Gaseous acidosis A. Secretion of vasopressin and adrenaline
C. Nongaseous acidosis B. Activity of the parasympathetic nervous
D. Gaseous alkalosis system
E. Metabolic alkalosis C. Activity antinociceptive system
D. Secretion of vasopressin and a decrease in
41. Examination of a 2 year-old child with renal adrenaline
insufficiency revealed hyperoxaluria that caused E. Secretion of adrenaline and a decrease in
depositing of calcium oxalate stones in the vasopressin
kidneys. The reason for this condition is a
disturbance of metabolism of the following 46. A patient with pituitary tumor complains of
aminoacid: increased daily diuresis (polyuria). Glucose
A. Glycine concentration in blood plasma equals 4,8
B. Lysine mmol/l. What hormone can be the cause of this
C. Methionine if its secretion is disturbed?
D. Arginine A. Vasopressin
E. Histidine B. Aldosterone
C. Natriuretic hormone
42. Cardinal symptoms of primary D. Insulin
hyperparathyroidism are osteoporosis andrenal E. Angiotensin I
lesion along with development of urolithiasis.
What substance makes up the basis of these 47. After a person had drunk 1,5 liters of water,
calculi in this disease? the amount of urine increased significantly, and
A. Calcium phosphate its relative density decreased to 1,001. These
B. Uric acid changes are a result of decreased water
C. Cystine reabsorption in the distal nephron portion due to
D. Bilirubin reduced secretion of:
E. Cholesterol A. Vasopressin
B. Aldosterone
43. In the laboratory examination of blood of a C. Angiotensin II
person who was bitten by a change, hemolysis D. Renin
of erythrocytes, hemoglobinuria was revealed. E. Prostaglandins
The action of a mixed poison is due to the
presence of an enzyme in it: 48. A man has a considerable decrease in
A. Phospholipase A2 diuresis as a result of 1,5 l blood loss. The
B. Phospholipase D primary cause of such diuresis disorder is the
C. Phospholipase S. hypersecretion of the following hormone:
D. Phospholipase A1 A. Vasopressin

63
hange E hange E
XC di XC di
F- t F- t
PD

PD
or

or
!

!
W

W
O

O
N

N
Y

Y
U

U
B

B
to

to
ww

ww
om

om
k

k
lic

lic
C

C
.c B. Corticotropin level of glucose in blood is normal, there is trno

.c
w

w
tr re re
.

.
ac ac
k e r- s o ft w a k e r- s o ft w a

C. Natriuretic glucose in urine. What hormone deficit can


D. Cortisol cause such changes?
E. Parathormone A. Vasopressin
B. Oxytocin
49. Analysis of urine from a 24-year-old man C. Insulin
revealed the following changes: daily diuresis - D. Triiodothyronine
10 l, relative density - 1,001, qualitative E. Cortisol
alterations are absent. A patient complains of
excessive thirst, frequent urination. What is the 53. A 19-year-old male was found to have an
most likely cause of this disease? elevated level of potassium in the secondary urine.
A. Vasopressin hyposecretion These changes might have been caused by the
B. Glucocorticoid hypersecretion increase in the following hormone level:
C. Vasopressin hypersecretion A. Aldosterone
D. Relative insulin insufficiency B. Oxytocin
C. Adrenaline
E. Aldosteron hypersecretion D. Glucagon
E. Testosterone
50. A 50 year old patient complains of excessive
thirst, drinks a lot of water; expressed polyuria. 54. A concentrated solution of sodium chloride
Blood glucose - 4.8 mmol / l. There is no was intravenously injected to an animal. This
glucose and acetone bodies in urine, colorless caused decreased reabsorption of sodium ions in
urine, specific gravity - 1.002 - 1.004. What is the renal tubules. It is the result of the following
the cause of polyuria? changes of hormonal secretion:
A. Lack of vasopressin A. Aldosterone reduction
B. Hypothyroidism B. Aldosterone increase
C. Insulin Deficiency C. Vasopressin reduction
D. Aldosteronism D. Vasopressin increase
E. Thyrotoxicosis E. Reduction of atrial natriuretic factor
51. A patient with damage to the posterior lobe 55. At ria of a test animal were super distended
of the pituitary gland has increase in daily with blood,which resulted in decreased
diuresis to 10-15 liters What is the main reabsorption of Na+ and water in renal
mechanism in the development of polyuria? tubules.This can be explained by the effect of
A. Vasopressin deficiency the following factor on the kidneys:
B. Excess vasopressin A. Natriuretic hormone
C. Excess aldosterone B. Aldosterone
D. Excess natriuretic factor C. Renin
E. Corticotropin deficiency D. Angiotensin
E. Vasopressin
52. A 20 year old patient complains of excessive
thirst and urinary excretion upto 10 L a day. The

64
hange E hange E
XC di XC di
F- t F- t
PD

PD
or

or
!

!
W

W
O

O
N

N
Y

Y
U

U
B

B
to

to
ww

ww
om

om
k

k
lic

lic
C

C
.c

.c
Biochemistry of nervous tissue
w

w
tr re tr re
.

.
ac ac
k e r- s o ft w a k e r- s o ft w a

1. A substance that is synthesized in the central A. Hyperpolarization


nervous system and simulates the effects of B. Depolarization
morphine can be used for analgesia. Specify the C. Action potential
following substance: D. Local response
A. β-endorphin E. There will be no changes
B. Oxytocin
C. Vasopressin 7. A 50-year-old man came to a hospital with
D. Calcitonin complaints of memory disorders, painful
E. Somatoliberin sensations along the nerve trunks, decreased
mental ability, circulatory disorders and
2. Cell membrane restpotential changed from-85 dyspepsia. Anamnesis states excessive alcohol
to-90mV. It can becaused by activation of the consumption. What vitamin deficiency can
following cell membrane channels: result in such symptoms?
A. Potassium A. Thiamine
B. Sodium B. Niacin
C. Potassium and sodium C. Retinol
D. Calcium E. Potassium and calcium D. Calciferol
E. Riboflavin
3. The speed of the excitation nerve fibers is
120 m / s. Which of the above factors, above all, 8. Decarboxylation of glutamate induces
provides such speed? production of gamma aminobutyric acid
A. Presence of myelin sheath (GABA) neurotransmitter. After breakdown,
B. Great resting potential GABA is converted into a metabolite of the
C. Large amplitude potential effects citric acid cycle, that is:
D. Low threshold of depolarization A. Succinate
E. A big factor of reliability B. Citric acid
C. Malate
4. The patient is trembling hands, which is D. Fumarate
associated with Parkinson's disease. What E. Oxaloacetate
mediator deficiency in striopalidar structures
leads to such symptoms? 9. Degenerative changes in posterior and lateral
A. Dopamine columns of spinal cord (funicular myelosis)
B. GABA caused by methylmalonic acid accumulation
C. Substance P occur in patients with B12-deficiency anemia.
D. Norepinephrine This results in synthesis disruption of the
E. Serotonin following substance:
A. Myelin
5. An 84-year-old patient suffers from B. Acetylcholine
parkinsonism. One of the pathogenetic C. Norepinephrine
development elements of this disease is D. Dopamine
deficiency of a certain mediator in some of the E. Serotonin
brain structures. Name this mediator:
A. Dopamine 10. After pharmacological blockade ion
B. Adrenaline channels of the nerve fiber membrane resting
C. Noradrenaline potential decreased from -90 to -80 mV. What
D. Histamine channels were blocked?
E. Acetylcholine A. Potassium
B. Sodium
6. In the experiment, the permeability of the C. Calcium
excitable cell membrane to potassium ions was D. Magnesium
magnified. What changes in the electrical state E. Chloric
of the membrane will occur?
65
hange E hange E
XC di XC di
F- t F- t
PD

PD
or

or
!

!
W

W
O

O
N

N
Y

Y
U

U
B

B
to

to
ww

ww
om

om
k

k
lic

lic
C

C
.c 11. Disruption of nerve fiber myelinogenesis concentration increases in the brain under the

.c
w

w
tr re tr re
.

.
ac ac
k e r- s o ft w a k e r- s o ft w a

causes neurological disorders and mental action of antidepressants?


retardation. These symptoms are typical for A. Serotonin
hereditary and acquired alterations in the B. Dopamine
metabolism of: C. Glycine
A. Sphingolipids D. Taurine
B. Neutral fats E. Norepinephrine
C. Higher fatty acids
D. Cholesterol 16. Brain cells are very sensitive to energy
E. Phosphatidic acid deficiency, which can be caused by a high
content of ammonia, which stimulates the
12. A patient complained about dizziness, outflow of alpha-ketoglutarate from:
memory impairment, periodical convulsions. It A. Citric acid cycle
was revealed that these changes were caused by B. Ornithine cycle
a product of decarboxylation of glutamic acid. C. Glycolysis
Name this product: D. Glycogenolysis
A. GABA E. Pentose phosphate pathway
B. Pyridoxal phosphate
C. TDP 17. Depressions and emotional insanities result
D. ATP from the deficit of noradrenalin, serotonin and
E. UDP other biogenic amines in the brain. Their
concentration in the synapses can be increased
13. A patient presents with dysfunction of the by means of the antidepressants that inhibit the
cerebral cortex accompanied by epileptic following enzyme:
seizures. He has been administered a biogenic A. Monoamine oxidase
amine synthetized from glutamate and B. Diamine oxidase
responsible for central inhibition. What C. L-amino-acid oxidase
substance is it? D. D-amino-acid oxidase
A. γ-aminobutyric acid E. Phenylalanine-4-monooxygenase
B. Serotonin C.
С. Dopamine 18. Pharmacological effects of antidepressants
D. Acetylcholine are based upon blocking (inhibiting) the enzyme
E. Histamine that acts as a catalyst for the breakdown of
biogenic amines noradrenalin and serotonin in
14. An unconscious patient was taken by the mitochondria of cephalic neurons. What
ambulance to the hospital. On objective enzyme takes part in this process?
examination the patient was found to have no A. Monoamine oxidase
reflexes, periodical convulsions, irregular B. Transaminase
breathing. After laboratory examination the C. Decarboxylase
patient was diagnosed with hepatic coma. D. Peptidase
Disorders of the central nervous system develop E. Lyase
due to the accumulation of the following
metabolite: 19. A 9-month-old infant is fed with artificial
A. Ammonia formulas with unbalanced vitamin B6
B. Urea concentration. The infant presents with pellagral
C. Glutamine dermatitis, convulsions, anaemia. Convulsion
D. Bilirubin development might be caused by the disturbed
E. Histamine formation of:
A. GABA
15. For the treatment of depression of different B. Histamine
genesis, antidepressants are used, which are C. Serotonin
monoamine oxidase inhibitors. What substance D. DOPA
is a "neurotransmitter of well-being" and its E. Dopamine

66
hange E hange E
XC di XC di
F- t F- t
PD

PD
or

or
!

!
W

W
O

O
N

N
Y

Y
U

U
B

B
to

to
ww

ww
om

om
k

k
lic

lic
C

C
.c inhibitor aldehyde dehydro- nazy The increase

.c
w

w
tr re tr re
.

.
ac ac
k e r- s o ft w a k e r- s o ft w a

20. Cerebral trauma caused increase of in blood of what tabolita aversion to al to


ammonia formation. What aminoacid takes part someone?
in removal of ammonia from A. Acetaldehyde
cerebral tissue? B. Ethanol
A. Glutamic C. Malonic aldehyde
B. Tyrosine D. Propionic aldehyde
C. Valine E. Methanol
D. Tryptophan
E. Lisine 25. Soldiers who were injured in the midst of
the battle may not be aware of the pain until it is
21. Ammonia is a very toxic substance, completed. What hormones are opiate
especially for nervous system. What substance antinociceptive system reduce the sensation of
takes the most active part in ammonia pain?
detoxication in brain tissues? A. Endorfines
A. Glutamic acid B. Serotonines
B. Lysine C. Vasopressin
C. Proline D. Aldosterone
D. Histidine E. Oxytocine
E. Alanine
26. In an excitable cell the ion channels were
22. A patient presented to a hospital with blocked. It hasn’t changed essentially the value
complaints about quick fatigability and of rest potential, but the cell lost its ability to
significant muscle weakness. Examination generate AP (action potential). What channels
revealed an autoimmune disease that were blocked?
causes functional disorder of receptors in the A. Natrium
neuromuscular synapses. This will result in the B. Potassium
disturbed activity of the following mediator: C. Natrium and potassium
A. Acetylcholine D. Chloric
B. Noradrenaline E. Calcium
C. Dopamine
D. Serotonin 27. The infant has epileptiform convulsions
E. Glycine caused by vitamin B6 deficiency. This is due to
a decrease in the nervous tissue of the inhibitory
23. Glutamate decarboxylation results in mediator, γ-aminobutyric acid. What is the
formation of inhibitory transmitter in CNS. enzyme's activity reduced in this case?
Name it: A. Glutamate decarboxylase
A. GABA B. Alanine aminotransferase
B. Glutathione C. Glutamate dehydrogenase
C. Histamine D. Pyridoxal kinase
D. Serotonin E. Glutamate Synthetase
E. Asparagine

24. In medical practice for alcoholism


prevention widely used teturam, which is

67
hange E hange E
XC di XC di
F- t F- t
PD

PD
or

or
!

!
W

W
O

O
N

N
Y

Y
U

U
B

B
to

to
ww

ww
om

om
k

k
lic

lic
C

C
.c

.c
Biochemistry of muscle tissue
w

w
tr re tr re
.

.
ac ac
k e r- s o ft w a k e r- s o ft w a

1. Buffer capacity of a worker’s blood was


decreased due to exhausting muscular work. By 6. A 35-year-old man developed acute heart
coming of what acid substance in the blood can failure while running for a long time. What
this state be explained? changes in ionic composition can be observed in
A. Lactate the cardiac muscle?
B. Pyruvate A. Accumulation of Na+ and Ca2+ ions in the
C. 1,3-bisphosphoglycerate myocardium cells
D. α-ketoglutarate B. Accumulation of K+ and Mg2+ ions in the
E. 3-phosphoglycerate myocardium cells
C. Reduction of Na+ and Ca2+ ions in the
2. A biochemical urine analysis has been myocardium cells
performed for a patient with progressive D. Reduction of K+ and Mg2+ ions in the
muscular dystrophy. In the given case muscle extracellular space
disease can be confirmed by the high content of E. Reduction of Na+ and Ca2+ ions in the
the following substance in urine: extracellular space
A. Creatine
B. Porphyrin 7. A patient is diagnosed with cardiac infarction.
C. Urea Blood test for cardiospecific enzymes activity
D. Hippuric acid was performed. Which of the enzymes has three
E. Creatinine isoforms?
A. Creatine kinase
3. A considerable increase of activity of МВ- B. Lactate dehydrogenase
forms of CPK (creatinephosphokinase) and C. Aspartate transaminase
LDH-1 was revealed on the examination of D. Alanine transaminase
patient’s blood. What is the most likely E. Pyruvate kinase
pathology?
A. Miocardial infarction 8. A 50-year-old woman diagnosed with cardiac
B. Hepatitis infarction has been delivered into an intensive
C. Rheumatism care ward. What enzyme will be the most active
D. Pancreatitis during the first two days?
E. Cholecystitis A. Aspartate aminotransferase
B. Alanineaminotransferase
4. With intense physical work, muscle tissue C. Alanineaminopeptidase
accumulates lactic acid, which diffuses into the D. LDH4
blood and is absorbed by the liver and the heart. E. LDH5
What process ensures the recovery of glycogen
stores in the muscles? 9. Untrained people often have muscle pain
A. The Cori Cycle after sprints as a result of lactate accumulation.
B. Citric Acid Cycle This might be caused by intensification of the
C. Urea Cycle following biochemical process:
D. A Cycle of Tricarboxylic Acids A. Glycolysis
E. Hexose Monophosphate Pathway B. Gluconeogenesis
C. Pentose phosphate pathway
5. A sportsman needs to improve his sporting D. Lipogenesis
results. He was recommended to take a E. Glycogenesis
preparation that contains carnitine. What
process is activated the most by this compound? 10. A patient with suspected diagnosis
A. Fatty acids transporting "progressing muscular dystrophy"got his urine
B. Amino acids transporting tested.What compound will confirm this
C. Calcium ions transporting diagnosis if found in urine?
D. Glucose transporting A. Kreatine
E. Vitamin K transporting B. Collagen
68
hange E hange E
XC di XC di
F- t F- t
PD

PD
or

or
!

!
W

W
O

O
N

N
Y

Y
U

U
B

B
to

to
ww

ww
om

om
k

k
lic

lic
C

C
.c C. Porphyrin C. Acetyl CoA

.c
w

w
tr re tr re
.

.
ac ac
k e r- s o ft w a k e r- s o ft w a

D. Myoglobin D. Glucose 6-phosphate


E. Calmodulin E. Oxaloacetate

11. Rheography of an 18 year old student during 16. A 49-year-old driver complains about
exercise showed redistribution of blood flow unbearable constricting pain behind the
between organs. The peak blood flow will be breastbone irradiating to the neck. The pain
observed in the following vessels: arose 2 hours ago. Objectively: the patient’s
A. Skeletal muscles condition is grave, he is pale, heart tones are
B. Liver decreased. Laboratory studies revealed high
C. Cerebrum activity of creatine kinase and LDH1. What
D. Kidneys disease are these symptoms typical for?
E. Gastrointestinal tract A. Acute myocardial infarction
B. Acute pancreatitis
12. A 46-year-old female patient has continuous C. Stenocardia
history of progressive muscular (Duchenne’s) D. Cholelithiasis
dystrophy. Which blood enzyme changes will E. Diabetes mellitus
be of diagnostic value in this case?
A. Creatine phosphokinase 17. Cytoplasm of the myocytes contains a lot of
B. Lactate dehydrogenase dissolved metabolites resulting from glucose
C. Pyruvate dehydrogenase oxidation. Name the metabolite that turns
D. Glutamate dehydrogenase directly into lactate:
E. Adenylate cyclase A. Pyruvate
B. Oxaloacetate
13. A 50-year-old patient complains about C. Glycerophosphate
general weakness, appetite loss and cardiac D. Glucose-6-phosphate
arrhythmia. The patient presents with muscle E. Fructose-6-phosphate
hypotonia, flaccid paralyses, weakened
peristaltic activity of the bowels. Such condition 18. A patient came to the hospital complaining
might be caused by: about quick fatigability and apparent muscle
A. Hypokaliemia weakness. Examination revealed an
B. Hypoproteinemia autoimmune disease that causes disorder of
C. Hyperkaliemia functional receptor condition in neuromuscular
D. Hypophosphatemia synapses. What transmitter will be blocked?
E. Hyponatremia A. Acetylcholine
B. Noradrenalin
14. Some students developed myodynia after C. Dopamine
continuous physical activity during physical D. Serotonin
education. The reason for such condition was E. Glycine
accumulation of lactic acid in the skeletal
muscles. It was generated in the students’ 19. During processing of the atypical
bodies after activation of the following process: cardiomyocytes of the synaptic node with a
A. Glycolysis biologically active substance, an increase in
B. Gluconeogenesis their membrane potential was recorded due to
C. Pentose-phosphate cycle increased permeability for potassium ions. What
D. Lipolysis biologically active substance affects
E. Glycogeny cardiomyocytes?
A. Acetylcholine
15. After a sprint an untrained person develops B. Adrenalin
muscle hypoxia. This leads to the accumulation C. Noradrenaline
of the following metabolite in muscles: D. Thyroxine
A. Lactate E. Atriopeptide
B. Ketone bodies

69
hange E hange E
XC di XC di
F- t F- t
PD

PD
or

or
!

!
W

W
O

O
N

N
Y

Y
U

U
B

B
to

to
ww

ww
om

om
k

k
lic

lic
C

C
.c 20. 12 hours after an accute attack of A. Transporting of fatty acids to mitochondrions

.c
w

w
tr re tr re
.

.
ac ac
k e r- s o ft w a k e r- s o ft w a

retrosternal pain a patient presented a jump of B. Regulation of Ca2+ rate in mitochondrions


aspartate aminotransferase activity in blood C. Substrate phosphorylation
serum. What pathology is this deviation typical D. Lactic acid utilization
for? E. Actin and myosin synthesis
A. Myocardium infarction
B. Viral hepatitis 24. After a sprint an long distantion the skeletal
C. Collagenosis muscle training person uses glucose for the
D. Diabetes mellitus purpose of obtaining energy ATP for muscle
E. Diabetes insipidus contraction. Specify basic glucose utilization
process in these conditions:
21. A patient presents high activity of LDH 1,2, A. Aerobic glycolysis
aspartate aminotransferase, creatine B. Anaerobic glycolysis
phosphokinase. In what organ (organs) is the C. Glycogenolysis
development of a pathological process the most D. Gluconeogenesis
probable? E. Glycogenesis
A. In the heart muscle (initial stage of
myocardium infarction) 25. The gluconeogenesis is activated in the liver
B. In skeletal muscles (dystrophy, atrophy) after intensive physical trainings.What
C. In kidneys and adrenals substance is utilized in gluconeogenesis first of
D. In connective tissue all in this case:
E. In liver and kidneys A. Lactate
B. Pyruvate
22. A traumatology unit received a patient with C. Glucose
crushed muscular tissue. What biochemical D. Glutamate
indicator of urine will be raised in this case? E. Alanine
A. Creatinine
B. Total lipids 26. 6 hours after the myocardial infarction a
C. Glucose patient was found to have elevated level of
D. Mineral salts lactate dehydrogenase in blood. What is enzyme
E. Uric acid should be expected in this case?
A. LDH1
23. A 1 y.o. child with symptoms of muscle B. LDH2
affection was admitted to the hospital. C. LDH3
Examination revealed carnitine deficit D. LDH4
inmuscles. Biochemical base of this pathology E. LDH5
is disturbed process of:

70
hange E hange E
XC di XC di
F- t F- t
PD

PD
or

or
!

!
W

W
O

O
N

N
Y

Y
U

U
B

B
to

to
ww

ww
om

om
k

k
lic

lic
C

C
.c

.c
Biochemistry of connective tissue
w

w
tr re tr re
.

.
ac ac
k e r- s o ft w a k e r- s o ft w a

1. Examination of a patient revealed typical C. Nucleotides


presentations of collagenosis. This pathology is D. Vitamins
characterized by increase of the following urine E. Phospholipids
index:
A. Hydroxyproline 6. A patient in the dental department has been
B. Arginine diagnosed with Paget's disease, which is
C. Glucose accompanied by collagen degradation. The
D. Mineral salts decisive fact for the diagnosis was the detection
E. Ammonium salts in the patient's urine of an elevated level:
A. Hydroxproline
2. A child has physical and mental retardation, B. Arginine
serious abnormalities in connective tissue of C. Tryptophan
internal organs; urine contains keratan sulfates. D. Serina
This is caused by metabolic disorder of the E. Alanina
following substance:
A. Glycosaminoglycan 7. A 60 year old man complains about in the
B. Collagen joints. In the serum of the patient, the
C. Elastin concentration of C-reactive protein and
D. Fibronectin oxyproline was found to increase. what disease
E. Hyaluronic acid these symptoms can be associated with:
A. Rheumatism
3. Inherited diseases, such as B. Gout
mucopolysaccharidoses, are manifested in C. Hepatitis
metabolic disorders of connective tissue, bone D. Jaundice
and joint pathologies. The sign of this disease is E. Diabetes
the excessive urinary excretion of the following
substance: 8. A 36-year-old female patient has a history of
A. Glycosaminoglycans collagen disease. Urine analysis is likely to
B. Amino acids reveal an increased concentration of the
C. Glucose following metabolite:
D. Lipids A. Oxyproline
E. Urea B. Indican
C. Creatinine
4. A 28 year old pregnant woman had the D. Urea
enzymes in the cells of amniotic fluid analyzed. E. Urobilinogen
The analysis revealed insufficient activity of β-
glucuronidase. What pathological process is it? 9. A 30-year-old woman first developed pain,
A. Mucopolysaccharidosis swelling, and skin redness in the area of joints
B. Glycogenosis about a year ago. Provisional diagnosis is
C. Aglycogenosis rheumatoid arthritis. One of the likely causes of
D. Collagenosis this disease is change in the structure of the
E. Lipidosis following connective tissue protein:
A. Collagen
5. The five year old boy was observed small B. Mucin
stature, mental retardation, limited movement, C. Myosin
rough facial features. These features have D. Ovalbumin
become noticeable from 18 months of age. He E. Troponin
was diagnosed with a deficiency of L-
iduronidase. The exchange of which compounds 10. Collagenosis patients typically present with
is broken? connective tissue destruction processes.The
A. Glycosminoglycans presence of the seprocesses can beconfirmed by
B. Proteins the increasein:
71
hange E hange E
XC di XC di
F- t F- t
PD

PD
or

or
!

!
W

W
O

O
N

N
Y

Y
U

U
B

B
to

to
ww

ww
om

om
k

k
lic

lic
C

C
.c A. Blood oxyproline and oxylysine alkaline phosphatase in the intercellular

.c
w

w
tr re tr re
.

.
ac ac
k e r- s o ft w a k e r- s o ft w a

B. Blood creatine and creatinine substance. What cell produces this enzyme?
C. LDH-isoenzyme activity in the blood A. Osteoblast
D. Transaminase activity in the blood B. Osteocyte
E. Blood urates C. Osteoclast
D. Chondroblast
11. A 53-year-old male patient is diagnosed E. Chondrocyte
with Paget’s disease. The concentration of
oxyproline in daily urine is sharply increased, 16. When a wound heals, a scar takes its place.
which primarily means intensified disintegration What substance is the main component of its
of: connective tissue?
A. Collagen A. Collagen
B. Keratin B. Elastin
C. Albumin C. Keratan sulfate
D. Hemoglobin D. Chondroitin sulfate
E. Fibrinogen E. Hyaluronic acid

12. A 63-year-old woman developed symptoms 17. A patient with systemic scleroderma has an
of rheumatoid arthritis. Their increase of which intensified collagen destruction. Collagen
blood values indicators could be the most destruction will be reflected by intensified
significant in proving the diagnosis? urinary excretion of the following amino acid:
A. Additive glycosaminoglycans A. Oxyproline
B. Lipoproteids B. Alanine
C. Acid phosphatase C. Tryptophane
D. General cholesterol D. Serine
E. R-glycosidase E. Phenylalanine

13. Osteolaterism is charcterized by a decrease 18. A patient who is ill with scurvy displays
in collagen strength caused by much less disturbed processes of connective tissue
intensive formation of crosslinks in collagen formation that leads to loosening and falling of
fibrils. This phenomenon is caused by the low teeth. Disturbed activity of what enzyme causes
activity of the following enzyme: these symptomps?
A. Lysyl oxidase A. Lisilhydroxylase
B. Monoamino-oxidase B. Glycosiltransferase
C. Prolyl hydroxylase C. Elastase
D. Lysyl hydroxylase D. Procollagenpeptidase of N-terminal peptide
E. Collagenase E. Procollagenpeptidase of C-terminal peptide

14. A 34-year-old patient has a history of 19. In spring a patient experiences petechial
periodontitis. As a result of increased collagen haemorrhages, loosening of teeth, high liability
degradation, there is a significantly increased to colds. A doctor supposes hypobitaminosis C.
urinary excretion of one of the amino acids. In this respect loosening of teeth can be
Which one? explained by:
A. Hydroxyproline A. Structural failure of collagen in the
B. Valine periodontal ligaments
C. Alanine B. Structural change of glycosaminoglycan
D. Glycine C. Increased permeability of periodont
E. Serine membranes
D. Mechanical damage of teeth
15. Calcification of the intercellular substance E. Disturbed oxidation-reduction process in the
of bone tissue is accompanied by the deposition periodont
of hydroxyapatite crystals along the collagen
fibers. This process requires the presence of

72
hange E hange E
XC di XC di
F- t F- t
PD

PD
or

or
!

!
W

W
O

O
N

N
Y

Y
U

U
B

B
to

to
ww

ww
om

om
k

k
lic

lic
C

C
.c 20. Increased fragility of vessels, enamel and B. Formation of polypeptide chains

.c
w

w
tr re tr re
.

.
ac ac
k e r- s o ft w a k e r- s o ft w a

dentine destruction resulting from scurvy are C. Glycosylation of hydroxylysine residues


caused by disorder of collagen maturation. What D. Removal of C-ended peptide from
stage of procollagen modification is disturbed procollagen
under this avitaminosis? E. Detaching of N-ended peptide
A. Hydroxylation of proline

73
hange E hange E
XC di XC di
F- t F- t
PD

PD
or

or
!

!
W

W
O

O
N

N
Y

Y
U

U
B

B
to

to
ww

ww
om

om
k

k
lic

lic
C

C
.c

.c
Biochemistry of tooth and saliva
w

w
tr re tr re
.

.
ac ac
k e r- s o ft w a k e r- s o ft w a

1. What enzyme has a demineralizing effect - 6. The periodontal dentist must be assessed in
enhances the cleavage of the mineral the patient by factors of non-specific resistance
components of the tissues in the tooth? of saliva and excretory mucous membrane of
A. Sulfate phosphatase the oral cavity. What factor of non-specific
B. Alkaline phosphatase resistance should first be studied in the
C. Glucose-6-phosphatase investigated material?
D. Glucogenphosphorylase A. Lysozyme
E. Phosphotransferase B. Secretory IgA
C. Properdine
2. The hydroxyapatite crystals are deposited D. Interferon
along the collagen fibers in the process of E. Complement
grafting the intercellular substance of the tissue.
In order to realize this process, the presence of 7. What substance gives slime a viscous mucous
intercellular alkaline phosphatase must be character, performs a protective role and
present. What kind of creatine produces this including from mechanical damage to the oral
enzyme? mucosa?
A. Osteoblast A. Mutsin
B. Osteocyte B. Glucose
C. Osteoclasts C. Kalikrein
D. Hondroblaste D. Amilaza
E. Chondrocyte E. Lysozyme

3. The pH of saliva is 6.4 - 7.8 it is in the norm. 8. Some beards of saliva perform a protective
What changes in the enamel leads to a shift in function. Which one protects the oral mucosa
the pH of saliva in the sour beet (less than 6.2)? from mechanical damage?
A. Demineralization A. Mutsin
B. Calcification B. Lysozyme
C. Fluorosis C. Catalase
D. Mineralization D. Peroxidase
E. Strengthening of stability E. Renin

4. Cationic glycoproteins are the main 9. The patient with chronic inflammation of
components of salivation of the parotid glands. submaxilar salivary glands have observed
What Amino Acids cause their positive charge? giposalivation. What violation of the increment
A. Lizin, arginine, hististine of which biologically active substance is
B. Aspartate, glutamate, glucine observed at this process?
C. Aspartate, arginine, glutamate A. Parotin
D. Glutamate, valine, leucine B. Calcitonin
E. Cysteine, glucine, proline C. Paratyrin
D. Gluckagon
5. The protective function of saliva is due to E. Somatostatin
several mechanisms, including the presence of
an enzyme that has bactericidal action, causes 10. Patient with symptoms of reduced excretory
lysis of the polysaccharide complex of the function of the kidneys, an unpleasant smell
staphylococcal shell, streptococci. Indicate this from the mouth is indicated. What substance are
enzyme: increased excretion of the salivary glands is the
A. Lysozyme cause of this?
B. α-amylase A. Urea
C. Oligo-1,6-glucosidase B. Alpha amylase
D. Collagenase C. Lysozyme
E. β-glucuronidase D. Phosphatase
E. Mutsin
74
hange E hange E
XC di XC di
F- t F- t
PD

PD
or

or
!

!
W

W
O

O
N

N
Y

Y
U

U
B

B
to

to
ww

ww
om

om
k

k
lic

lic
C

C
.c 16. A 60 year old patient was found to havetra cak e r- s o ft w are

.c
w

w
tr re
.

.
ac
k e r- s o ft w a

11. The child has acute renal insufficiency. dysfunction of main digestive enzyme of saliva.
What biochemical indicators of saliva can be This causes the disturbance of primary
confirmed? hydrolysis of:
A. Increasing the level of residual nitrogen A. Carbohydrates
B. Increase in immunoglobulin A. B. Fats
C. Reduction of alkaline phosphatase C. Proteins
D. Increase in alpha amylase D. Cellulose
E. Reducing the phosphate level E. Lactose

12. Periodontitis is accompanied by activation 17. It is necessary to evaluate the digestible


of proteolysis in periodontal tissues. The properties of saliva. With what substrate for this
increase of which oral fluid component testifies it needs to be shifted?
to the activation of proteolysis? A. Starch
A. Amino acids B. Casein
B. Organic acids C. Fat
C. Glucose D. DNA
D. Biogenic Amines E. RNA
E. Cholesterol
18. When investigating human saliva it is
13. When treated with hydrogen peroxide in the necessary to assessits hydrolytic properties.
mucous membrane of the patient's oral cavity, What substance should be used as a substrate in
the blood was painted in a brown color instead the process?
of pene formation. When the concentration of A. Starch
any of the enzymes listed below is reduced, is it B. Proteins
possible? C. Fats
A. Catalase D. Fiber
B. Pseudocholinesterase E. Amino acids
C. Glucose-6-phosphate dehydrogenase
D. Acetyltransferase 19. Afther using cookies, sweets in mixed
E. Methemoglobin reductase saltines temporarily increases the level of
lactate. The activation of which biomic process
14. In periodontites, lipid peroxidation develops leads to this?
in the periodontal tissues, and the content of A. Anaerobic digestion
malonic dialdehyde, hydrogen peroxide B. Tissue breathing
increases in the oral cavity. Which of the C. Aerobic Glucose
enzymes is antioxidant protection? D. Gluconeogenesis
A. Superoxide dismutase, catalase E. Microsomal oxidation
B. Amilase, trypsin
C. Maltase, Chymotrypsin 20. In the saliva of patient is increased content
D. Lactase, lysozyme of lactate was detected. Activation of a process
E. Saharaza, prothrombin. is the main reason for the increase of lactate?
A. Anaerobic decomposition of glucose
15. With age, the activity of the peripheral B. Aerobic decomposition of glucose
salivary glands decreases. Which does an C. Decomposition of glycogen
enzyme's activity decrease in slime? D. Hydrolysis of carbohydrates
A. Amilaza E. Glucose-lactate cycle
B. Lysozyme
C. Phosphatase 21. To form a mineral matrix of hard tooth
D. Geksokinase tissues, a high concentration of phosphate ions
E. Malthus is required, which is formed during the process
of hydrolysis of phosphorous-bonded bonds
with the participation of alkaline phosphatase.

75
hange E hange E
XC di XC di
F- t F- t
PD

PD
or

or
!

!
W

W
O

O
N

N
Y

Y
U

U
B

B
to

to
ww

ww
om

om
k

k
lic

lic
C

C
.c What ions of the metal are the triggers of this D. Calcium chloride

.c
w

w
tr re tr re
.

.
ac ac
k e r- s o ft w a k e r- s o ft w a

process: E. Kalia bromide


A. Zinc
B. Magnium 27. A patient with renal insufficiency developed
C. Iron osteodystrophy, which is accompanied by
D. Calcium intensive demineralization of the hips. Which
E. Natrium active form of vitamin violation’s of the
formation is the cause of this complication?
22. In economically developed countries, A. Calcipherol
common caries is a cureus of teeth. This disease B. Retinol
affects more than 95% of the population. What C. Thiamin
role plays in the demineralization of hard tooth D. Naftohinon
tissues in caries? E. Ryboflavin
A. Organic acids
B. Malnutrition 28. The enamel is characterized by high stability
C. Extreme effects on the organism to the action of various mechanical and
D. Disturbance of regulation of metabolism chemical factors. Which component is synthesis
E. Insufficient vitamin C of provides such resistance?
A. Fluorapatite
23. The high excess concentration of glucose in B. Hydroxyapatite
oral solution in diabetes leads to development: C. Chlorapatite
A. Multiple caries D. Collagen
B. Hyperplasia enamel E. Carbonate apatite
C. Hypoplasia of enamel
D. Fluorosis 29. Calcification of dental tissues is
E. Enhanced calcification of enamel significantly influenced by osteocalcin protein
which has an ability to bind calcium ions due to
24. In the Transcarpathian settlement, after the presence of the following modified amino
prophylactic examination of many children have acid residues in the polypeptide chain:
found a multiple of caries. With mineral of the A. γ-carbon glutamine
insufficiency in food you can link development B. Alanine
of careers? C. γ-aminobutyric
A. Fluoride D. Carboxy aspargine
B. Iodine E. δ-aminopropionic
C. Molibden
D. Iron 30. A 35-year-old patient consulted a dentist
E. Cobalt about low density of dental tissues, increased
fragility of teeth on eating solid food. In order to
25. A child has disturbed processes of determine Ca/P relation a scrape of enamel was
ossification and "punctate"enamel. What sent to the laboratory. What value of this index
microelement metabolism is disturbed? is suggestive of intensified demineralization?
A. Fluorine A. 0,9
B. Iron B. 1,67
C. Zinc C. 1,85
D. Chromium D. 2,5
E. Copper E. 1,5

26. A saline metal was assigned for salvage 31. The preparation comlex for periodontitis
therapy at the initial caries of the teeth. Which treatment includes the medicine from the group
drug is determinated of this process? of water soluble vitamins, bioflavonide
A. Sodium fluoride derivative, which is prescribed together with
B. Sodium bromide ascorbic acid. This preparation has anti-
C. Sodium chloride

76
hange E hange E
XC di XC di
F- t F- t
PD

PD
or

or
!

!
W

W
O

O
N

N
Y

Y
U

U
B

B
to

to
ww

ww
om

om
k

k
lic

lic
C

C
.c oxidative properties, decreases gingival E. Nicotinamide

.c
w

w
tr re tr re
.

.
ac ac
k e r- s o ft w a k e r- s o ft w a

hemorrhage. What preparation is meant?


A. Rutin 36. The child has a delayed teething, their
B. Calcium pantothenate wrong arrangement, upon examination
C. Calcium panganate noticeable dryness of the oral cavity, in the
D. Cyanocobalamin corners of the mouth there are cracks with
E. Folic acid suppuration. With the lack of the vitamin this
condition may be related?
32. Examination of a child who hasn’t got fresh A. Vitamin D.
fruit and vegetables during winter revealed B. Vitamin C.
numerous subcutaneous hemorrhages, C. Vitamin E.
gingivitis, carious cavities in teeth. What D. Vitamin K.
vitamin combination should be prescribed in E. Vitamin A.
this case?
A. Ascorbic acid and rutin 37. In a child of the first year of life is observed
B. Thiamine and pyridoxine an increase in the size of the head and abdomen,
C. Folic acid and cobalamin late teething, violation of the enamel structure.
D. Riboflavin and nicotinamide The consequence of vitamin deficiencies are
E. Calciferol and ascorbic acid these changes?
A. Hypovitaminosis D
33. The patient has an increase in the B. Hypovitaminosis C
permeability of the walls of blood vessels with C. Hypovitaminosis A
the development of increased bleeding and the D. Hypovitaminosis B1
appearance of small point hemorrhages on the E. Hypovitaminosis B2
skin, tooth loss. How does the vitamin exchange
disorder explain these symptoms? 38. A child with renal insufficiency exhibits
A. Hypovitaminosis C delayed teeth eruption. This is mostlikely
B. Hypervitaminosis D caused by the abnormal formationof the
C. Hypervitaminosis C following substance:
D. Hypovitaminosis D A. 1,25 (OH)2D3
E. Hypovitaminosis A B. Glycocyamine
C. Glutamate
34. In examining the oral cavity of the patient, D. α-ketoglutarate
the dentist paid attention to the presence of an E. Hydroxylysine
inflammatory and dystrophic process in the
mucous membrane (Hunter’s glossitis, atrophic 39. A patient has enamel erosion. What vitamin
stomatitis). A blood test revealed hyperchromic should be administered for itstreatment?
anemia. What factor is the cause of this disease? A. D3
A. Hypovitaminosis B12 B. C
B. Hypovitaminosis B1 C. K
C. Hypovitaminosis B6 D. B1
D. Increasing pH of the gastric juice E. PP
E. Hypovitaminosis A
40. A 5 years old child has insufficient
35. 10 month old child has high excitability, calcification of enamel, tooth decay. Which
sleep disturbance, amyotonia, retarded dentition, vitamin hypovitaminosis leads to the
teeth erupt withinadequate enamel calcification. development of this process?
Thesechanges are caused by deficiency of A. Calciferol
thefollowing vitamin: B. Tocopherol
A. Cholecalciferol C. Biotin
B. Riboflavin D. Nicotinic acid
C. Thiamine E. Folic acid
D. Retinol

77
hange E hange E
XC di XC di
F- t F- t
PD

PD
or

or
!

!
W

W
O

O
N

N
Y

Y
U

U
B

B
to

to
ww

ww
om

om
k

k
lic

lic
C

C
.c 41. Hormonal form of a certain vitamin induces B. Calciferol

.c
w

w
tr re tr re
.

.
ac ac
k e r- s o ft w a k e r- s o ft w a

genome level synthesis of Ca binding proteins C. Thiamine


and enterocytes thus regulating the intestinal D. Biotin
absorption of Ca2+ ions required for dental E. Phyloquinone
tissuedevelopment. What vitamin is it?
A. D3 44. There are various diseases that cause sharp
B. A increase of active oxygen, leading tocell
C. B1 membranes destruction. Antioxidantsare used to
D. E prevent it from happening. Themost potent
E. K natural antioxidant is:
A. Alpha-tocopherol
42. While the examination of patient’s oral B. Glycerol
cavity the dentist found xerostomia, numerous C. Vitamin D
erosions. What vitamin deficit caused this D. Fatty acids
effect? E. Glucose
A. Vitamin A
B. Vitamin K 45. When treating sialadenitis (inflammation of
C. Vitamin P the salivary glands), preparations of vitamins
D. Vitamin H are used. Which of the following vitamins plays
E. Vitamin PP an important role in antioxidant defense?
A. Tocopherol
43. In order to prevent gum inflammation and to B. Pantothenic acid
improve regeneration of epithelial periodontium C. Riboflavin
cells manufacturers add to the tooth pastes one D. Thiamine
of the following vitamins: E. Pyridoxine
A. Retinol

78

You might also like